You are on page 1of 173

MIT OpenCourseWare http://ocw.mit.

edu

18.102 Introduction to Functional Analysis


Spring 2009

For information about citing these materials or our Terms of Use, visit: http://ocw.mit.edu/terms.

LECTURE NOTES FOR 18.102, SPRING 2009

Lecture 1. Tuesday, 3 Feb. Linear spaces, metric spaces, normed spaces. Banach spaces. Examples Eu clidean spaces, continuous functions on a closed interval C 0 ([0, 1]) with supremum norm. The (Riemannian) L1 norm, outline that this is not complete on C 0 ([0, 1]). Brief description of l2 This is Hilbert space, but not explained. What is it all for? Main aims:- To be able to carry out standard constructions in (linear) functional analysis: Abstract Hilbert space one in each dimension Concrete Hilbert space Many, such as L2 ([0, 1]) Example of a theorem:- The Dirichlet problem. Let V : [0, 1] R be a real-valued function. We are interested in oscillating modes on the interval; something like this arises in quantum mechanics for instance. Namely we want to know about functions u(x) twice continuously dierentiable on [0, 1] which satisfy the dierential equation d2 u (x) + V (x)u(x) = u(x) dx2 where is an unknown constant that is we want to know which s can occur. Well, of course all s can occur with u 0 but this is the trivial solution which will always be there for such an equation. What other solutions are there? Well, there is an innite sequence of s for which there is a non trivial solution of (1.1) j R they are all real no non-real complex s can occur. For each of these there is at least one (and maybe more) independent solution uj . We can say a lot more about everything here but one main aim of this course is to get at least to this point. Now, in fact (1.1) is just the eigenvalue equation. What we are dealing with here is an innite matrix. This is not obvious, and in fact is not a very good way of looking at things (there was such a matrix approach to quantum mechanics in the early days but it was replaced by the sort of operator theory on Hilbert space that we will use here.) Still, we are in some sense dealing with innite dimensional matrices. One of the crucial dierences between innite and nite dimensional settings is that topology is encountered. This is enshrined here in the notion of a normed linear space Linear space:- Should I break out the axioms? It is a space V in which we can add elements and multiply by scalars with rules very similar to the basic examples of Rn or Cn . Note that for us the scalars are either the real numbers of the complex numbers usually the latter. Lets be neutral and denote by K either R or C but of course consistently. Then our set V the set of vectors with which we will deal, comes with two laws. These are maps (1.1) (1.2) + : V V V, : K V V.

which we denote not by +(v, w) and (s, v) but by v + w and sv. Then we impose the axioms of a vector space look them up! These are commutative group axioms for + axioms for the action of K and the distributive law. Our examples: The trivial case of a nite dimensional vector space. The lp spaces. These are spaces of sequences the rst problem set is all about them. Thus l2 which is a Hilbert space consists of all the sequences

LECTURE NOTES FOR 18.102, SPRING 2009

a : N C also denoted {aj } where aj = a(j), such that j=1 (1.3)


j=1

|aj |2 < .

Seriously non-trivial examples such as C([0, 1]) the space of continuous func tions on [0, 1] (say with complex values). All these vector spaces carry norms, and that is what we want to talk about for most of the semester. Denition 1. A norm on a vector space V is a function (1.4) : V [0, ) which satises the three properties: (1) v = 0 i v = 0. (2) tv = |t|v for all t K and v V. (3) Triangle inequality. v + w v + w for all v, w V. Then show that d(v, w) = v w is a metric on V. It follows that all the notions from 18.100 come into play open sets, balls, closed sets, convergence of sequences, compact sets, connected sets, complete metric spaces. We will use them all! Denition 2. A normed space which is complete with respect to the induced metric is called a Banach space. I then discussed the supremum norm on C([0, 1]); (1.5) u = sup |u(x)|.
x[0,1]

I said, but did not prove, that it this gives a Banach space. I also disussed the L1 norm on C([0, 1]) : 1 (1.6) uL1 = |u(x)|dx
0

and indicated why it is not complete. This is basically why we need to study the Lebesgue integral. One might ask is the space of Riemann integrable functions on [0, 1] complete with respect to the L1 norm indeed someone did. The answer is that uL1 is not even a norm on the space of Riemann integable functions. Namely it is only a seminorm the second two conditions are ne but there are non-zero functions with integral zero. So, the question is not quite precise, which is why I did not talk about it. One the other hand one can ddle with the space (take a quotient) so that one gets a norm and then it is not complete. So the morally correct answer is NO, it is not a Banach space and this is true for a better (or worse) reason than it simply not being a normed space in the rst place! We will get to this.

LECTURE NOTES FOR 18.102, SPRING 2009

Problem set 1, Due 11AM Tuesday 10 Feb. Full marks will be given to anyone who makes a good faith attempt to answer each question. The rst four problems concern the little L p spaces lp . Note that you have the choice of doing everything for p = 2 or for all 1 p < . Problem 1.1 Write out a proof (you can steal it from one of many places but at least write it out in your own hand) either for p = 2 or for each p with 1 p < that lp = {a : N C; |aj |p < , aj = a(j)}
j=1

is a normed space with the norm


1 p ap = |aj |p .

j=1

This means writing out the proof that this is a linear space and that the three conditions required of a norm hold. Problem 1.2 The tricky part in Problem 1.1 is the triangle inequality. Suppose you knew meaning I tell you that for each N 1 p N |aj |p is a norm on CN
j=1

would that help? Problem 1.3 Prove directly that each lp as dened in Problem 1.1 or just l2 is complete, i.e. it is a Banach space. At the risk of oending some, let me say that this means showing that each Cauchy sequence converges. The problem here is to nd the limit of a given Cauchy sequence. Show that for each N the sequence in CN obtained by truncating each of the elements at point N is Cauchy with respect to the norm in Problem 1.2 on CN . Show that this is the same as being Cauchy in CN in the usual sense (if you are doing p = 2 it is already the usual sense) and hence, this cut-o sequence converges. Use this to nd a putative limit of the Cauchy sequence and then check that it works. Problem 1.4 Consider the unit sphere in lp where if you want you can set p = 2. This is the set of vectors of length 1 : S = {a lp ; ap = 1}. (1) Show that S is closed. (2) Recall the sequential (so not the open covering denition) characterization of compactness of a set in a metric space (e.g. by checking in Rudin). (3) Show that S is not compact by considering the sequence in lp with kth element the sequence which is all zeros except for a 1 in the kth slot. Note that the main problem is not to get yourself confused about sequences of sequences! Problem 1.5 Show that the norm on any normed space is continuous.

MIT OpenCourseWare http://ocw.mit.edu

18.102 Introduction to Functional Analysis


Spring 2009

For information about citing these materials or our Terms of Use, visit: http://ocw.mit.edu/terms.

LECTURE NOTES FOR 18.102, SPRING 2009

Lecture 2. Thursday, 5 Feb. Plan:- Linear maps between normed spaces are continuous i they are bounded. The best bound gives a norm. If the second space is Banach the space of linear operators is Banach. Corollary the dual space of a normed space is a Banach space. Examples: Integral operators on C 0 ([0, 1]) with respect to supremum or L1 norms. Dierentiation as an operator from C 1 ([0, 1]) to C 0 ([0, 1]) In practice:- I did not get to the part about dierentiation. Reading: (1) Wilde:- Chapter 2 to 2.7 (2) Chen:- First part of Chapter 6 and of Chapter 7. (3) Ward:- Chapter 3, rst 2 sections.

MIT OpenCourseWare http://ocw.mit.edu

18.102 Introduction to Functional Analysis


Spring 2009

For information about citing these materials or our Terms of Use, visit: http://ocw.mit.edu/terms.

LECTURE NOTES FOR 18.102, SPRING 2009

Lecture 3. Tuesday, 10 Feb. Recalled the proof from last time that the bounded operators from a normed space into a Banach space form a Banach space mainly to suggest that it is not so hard to remember how such a proof goes. Then proved that a normed space is Banach i every absolutely summable series is convergent. Absolute summability means that the sum of the norms is nite. Then did most of the proof that every normed space can be completed to a Banach space using this notion of absolutely summable sequences. The last part and a guide to how to attempt the part of the proof that is the rst question on the next homework. The proof of the result about completeness from the early part of the leture is in (1) Wilde:- Proposition 1.6 (2) Chen:- I didnt nd it. (3) Ward:- Lemma 2.1 (easy way only) Here is a slightly abbreviated version of what I did in lecture today on the completion of a normed space. The very last part I asked you to nish as the rst part of the second problem set, not due until February 24 due to the vagaries of the MIT calendar (but up later today). This problem may seem rather heavy sledding but if you can work through it all you will understand, before we get to it, the main sorts of arguments needed to prove most of the integrability results we will encounter later. Let V be a normed space with norm V . A completion of V is a Banach space B with the following properties: (1) There is an injective (1-1)linear map I : V B (2) The norms satisfy (3.1) I(v)B = vV v V. (3) The range I(V ) B is dense in B. Notice that if V is itself a Banach space then we can take B = V with I the identity map. So, the main result is: Theorem 1. Each normed space has a completion. Proof (the last bit is left to you). First we introduce the rather large space (3.2) V = {uk }k=1 ; uk V and uk <
k=1

the elements of which I called the absolutely summable series in V. Now, I showed in the earlier result that each element of V is a Cauchy sequence N meaning the corresponding sequence of partial sums vN = uk is Cauchy if
k=1

{uk } is absolutely summable. Now V is a linear space, where we add sequences, and multiply by constants, by doing the operations on each component: (3.3) t1 {uk } + t2 {u } = {t1 uk + t2 u }.
k k This always gives an absolutely summable series by the triangle inequality:
(3.4) t1 uk + t2 u |t1 | uk + |t2 | u . k k
k k k

LECTURE NOTES FOR 18.102, SPRING 2009

Within V consider the linear subspace uk < , uk = 0 (3.5) S = {uk };


k k

of those which converge to 0. As always for a linear subspace of a linear space we can form the quotient (3.6) B = V /S the elements of which are the cosets of the form {uk } + S V where {uk } V . We proceed to check the following properties of this B. (1) A norm on B is dened by (3.7) bB = lim
n n k=1

uk , {uk } b.

(2) The original space V is imbedded in B by (3.8) V v I(v) = {uk } + S, u1 = v, uk = 0 k > 1 and the norm satises (3.1). (3) I(V ) B is dense. (4) B is a Banach space with the norm (3.7). So, rst that (3.7) is a norm. The limit on the right does exist since the limit of the norm of a Cauchy sequence always exists namely the sequence of norms is itself Cauchy but now in R. Moreover, adding an element of S to {uk } does not change the norm of the sequence of partial sums, since the addtional term tends to zero in norm. Thus bB is well-dened for each element b B and bB = 0 means exactly that the sequence {uk } used to dene it tends to 0 in norm, hence is in S hence b = 0. The other two properties of norm are reasonably clear, since if b, b B are represented by {uk }, {u } in V then tb and b + b are reprented by k {tuk } and {uk + u } and k
n

lim

n k=1

tuk = |t| lim


n

n k=1

uk ,

lim

(uk + u ) = A = k
n

k=1

(3.9)

for > 0 N s.t. n N, A A


n k=1

(uk + u ) = k

k=1

uk +

u ) n N = k > 0 =

A bB +

k=1 b B

b + b B bB + b B . Now the norm of the element I(v) = v, 0, 0, , is the limit of the norms of the sequence of partial sums and hence is vV so I(v)B = vV and I(v) = 0 therefore implies v = 0 and hence I is also injective. So, we need to check that B is complete, and also that I(V ) is dense. Here is an extended discussion of the diculty of course maybe you can see it directly

10

LECTURE NOTES FOR 18.102, SPRING 2009

yourself (or have a better scheme). Note that I want you to write out your own version of it carefully for the next problem set. Okay, what does it mean for B to be a Banach space, well as we saw in class today it means that every absolutely summable series in B is convergent. Such a (n) (n) series {bn } is given by bn = {uk } + S where {uk } V and the summability condition is that N (n) (3.10) > bn B = lim uk V .
n n N k=1

So, we want to show that

bn = b converges, and to do so we need to nd the

limit b. It is supposed to be given by an absolutely summable series. The problem (n) is that this series should look like uk in some sense because it is supposed
n k

to represent the sum of the bn s. Now, it would be very nice if we had the estimate (n) (3.11) uk V <
n k

since this should allow us to break up the double sum in some nice way so as to get an absolutely summable series out of the whole thing. The trouble is that (3.11) need not hold. We know that each of the sums over k for given n converges, but not the sum of the sums. All we know here is that the sum of the limits of the norms in (3.10) converges. So, that is the problem! One way to see the solution is to note that we do not (n) have to choose the original {uk } to represent bn we can add to it any element (n) of S. One idea is to rearrange the uk I am thinking here of xed n so that it converges even faster. Given > 0 we can choose N1 so that for all N N1 , (N ) (n) (3.12) | uk V bn B | , uk V .
kN kN

Then in fact we can choose successive Nj < Nj1 (remember that little n is xed here) so that (N ) (n) (3.13) | uk V bn B | 2j , uk V 2j .
kNj kNj (n) N1 k=1 (n) (n) Nj k=Nj1 (n)

Now, resum the series dening instead v1

uk , vj

uk

and do

this setting = 2n for the nth series. Check that now (n) (3.14) vk V < .
n k (n)

Of course, you should also check that bn = {vk } + S so that these new summable series work just as well as the old ones. After this ddling you can now try to nd a limit for the sequence as (p) (3.15) b = {wk } + S, wk = vl V.
l+p=k

So, you need to check that this {wk } is absolutely summable in V and that bn b as n . I may add some more to discussion of completeness if needed.

LECTURE NOTES FOR 18.102, SPRING 2009

11

Finally then there is the question of showing that I(V ) is dense in B. You can do this using the same idea as above in fact it might be better to do it rst. Given an element b B we need to nd elements in V, vk such that I(vk ) bB 0 as Nj k . Take an absolutely summable series uk representing b and take vj = uk
k=1

where the Nj s are constructed as above and check that I(vj ) b by computing (3.16) I(vj ) bB = lim u p V up V .
p p>Nj p>Nj

12

LECTURE NOTES FOR 18.102, SPRING 2009

Problem set 2, Due 11AM Tuesday 24 Feb. I was originally going to make this problem set longer, since there is a missing Tuesday. However, I would prefer you to concentrate on getting all four of these questions really right! Problem 2.1 Finish the proof of the completeness of the space B constructed in lecture on February 10. The description of that construction can be found in the notes to Lecture 3 as well as an indication of one way to proceed. Problem 2.2 Lets consider an example of an absolutely summable sequence of step functions. For the interval [0, 1) (remember there is a strong preference for left-closed but right-open intervals for the moment) consider a variant of the construction of the standard Cantor subset based on 3 proceeding in steps. Thus, remove the central interval [1/3, 2/3). This leave C1 = [0, 1/3) [2/3, 1). Then remove the central interval from each of the remaining two intervals to get C2 = [0, 1/9) [2/9, 1/3) [2/3, 7/9) [8/9, 1). Carry on in this way to dene successive sets Ck Ck1 , each consisting of a nite union of semi-open intervals. Now, consider the series of step functions fk where fk (x) = 1 on Ck and 0 otherwise. (1) Check that this is an absolutely summable series. (2) For which x [0, 1) does |fk (x)| converge?
k

(3) Describe a function on [0, 1) which is shown to be Lebesgue integrable (as dened in Lecture 4) by the existence of this series and compute its Lebesgue integral. (4) Is this function Riemann integrable (this is easy, not hard, if you check the denition of Riemann integrability)? (5) Finally consider the function g which is equal to one on the union of all the intervals which are removed in the construction and zero elsewhere. Show that g is Lebesgue integrable and compute its integral. Problem 2.3 The covering lemma for R2 . By a rectangle we will mean a set of the form [a1 , b1 ) [a2 , b2 ) in R2 . The area of a rectangle is (b1 a1 ) (b2 a2 ). (1) We may subdivide a rectangle by subdividing either of the intervals re placing [a1 , b1 ) by [a1 , c1 ) [c1 , b1 ). Show that the sum of the areas of rectangles made by any repeated subdivision is always the same as that of the original. (2) Suppose that a nite collection of disjoint rectangles has union a rectangle (always in this same half-open sense). Show, and I really mean prove, that the sum of the areas is the area of the whole rectange. Hint:- proceed by subdivision. (3) Now show that for any countable collection of disjoint rectangles contained in a given rectange the sum of the areas is less than or equal to that of the containing rectangle. (4) Show that if a nite collection of rectangles has union containing a given rectange then the sum of the areas of the rectangles is at least as large of that of the rectangle contained in the union. (5) Prove the extension of the preceeding result to a countable collection of rectangles with union containing a given rectangle. Problem 2.4

LECTURE NOTES FOR 18.102, SPRING 2009

13

(1) Show that any continuous function on [0, 1] is the uniform limit on [0, 1) of a sequence of step functions. Hint:- Reduce to the real case, divide the interval into 2n equal pieces and dene the step functions to take inmim of the continuous function on the corresponding interval. Then use uniform convergence. (2) By using the telescoping trick show that any continuous function on [0, 1) can be written as the sum (3.17) fj (x) x [0, 1)
i

where the fj are step functions and

|fj (x)| < for all x [0, 1).

(3) Conclude that any continuous function on [0, 1], extended to be 0 outside this interval, is a Lebesgue integrable function on R.

14

LECTURE NOTES FOR 18.102, SPRING 2009

Solutions to Problem set 1 Full marks will be given to anyone who makes a good faith attempt to answer each question. The rst four problems concern the little L p spaces lp . Note that you have the choice of doing everything for p = 2 or for all 1 p < . Everyone who handed in a script received full marks. Problem 1.1 Write out a proof (you can steal it from one of many places but at least write it out in your own hand) either for p = 2 or for each p with 1 p < that lp = {a : N C; |aj |p < , aj = a(j)}
j=1

is a normed space with the norm


1 p ap = |aj |p .

j=1

This means writing out the proof that this is a linear space and that the three conditions required of a norm hold. Solution:- We know that the functions from any set with values in a linear space form a linear space under addition of values (dont feel bad if you wrote this out, it is a good thing to do once). So, to see that lp is a linear space it suces to see that it is closed under addition and scalar multiplication. For scalar multiples this is clear: (3.18) |tai | = |t||ai | so tap = |t|ap

which is part of what is needed for the proof that p is a norm anyway. The fact that a, b lp imples a + b lp follows once we show the triangle inequality or we can be a little cruder and observe that |ai + bi |p (2 max(|a|i , |bi |))p = 2p max(|a|p , |bi |p ) 2p (|ai | + |bi |) i p p p (3.19) a + b = |a + b | 2 (ap + bp ),
p i i j

where we use the fact that tp is an increasing function of t 0. Now, to see that lp is a normed space we need to check that ap is indeed a norm. It is non-negative and ap = 0 implies ai = 0 for all i which is to say a = 0. So, only the triangle inequality remains. For p = 1 this is a direct consequence of the usual triangle inequality: (3.20) a + b1 = |ai + bi | (|ai | + |bi |) = a1 + b1 .
i i

For 1 < p < it is known as Minkowskis inequality. This in turn is deduced from Hlders inequality which follows from Youngs inequality! The latter says o if 1/p + 1/q = 1, so q = p/(p 1), then (3.21) p q + , 0. p q xp q x + p q

To check it, observe that as a function of = x, (3.22) f (x) =

LECTURE NOTES FOR 18.102, SPRING 2009

15

if non-negative at x = 0 and clearly positive when x >> 0, since xp grows faster than x. Moreover, it is dierentiable and the derivative only vanishes at xp1 = , where it must have a global minimum in x > 0. At this point f (x) = 0 so Youngs inequality follows. Now, applying this with = |ai |/ap and = |bi |/bq (assuming both are non-zero) and summing over i gives Hlders inequality o |ai |p |bi |q | ai bi |/ap bq |ai ||bi |/ap bq + =1 ap p bq q p q i i i (3.23) = | ai bi | ap bq .
i

Of course, if either ap = 0 or bq = 0 this inequality holds anyway. Now, from this Minkowskis inequality follows. Namely from the ordinary trian gle inequality and then Minkowskis inequality (with q power in the rst factor) (3.24) |ai + bi |p = |ai + bi |(p1) |ai + bi |
i i

|ai + bi |(p1) |ai | + |ai + bi |(p1) |bi |


i i

1/q |ai + bi |
p

(ap + bq )

gives after division by the rst factor on the right (3.25)


p

a + bp ap + bp .

Thus, l is indeed a normed space. I did not necessarily expect you to go through the proof of Young-Hldero Minkowksi, but I think you should do so at some point since I will not do it in class. Problem 1.2 The tricky part in Problem 1.1 is the triangle inequality. Suppose you knew meaning I tell you that for each N 1 p N |aj |p is a norm on CN
j=1

would that help? Solution:- Yes indeed it helps. If we know that for each N 1 1 1 p p p N N N (3.26) |aj + bj |p |aj |p + |bj |p
j=1 p j=1 j=1

then for elements of l the norms always bounds the right side from above, meaning 1 p N (3.27) |aj + bj |p ap + bp .
j=1

Since the left side is increasing with N it must converge and be bounded by the right, which is independent of N. That is, the triangle inequality follows. Really

16

LECTURE NOTES FOR 18.102, SPRING 2009

this just means it is enough to go through the discussion in the rst problem for nite, but arbitrary, N. Problem 1.3 Prove directly that each lp as dened in Problem 1.1 or just l2 is complete, i.e. it is a Banach space. At the risk of oending some, let me say that this means showing that each Cauchy sequence converges. The problem here is to nd the limit of a given Cauchy sequence. Show that for each N the sequence in CN obtained by truncating each of the elements at point N is Cauchy with respect to the norm in Problem 1.2 on CN . Show that this is the same as being Cauchy in CN in the usual sense (if you are doing p = 2 it is already the usual sense) and hence, this cut-o sequence converges. Use this to nd a putative limit of the Cauchy sequence and then check that it works. Solution:- So, suppose we are given a Cauchy sequence a(n) in lp . Thus, each (n) element is a sequence {aj } in lp . From the continuity of the norm in Problem j=1 1.5 below, a(n) must be Cauchy in R and so converges. In particular the sequence is norm bounded, there exists A such that a(n) p A for all n. The Cauchy condition itself is that given > 0 there exists M such that for all m, n > M, 1 p (n) (m) (3.28) a(n) a(m) p = |ai ai |p < /2.
i

Now for each i, a a(m) p so each of the sequences ai be Cauchy in C. Since C is complete (3.29)
(n) lim a n i

(n) |ai

(m) ai |

(n)

(n)

must

= ai exists for each i = 1, 2, . . . .

So, our putative limit is a, the sequence {ai } . The boundedness of the norms i=1 shows that N (n) (3.30) |ai |p Ap
i=1

and we can pass to the limit here as n since there are only nitely many terms. Thus N (3.31) |ai |p Ap N = ap A.
i=1

Thus, a lp as we hoped. Similarly, we can pass to the limit as m in the nite inequality which follows from the Cauchy conditions (3.32) to see that for each N (3.33) and hence (3.34) a(n) a < n > M. Thus indeed, a(n) a in lp as we were trying to show.
N (n) 1 ( |ai ai |p ) p /2 i=1

N i=1

|ai

(n)

ai

(m) p

| ) p < /2

LECTURE NOTES FOR 18.102, SPRING 2009

17

Notice that the trick is to back o to nite sums to avoid any issues of inter changing limits. Problem 1.4 Consider the unit sphere in lp where if you want you can set p = 2. This is the set of vectors of length 1 : S = {a lp ; ap = 1}. (1) Show that S is closed. (2) Recall the sequential (so not the open covering denition) characterization of compactness of a set in a metric space (e.g. by checking in Rudin). (3) Show that S is not compact by considering the sequence in lp with kth element the sequence which is all zeros except for a 1 in the kth slot. Note that the main problem is not to get yourself confused about sequences of sequences! Solution:- By the next problem, the norm is continuous as a function, so (3.35) S = {a; a = 1} is the inverse image of the closed subset {1}, hence closed. Now, the standard result on metric spaces is that a subset is compact if and only if every sequence with values in the subset has a convergent subsequence with limit in the subset (if you drop the last condition then the closure is compact). In this case we consider the sequence (of sequences) 0 i = n (n) (3.36) ai = . 1 i = n This has the property that a(n) a(m) p = 2 p whenever n = m. Thus, it cannot have any Cauchy subsequence, and hence cannot have a convergent subsequence, so S is not compact. This is important. In fact it is a major dierence between nite-dimensional and innite-dimensional normed spaces. In the latter case the unit sphere cannot be compact whereas in the former it is. Problem 1.5 Show that the norm on any normed space is continuous. Solution:- Right, so I should have put this problem earlier! The triangle inequality shows that for any u, v in a normed space (3.37) which implies that (3.38) |u v| u v.
This shows that is continuous, indeed it is Lipschitz continuous.
u u v + v, v u v + u
1

MIT OpenCourseWare http://ocw.mit.edu

18.102 Introduction to Functional Analysis


Spring 2009

For information about citing these materials or our Terms of Use, visit: http://ocw.mit.edu/terms.

18

LECTURE NOTES FOR 18.102, SPRING 2009

Lecture 4. Thursday, 12 Feb I talked about step functions, then the covering lemmas which are the basis of the denition of Lebesgue measure which we will do after the integral then properties of monotone sequences of step functions. To be denite, but brief, by an interval we will mean [a, b) an interval closed on the left and open on the right at least for a little while. This is just so the length of the interval, b a, only vanishes when the interval is empty (not true for closed intervals of course) and also so that we can decompose an interval, in this sense, into two disjoint intervals by choosing any interior point: (4.1) Now, by a step function (4.2) f : R C [a, b) = [a, t) [t, b), a < t < b.

(although often we will restrict to functions with real values) we mean a function which vanishes outside a nite union of disjoint intervals and is constant on each of them. Thus f (R) is nite the function only takes nitely many values and (4.3) f 1 (c) is a nite union of disjoint intervals, c = 0.
N i=1

It is also often convenient to write a step function as a sum (4.4) f= ci [ai ,bi )

of multiples of the characteristic functions of our intervals. Note that such a pre sentation is not unique but can be made so by demanding that the intervals be disjoint and maximal so f is does not take the same value on two intervals with a common endpoint. Now, a constant multiple of a step function is a step function and so is the sum of two step functions clearly the range is nite. Really this reduces to checking that the dierence [a, b) \ [a , b ) and the union of two intervals is always a union of intervals. The absolute value of a step function is also a step function. A similar argument shows that the integral, dened by (4.5) f= ci (bi ai )
R i

from (4.4) is independent of the presentation used to dene it. It is of course equal to the Riemann integral which is one way of seeing that it is well-dened (but of course the result is much more elementary). Proposition 1. The step functions on R in the sense dened above form a normed space with the L1 norm (4.6) f L1 = |f |.
R

So, we will complete this space instead of the continuous functions it is both more standard and a little easier. The fact that we can directly construct a con crete completion is due to Mikusiski. Already at this stage we can dene a n Lebesgue integrable function, however we need to do some work to esh out the denition.

LECTURE NOTES FOR 18.102, SPRING 2009

19

Denition 3. A function g : R C is Lebesgue integrable if there exists an absolutely summable sequence of step functions fn , i.e. satisfying (4.7) |fn | <
n

such that (4.8) f (x) =


n

fn (x) x R such that

|fn (x)| < .

So, the denition is a little convoluted there must exist a sequence of step functions the sum the integrals of the absolute values of which converges and such that the sum itself converges to the function but only at the points of absolute converge of the (pointwise) series. Tricky, but one can get used to it and ultimately simplify it. The main attraction of this denition is that it is self-contained and in principle everything can be deduced from it. Now, the rst thing we need is the covering lemma basically some properties of countable collections of our intervals such as will arise when we look at a sequence, or series, of step functions. I leave it to you to devise careful proofs of the following two facts. Lemma 1. If Ci = [ai , bi ), i = 1, . . . , N, is a nite collection of intervals then (4.9) Ci [a, b) i and Ci Cj = i = j =
N (bi ai ) (b a). i=1

On the other hand (4.10) [a, b)


N
i=1

Ci =

N (bi ai ) (b a). i=1

You can prove this by inserting division points etc. Now, what we want is the same thing for a countable collection of intervals. Proposition 2. If Ci = [ai , bi ), i N, is a countable collection of intervals then (4.11) or (4.12) [a, b)
N
i=1 (bi ai ) (b a) i=1 i=1

Ci [a, b) i and Ci Cj = i = j =

Ci =

(bi ai ) (b a).

Proof. You might think these are completely obvious, and the rst is the hypoth esis (4.9) holds for any nite subcollection and hence the nite sum is always less than the xed number b a and hence so is the innite sum which therefore converges. On the other hand (4.12) is not quite so obvious since it depends on Heine-Borel. To be able to apply (4.10) choose > 0. Now extend each interval by replacing the lower limit by ai 2i and consider the open intervals which therefore have the property
(4.13) [a, b ] [a, b) (ai 2i , bi ).
i

20

LECTURE NOTES FOR 18.102, SPRING 2009

Now, by Heine-Borel the compactness of closed bounded intervals a nite subcol lection of these open intervals covers [a, b ) so (4.10) does apply to the semi-open intervals and shows that for some nite N (hence including the nite subcollection) (4.14)
N (bi ai ) 2i b a = (bi ai ) b a 2 i=1 i=1

where the sum here might be innite, but then it is all the more true. The fact that this is true for all > 0 now proves (4.12). Welcome to measure theory. Okay, now the basis result on which most of the properties of integrable functions hinge is the following monotonicity lemma. Lemma 2. Let fn be a sequence of step functions which decreases monotonically to 0 (4.15) then (4.16) lim fn = 0. Note that decreases here means that fn (x) is, for each x, a non-increasing sequence which has limit 0. Of course this means that all the fn are non-negative. Moreover the rst one vanishes outside some interval [a, b) hence so do all of them. The crucial thing about this lemma is that we get the vanishing of the limit of the sequence of integrals without having to assume uniformity of the limit. Proof. Going back to the denition of the integral of step functions, clearly fn (x) fn+1 (x) for all x implies that fn is a decreasing (meaning non-increasing) se quence. So there are only two possibilities, it converges to 0, as we claim, or it converges to some positive value. This means that there is some > such that fn > for all n, so we just need to show that this is not so.
Given an > 0 consider the sets
(4.17) Sj = {x [a, b); fj (x) }. fn (x) 0 x R,

(Here [a, b) is an interval outside which f1 vanishes and hence all the fn vanish outside it). Each of the Sj is a nite union of intervals. Moreover
(4.18) Sj = [a, b)
j

since fn (x) 0 for each x. In fact the Sj increase with j and if we set B1 = S1 and (4.19) Bj = Sj+1 \ Sj

then the Bj are all disjoint, each consists of nitely many intervals and
(4.20) Bj = [a, b). Thus, both halves of Proposition 2 apply to the intervals forming the Bj . If we let l(Bj ) be the length of Bj the sum of the lengths of the nitely many intervals of

LECTURE NOTES FOR 18.102, SPRING 2009

21

which it is composed then (4.21)


j

l(Bj ) = b a.

Now, let A be such that f1 (x) A and hence it is an upper bound for all the fn s. We can choose N so large that (4.22) l(Bj ) < .
jN

Dividing the integral for fk , k N, into the part over SN and the rest we see that (4.23) fk (b a) + A. The rst estimate comes for the fact the fact that fk on SN , and the second that the total lengths of the remaining intervals is no more than (and the function is no bigger than A.) Thus the integral is eventually small! We can make this result look stronger as follows. Proposition 3. Let gn be a sequence of real-valued step functions which is nondecreasing and such that (4.24) then (4.25)
n n

lim gn (x) [0, ] x R

lim

gn [0, ].

I have written out [0, ] on the right here to make sure that it is clear that we only demand that the non-decreasing sequence gn (x) becomes 0 or positive in the limit including the possibility that it converges to + and then the same is true of the sequence of integrals. Proof. Consider the functions (4.26) fn (x) = max(0, gn (x)) x R. This is a sequence of non-negative step functions which decreases to 0 at each point and (4.27) gn fn . Thus, as claimed, the result follows from the lemma.

Next time we will use this to show that the denition of a Lebesgue integrable function above makes some sense. In particular we will show that the integral is well-dened!

MIT OpenCourseWare http://ocw.mit.edu

18.102 Introduction to Functional Analysis


Spring 2009

For information about citing these materials or our Terms of Use, visit: http://ocw.mit.edu/terms.

22

LECTURE NOTES FOR 18.102, SPRING 2009

Lecture 5. Thursday, 19 Feb I may not get quite this far, since I do not want to rush unduly!
Let me denote by L1 (R) the space of Lebesgue integrable functions on the line
as dened last time. Proposition 4. L1 (R) is a linear space. Proof. The space of all functions on R is linear, so we just need to check that L1 (R) is closed under multiplication by constants and addition. The former is easy enough multiplication by 0 gives the zero function which is integrable. If g L1 (R) then by denition there is an absolutely summable series of step function with elements fn such that (5.1) |fn | < , f (x) = fn (x) x s.t. |fn (x)| < .
n n n

Then if c = 0, cfn works for cg. The sum of two functions g and g L1 (R) is a little trickier. The obvious thing to do is to take the sum of the series of step functions. This will lead to trouble! Instead, suppose that fn and fn are series of step functions showing that g, g L1 (R). Then consider fk n = 2k 1 (5.2) hn (x) = fk n = 2k. This is absolutely summable since (5.3) |hn | = |fk | + |fk | < .
n k k

More signicantly, the series |hn (x)| converges if and only if both the series n |fk (x)| and the series |fk (x) converge. Then, because absolutely convergent
k k

series can be rearranged, it follows that (5.4) |hn (x)| < = hn (x) = fk (x) + fk (x) = g(x) + g (x).
n n 1 k k

Thus, g + g L (R).

So, the message here is to be a bit careful about the selection of the approxi mating absolutely summable series. Here is another example. Denition 4. A set E R is of measure zero if there exists an absolutely summable series of step functions fn such that (5.5) |fn (x)| = x E.
n

Proposition 5. If g L (R) and g : R C is such that g = g on R \ E where E is of measure zero, then g L1 (R).
1

Proof. What do we have to play with here an absolutely summable series fn of step functions which approximates g and another one, fn which is such that

LECTURE NOTES FOR 18.102, SPRING 2009

23

|fn (x)| = for all x E. So, to approximate g consider the interlaced series

with terms (5.6) fk (x) n = 3k 2 hn (x) = fk (x) n = 3k 1 fn (x) n = 3k.

Thus we add, but then subtract, fk . This series is absolutely summable with (5.7) |hn | = |fk | + 2 |fk |. n k k

When does the pointwise series converge absolutely? We must have (5.8) |fk (x)| + 2 |fk (x)| < .
k k

The niteness of the second term implies that x E and the niteness of the rst / means that (5.9) hn (x) = g(x) = g (x) when (5.8) holds
n N k=1 fk (x), or this plus fN (x) which tends to zero

since the nite sum is always

with N by the absolute convergence of the series in (5.8). So indeed g L1 (R).

This certainly makes one conclude that sets of measure zero are small, except that we have not yet shown that L1 (R) really makes any sense. That it does starts to become clear when we check: Proposition 6. For any element f L1 (R) the integral (5.10) f= fn
n

is well-dened independent of which approximating absolutely summable series of step functions satisfying (5.1) is used to dene it.
Proof. We can suppose that fn and fn are two absolutely summable series as in (5.1). Now that we have a little experience, it is probably natural to look at fk (x) n = 2k 1 (5.11) hn (x) = fk (x) n = 2k.

This is absolutely summable and the pointwise series is absolutely convergent only when both series are absolutely convergent. The individual terms then tend to zero and so we see that (5.12) |hn (x)| < = hn (x) = 0.
n n

Moreover, from the absolutely convergence of the sequence of integrals (5.13) | hn | |hn | <
n n

24

LECTURE NOTES FOR 18.102, SPRING 2009

it follows that we can rearrange the series to see that (5.14) hn = fk fk .


n k k

Now, what we want is that these two sums are equal, so we want to see that the left side of this equality vanishes. This follows directly from the next result which is just a little more general than needed here so is separated o. Proposition 7. If an absolutely summable series of step functions satises (5.1) with f 0 then (5.15) fn = 0.
n

Proof. So, the only thing we have at our disposal is the monotonicity result from last time. The trick is to use it! The trick is to choose and N N and consider the new series of step functions with terms (5.16) g1 (x) =
N j=1

fj (x), gk (x) = |fN +k1 (x)|, k > 1.

Now, this is absolutely summable, since convergence is a property of the tail and in any case (5.17) |gk | |fn |.
k n

Moreover, since all the terms after the rst are non-negative, the partial sums of the series p (5.18) Gp (x) = gp (x) is non-decreasing.
k=1

It is again a sequence of step functions. Note that there are two possibilities, depending on x. If the original series |fn (x)| diverges, i.e. converges to +, then
n

the same is true of Gp since this is also a property of the tails. On the other hand, if |fn (x)| is nite, then for large p,
n N k=1 p1 j=1

(5.19)

Gp (x) =

fk (x) +

|fN +j (x)|

p+N 1 k=1

fk (x).

The right side converges to zero, so the limit of this series (which is nite) is non negative. So, the monotonicity proposition from last time applies to Gp and shows that (5.20) lim Gp 0
p

where divergence to + is a possibility. This however means that, for the N we originally chose, N (5.21) fk + |fN +k | 0.
j=1 k=1

LECTURE NOTES FOR 18.102, SPRING 2009

25

This then is true for every N. On the other hand, the series of integrals is nite, so given > 0 there exists M such that if N > M, N (5.22) |fk | < = fk N > M.
kM j=1

This then implies that (5.23)


k

fk 0.

This is half of what we want, but the other half follows by applying the same reasoning to fk . This the Proposition is proved. So, this is a pretty ne example of measure-integration reasoning. Corollary 1. The integral is a well-dened map (5.24) : L1 (R) C dened by setting (5.25) f=
n

fn

for any approximating sequence as in (5.1). In particular this integral is not trivial. Namely, if f is actually a step func tion then the sequence f1 = f, fj = 0 for all j > 1 is absoutely summable and approximates f in the sense of (5.1) so (5.26) f is consistent with the integral on step functions. So, we must be onto something here! It seems I was pretty carrie away today no doubt because there were not enough questions to slow me own. Hence I even went as far as to prove: Proposition 8. A countable union of sets of measure zero has measure zero. Proof. By denition, a set E is of measure zero if there exists an absolutely sum mable series of step functions, fn , so |fn | < such that
n

(5.27)

|fn (x)| = + on E.

So, the data here gives us a countable collection Ej , j = 1, . . . , of sets and for each (j) of them we have an absolutely summable series of step functions fn such that (j) (j) (5.28) |fn | < , Ej {x R; |fn (x)| = +}.
n n

The idea is to look for one series of step functions which is absolutely summable (j) and which diverges absolutely on each Ej . The trick is to rst improve the fn . Namely, the divergence in (5.28) is a property of the tail it persists if we toss

26

LECTURE NOTES FOR 18.102, SPRING 2009

out any nite number of terms. The absolute convergence means that for each j we can choose an Nj such that (j) (5.29) |fn | < 2j j.
nNj

Now, simply throw away the all the terms in fn before n = Nj . If we relabel this (j) new sequence as fn again, then we still have (5.28) but in addition we have not only absolute summability but also (j) j (j) (5.30) |fn | 2 j = |fn | < .
n j n

(j)

Thus, the double sum (of integrals of absolutely values) is absolutely convergent. (j) Now, let hk be the fn ordered in some reasonable way say by working along each row j + n = p in turn in fact any enumeration of the double sequence will work. This is an absolutely summable series, because of (5.30). Moreover the pointwise series (j) (5.31) |hk (x)| = + if |fn
(x)| = + for any j
k n

since the second sum is contained in the rst. Thus point of each Ej , so the union has measure zero.

|hk (x)| diverges at each

MIT OpenCourseWare http://ocw.mit.edu

18.102 Introduction to Functional Analysis


Spring 2009

For information about citing these materials or our Terms of Use, visit: http://ocw.mit.edu/terms.

LECTURE NOTES FOR 18.102, SPRING 2009

27

Lecture 6. Tuesday, Feb 24 By now the structure of the proofs should be getting somewhat routine but I will go on to the point that I hope it all becomes clear! So, recall the denitions of a Lebesgue integrable function on the line (forming the linear space L1 (R)) and of a set of measure zero E R. The rst thing we want to show is that the putative norm on L1 does make sense. Proposition 9. If f L1 (R) then |f | L1 (R) and if fn is an absolutely summable series of step functions converging to f almost everywhere then N (6.1) |f | = lim | fk |.
N k=1

So in some sense the denition of the Lebesgue integral involves no cancellations. There are extensions of the integral, we may even see one, which do exploit cancel lations. Proof. By denition if f L1 (R) then it is the limit, on the set of absolute conver gence, of a summable series of step functions, {fn }. We need to make such a series for |f |. The idea in this case is the obvious one. We know that (6.2) So, set (6.3) g1 (x) = |f1 (x)|, gk (x) = |
k j=1 n j=1

fj (x) f (x) if

|fj (x)| < .

fj (x)| |

k1 j=1

fj (x)| x R.

Then, for sure, (6.4)


N k=1

gk (x) = |

N j=1

fj (x)| |f (x)| if

|fj (x)| < .

So, what we need to check, for a start, is that {gj } is an absolutely summable series of step functions. The triangle inequality in the form ||v| |w|| |v w| shows that, for k > 1, (6.5) Thus (6.6)
k

|gk (x)| = ||

k j=1

fj (x)| |

k1 j=1

fj (x)|| |fk (x)|.

|gk |

|fk | <

so the gk s do indeed form an absolutely summable series. From its construction we know that (6.7)
N k=1

gk (x) = |

N j=1

fj (x)| |f (x)| if

|fn (x)| < .

28

LECTURE NOTES FOR 18.102, SPRING 2009

So, this is what we want except that the set on which

|gk (x)| < may be larger

than the set for which we have convergence here. Now, in the notes there is a result to handle this, but we can simply make the series converge less rapidly by adding a pointless subseries. Namely replace gk by gk (x) if n = 3k 2 (6.8) hn (x) = fk (x) if n = 3k 1 fk (x) if n = 3k. This series converges absolutely if and only if both the |gk (x)| and |fk (x)| series converge the convergence of the latter implies the convergence of the former so (6.9) |hn (x)| < |fk (x)|.
n k

On the other hand when this holds, (6.10) hn (x) = |f (x)|


n

since each partial sum is either a sum for gk , or this with fn (x) added. Since fn (x) 0, (6.10) holds whenever the series converges absolutely, so indeed |f | L1 (R). Then (6.1) follows. Now, the next thing we want to know is when norm vanishes. That is, when does |f | = 0? One way is fairly easy, I think I actually skated over this a bit in the lecture, so let me write it out carefully here. The result we are after is: Proposition 10. For an integrable function f L1 (R), the vanishing of |f | implies that f is a null function in the sense that f (x) = 0 x R \ E where E is of measure zero. Conversely, if (6.11) holds then f L1 (R) and |f | = 0. (6.11) Proof. The main part of this is the rst part, that the vanishing of |f | implies that f is null. This I will prove using the next Proposition. The converse is the easier direction. Namely, if f is null in the sense of (6.11) then, by the denition of a set of measure zero, there exists an absolutely summable series of step functions, fn , such that (6.12) E {x R; |fn (x)| = }.
n

Note that it is possible that the absolute series here diverges on a larger set than E. Still, if we consider the alternating series fk (x) if n = 2k 1 (6.13) gn (x) = fk (x) if n = 2k then (6.14)
n

gn (x) = 0 whenever

LECTURE NOTES FOR 18.102, SPRING 2009

29

since the latter condition is equivalent to (6.15)

|fn (x)| < . So in fact

|gn (x)| < = f (x) = gn (x) = 0


n 1 n

because of (6.12). Thus the null function f L (R), and so is |f | and from (6.15) (6.16) |f | = gk = lim fk = 0
k k

where the last statement follows from the absolute summability.

For the converse argument we will use the following result, which is also closely related to the completeness of L1 (R). Proposition 11. If fn L1 (R) is an absolutely summable series, in the sense that |fn | < then
n

(6.17)

E = {x R;

|fn (x)| = } has measure zero

and if f : R C and (6.18) then f L1 (R) and (6.19) f=


n

f (x) =

fn (x) x R \ E

fn .

So this basically says we can replace step function by integrable function in the denition and get the same result. Of course this makes no sense without the original denition. Proof. The proof is very like the proof of completeness of the completion of a normed space that was in Problems 2, here it is a little more concrete. Thus, by assumption each fn L1 (R), so there exists an absolutely summable series of step functions fn,j , so |fn,j | < and
j

(6.20)

|fn,j (x)| < = fn (x) = fn,j (x).


j j

We can expect f (x) to be given by the sum of the fn,j (x) over both n and j, but in general, this double series is not absolutely summable. However we can make it so. Simply choose Nn for each n so that (6.21) |fn,j | < 2n .
j>Nn

This is possible by the assumed absolute summability so the tail of the series is small. Having done this, we replace the series fn,j by (6.22) fn,1 = fn,j (x), fn,j (x) = fn,Nn +k1 (x) j 2.
jNn

30

LECTURE NOTES FOR 18.102, SPRING 2009

This still converges to fn on the same set as in (6.20). So in fact we can simply replace fn,j by fn,j and we have in addition the estimate (6.23) |fn,j | | n | + 2n+1 n. f
j

This follows from the triangle inequality since, using (6.21), N (6.24) |fn,1 + fn,j | |fn,1 | |fn,j | |fn,1 | 2n
k=1 j2

and the left side converges to |fn | by (6.1) as N . Using (6.21) again gives (6.23). So, now dropping the primes from the notation and using the new series as fn,j we can set (6.25) gk (x) = fn,j .
n+j=k

This gives a new series of step functions which is absolutely summable since N (6.26) |gk | |fn,j | ( |fn | + 2n+1 ) < .
k=1 n,j n

Now, using rearrangement of absolutely convergent series we see that (6.27) |fn,j (x)| < = f (x) = |gk (x)| = fn,j (x).
n,j k n j

From a result last week, we know that the set on the left here is of the form R \ E where E is of measure zero; E is the union of the sets En on which |fn,j (x)| = .
j

So, take another absolutel summable series of step functions hk which diverges on E (at least) and insert hk and hk between successive gk s as before. This new series still coverges to f by (6.27) and shows that f L1 (R) as well as (6.17). The nal result (6.19) also follows by rearranging the double series for the integral (which is also absolutely convergent). Now for the moment we only need the weakest part, (6.17), of this. That is for any absolutely summable series of integrable functions the absolute pointwise series converges o a set of measure zero can only diverge on a set of measure zero. It is rather shocking but this allows us to prove the rest of (10)! Namely, suppose f L1 (R) and |f | = 0. Then Proposition 11 applies to the series with each term being |f |. This is absolutely summable since all the integrals are zero. So it must converge pointwise except on a set of measure zero. Clearly it diverges whenever f (x) = 0, so (6.28) |f | = o = {x; f (x) = 0} has measure zero which is what we wanted to show. Finally this allows us to dene the standard Lebesgue space (6.29) and to check that L1 (R) = L1 (R)/N , N = {null functions} |f | is indeed a norm on this space.

LECTURE NOTES FOR 18.102, SPRING 2009

31

Problem set 3, Due 11AM Tuesday 3 Mar. This problem set is also intended to be a guide to what will be on the in-class test on March 5. In particular I will ask you to prove some of the properties of the Lebesgue integral, as below, plus one more abstract proof. Recall that equality a.e (almost everywhere) means equality on the complement of a set of measure zero. Problem 3.1 If f and g L1 (R) are Lebesgue integrable functions on the line show that (1) If f (x) 0 a.e. then f 0. (2) If f (x) g(x) a.e. then f g. (3) If f is complex valued then its real part, Re f, is Lebesgue integrable and | Re f | |f |. (4) For a general complex-valued Lebesgue integrable function (6.30) | f | |f |. Hint: You can look up a proof of this easily enough, but the usual trick is to choose [0, 2) so that ei f = (ei f ) 0. Then apply the preceeding estimate to g = ei f. (5) Show that the integral is a continuous linear functional (6.31) : L1 (R) C. Problem 3.2 If I R is an interval, including possibly (, a) or (a, ), we dene Lebesgue integrability of a function f : I C to mean the Lebesgue integrability of f (x) x I (6.32) f : R C, f (x) = 0 x R \ I. The integral of f on I is then dened to be (6.33) f = f.
I

(1) Show that the space of such integrable functions on I is linear, denote it L1 (I). (2) Show that is f is integrable on I then so is |f |. (3) Show that if f is integrable on I and I |f | = 0 then f = 0 a.e. in the sense that f (x) = 0 for all x I \ E where E I is of measure zero as a subset of R. (4) Show that the set of null functions as in the preceeding question is a linear space, denote it N (I). (5) Show that I |f | denes a norm on L1 (I) = L1 (I)/N (I). (6) Show that if f L1 (R) then f (x) x I (6.34) g : I C, g(x) = 0 xR\I is integrable on I.

32

LECTURE NOTES FOR 18.102, SPRING 2009

(7) Show that the preceeding construction gives a surjective and continuous linear map restriction to I (6.35) L1 (R) L1 (I). (Notice that these are the quotient spaces of integrable functions modulo equality a.e.)
Problem 3.3 Really continuing the previous one.
(1) Show that if I = [a, b) and f L1 (I) then the restriction of f to Ix = [x, b) is an element of L1 (Ix ) for all a x < b. (2) Show that the function (6.36) F (x) = f : [a, b) C
Ix

is continuous. (3) Prove that the function x1 cos(1/x) is not Lebesgue integrable on the interval (0, 1]. Hint: Think about it a bit and use what you have shown above. Problem 3.4 [Harder but still doable] Suppose f L1 (R). (1) Show that for each t R the translates (6.37)
1

ft (x) = f (x t) : R C

are elements of L (R). (2) Show that (6.38)


t0

lim

|ft f | = 0.

This is called Continuity in the mean for integrable functions. Hint: I will add one! (3) Conclude that for each f L1 (R) the map (it is a curve) (6.39) R t [ft ] L1 (R) is continuous. Problem 3.5 In the last problem set you showed that a continuous function on a compact interval, extended to be zero outside, is Lebesgue integrable. Using this, and the fact that step functions are dense in L1 (R) show that the linear space of continuous functions on R each of which vanishes outside a compact set (which depends on the function) form a dense subset of L1 (R). Problem 3.6 (1) If g : R C is bounded and continuous and f L1 (R) show that gf L1 (R) and that (6.40) |gf | sup |g | |f |.
R

(2) Suppose now that G C([0, 1] [0, 1]) is a continuous function (I use C(K) to denote the continuous functions on a compact metric space). Recall from the preceeding discussion that we have dened L1 ([0, 1]). Now, using the rst part show that if f L1 ([0, 1]) then (6.41) F (x) = G(x, () C )f
[0,1]

LECTURE NOTES FOR 18.102, SPRING 2009

33

(where is the variable in which the integral is taken) is well-dened for each x [0, 1]. (3) Show that for each f L1 ([0, 1]), F is a continuous function on [0, 1]. (4) Show that (6.42) L1 ([0, 1]) f F C([0, 1]) is a bounded (i.e. continuous) linear map into the Banach space of contin uous functions, with supremum norm, on [0, 1].

34

LECTURE NOTES FOR 18.102, SPRING 2009

Solutions to Problem set 2 I was originally going to make this problem set longer, since there is a missing Tuesday. However, I would prefer you to concentrate on getting all four of these questions really right! Problem 2.1 Finish the proof of the completeness of the space B constructed in lecture on February 10. The description of that construction can be found in the notes to Lecture 3 as well as an indication of one way to proceed. Solution. The proof could be shorter than this, I have tried to be fairly complete. To recap. We start of with a normed space V. From this normed space we construct the new linear space V with points the absolutely summable series in V. Then we consider the subspace S V of those absolutely summable series which converge to 0 in V. We are interested in the quotient space (6.43) B = V /S. What we know already is that this is a normed space where the norm of b = {vn }+S where {vn } is an absolutely summable series in V is (6.44) bB = lim
N N n=1

vn V .

This is independent of which series is used to represent b i.e. is the same if an element of S is added to the series. Now, what is an absolutely summable series in B? It is a sequence {bn }, thought of a series, with the property that (6.45) bn B < .
n

We have to show that it converges in B. The rst task is to guess what the limit should be. The idea is that it should be a series which adds up to the sum of the (n) bn s. Each bn is represented by an absolutely summable series vk in V. So, we can just look for the usual diagonal sum of the double series and set (n) (6.46) wj = vk .
n+k=j

The problem is that this will not in generall be absolutely summable as a series in V. What we want is the estimate (n) (6.47) wj = vk < .
j j j=n+k

The only way we can really estimate this is to use the triangle inequality and conclude that (n) (6.48) wj vk V .
j=1 k,n

Each of the sums over k on the right is nite, but we do not know that the sum over k is then nite. This is where the rst suggestion comes in: (n) We can choose the absolutely summable series vk representing bn such that (n) (6.49) vk bn B + 2n .
k

LECTURE NOTES FOR 18.102, SPRING 2009

35

Suppose an initial choice of absolutely summable series representing bn is uk , so N bn = limN uk and uk V < . Choosing M large it follows that
k=1 k

(6.50)

k>M

uk V 2n1 .
(n)

With this choice of M set v1

(n)

M k=1

uk and vk

= uM +k1 for all k 2. This does

still represent bn since the dierence of the sums, (6.51)


N k=1

vk

(n)

N k=1

uk =

N +M 1 k=N

uk

for all N. The sum on the right tends to 0 in V (since it is a xed number of terms). Moreover, because of (6.50), (6.52) M N N (n) vk V = uj V + uk uj + 2 uk uj + 2n
k j=1 k>M j=1 k>M j=1

for all N. Passing to the limit as N gives (6.49). Once we have chosen these nice representatives of each of the bn s if we dene the wj s by (6.46) then (6.47) means that (6.53) wj V bn B + 2n <
j n n

because the series bn is absolutely summable. Thus {wj } denes an element of V and hence b B. Finally then we want to show that bn = b in B. This just means that we need
n

to show (6.54) lim b


N n=1 N n=1

bn B = 0.

The norm here is itself a limit b with nth term (6.55) and the norm is then (6.56) lim
p

bn is represented by the summable series

wk

N n=1

vk

(n)

(wk

N n=1

vk )V .

(n)

k=1

Then we need to understand what happens as N ! Now, wk is the diagonal (n) (n) sum of the vj s so sum over k gives the dierence of the sum of the vj over the rst p anti-diagonals minus the sum over a square with height N (in n) and width

36

LECTURE NOTES FOR 18.102, SPRING 2009

p. So, using the triangle inequality the norm of the dierence can be estimated by the sum of the norms of all the missing terms and then some so (6.57)
p k=1

(wk

N n=1

vk )V

(n)

l+mL

vl

(m)

where L = min(p, N ). This sum is nite and letting p is replaced by the sum over l + m N. Then letting N it tends to zero by the absolute (double) summability. Thus (6.58) lim b
N n=1

bn B = 0

which is the statelent we wanted, that

bn = b.

Problem 2.2 Lets consider an example of an absolutely summable sequence of step functions. For the interval [0, 1) (remember there is a strong preference for left-closed but right-open intervals for the moment) consider a variant of the construction of the standard Cantor subset based on 3 proceeding in steps. Thus, remove the central interval [1/3, 2/3). This leave C1 = [0, 1/3) [2/3, 1). Then remove the central interval from each of the remaining two intervals to get C2 = [0, 1/9) [2/9, 1/3) [2/3, 7/9) [8/9, 1). Carry on in this way to dene successive sets Ck Ck1 , each consisting of a nite union of semi-open intervals. Now, consider the series of step functions fk where fk (x) = 1 on Ck and 0 otherwise. (1) Check that this is an absolutely summable series. (2) For which x [0, 1) does |fk (x)| converge?
k

(3) Describe a function on [0, 1) which is shown to be Lebesgue integrable (as dened in Lecture 4) by the existence of this series and compute its Lebesgue integral. (4) Is this function Riemann integrable (this is easy, not hard, if you check the denition of Riemann integrability)? (5) Finally consider the function g which is equal to one on the union of all the subintervals of [0, 1) which are removed in the construction and zero elsewhere. Show that g is Lebesgue integrable and compute its integral. Solution. (1) The total length of the intervals is being reduced by a factor of k 1/3 each time. Thus l(Ck ) = 2k . Thus the integral of f, which is non 3 negative, is actually 2k 2k |fk | = =2 (6.59) fk = k = 3 3k
k k=1

Thus the series is absolutely summable. (2) Since the Ck are decreasing, Ck Ck+1 , only if (6.60) xE= Ck
k

does the series

|fk (x)| diverge (to +) otherwise it converges.

LECTURE NOTES FOR 18.102, SPRING 2009

37

(3) The function dened as the sum of the series where it converges and zero otherwise fk (x) x R \ E (6.61) f (x) = k 0 xE is integrable by denition. Its integral is by denition (6.62) f= fk = 2
k

from the discussion above. (4) The function f is not Riemann integrable since it is not bounded and this is part of the denition. In particular for x Ck \ Ck+1 , which is not an empty set, f (x) = k. (5) The set F, which is the union of the intervals removed is [0, 1) \ E. Taking step functions equal to 1 on each of the intervals removed gives an absolutely summable series, since they are non-negative and the kth one has integral 1/3(2/3)k1 for k = 1, . . . . This series converges to g on F so g is Lebesgue integrable and hence (6.63) g = 1. Problem 2.3 The covering lemma for R2 . By a rectangle we will mean a set of the form [a1 , b1 ) [a2 , b2 ) in R2 . The area of a rectangle is (b1 a1 ) (b2 a2 ). (1) We may subdivide a rectangle by subdividing either of the intervals re placing [a1 , b1 ) by [a1 , c1 ) [c1 , b1 ). Show that the sum of the areas of rectangles made by any repeated subdivision is always the same as that of the original. (2) Suppose that a nite collection of disjoint rectangles has union a rectangle (always in this same half-open sense). Show, and I really mean prove, that the sum of the areas is the area of the whole rectange. Hint:- proceed by subdivision. (3) Now show that for any countable collection of disjoint rectangles contained in a given rectange the sum of the areas is less than or equal to that of the containing rectangle. (4) Show that if a nite collection of rectangles has union containing a given rectange then the sum of the areas of the rectangles is at least as large of that of the rectangle contained in the union. (5) Prove the extension of the preceeding result to a countable collection of rectangles with union containing a given rectangle. Solution. (1) For subdivision of one rectangle this is clear enough. Namely we either divide the rst side into two or the second side in two at an intermediate point c. After subdivision the area of the two rectanges is either (c a1 )(b2 a2 ) + (b1 c)(b2 a2 ) = (b1 c1 )(b2 a2 ) or (6.64) (b1 a1 )(c a2 ) + (b1 a1 )(b2 c) = (b1 c1 )(b2 a2 ).

38

LECTURE NOTES FOR 18.102, SPRING 2009

this shows by induction that the sum of the areas of any the rectangles made by repeated subdivision is always the same as the original. (2) If a nite collection of disjoint rectangles has union a rectangle, say [a1 , b2 ) [a2, b2 ) then the same is true after any subdivision of any of the rectangles, by the previous result. Moreover after such subdivision the sum of the areas is always the same. Look at all the points C1 [a1 , b1 ) which occur as an endpoint of the rst interval of one of the rectangles. Similarly let C2 be the corresponding set of end-points of the second intervals of the rectangles. Now divide each of the rectangles repeatedly using the nite number of points in C1 and the nite number of points in C2 . The total area remains the same and now the rectangles covering [a1 , b1 ) [A2 , b2 ) are precisely the Ai Bj where the Ai are a set of disjoint intervals covering [a1 , b1 ) and the Bj are a similar set covering [a2 , b2 ). Applying the one-dimensional result from class we see that the sum of the areas of the rectangles with rst interval Ai is the product (6.65) length of Ai (b2 a2 ).

Then we can sum over i and use the same result again to prove what we want. (3) For any nite collection of disjoint rectangles contained in [a1 , b1 ) [a2 , b2 ) we can use the same division process to show that we can add more disjoint rectangles to cover the whole big rectangle. Thus, from the preceeding result the sum of the areas must be less than or equal to (b1 a1 )(b2 a2 ). For a countable collection of disjoint rectangles the sum of the areas is therefore bounded above by this constant. (4) Let the rectangles be Di , i = 1, . . . , N the union of which contains the rectangle D. Subdivide D1 using all the endpoints of the intervals of D. Each of the resulting rectangles is either contained in D or is disjoint from it. Replace D1 by the (one in fact) subrectangle contained in D. Proceeding by induction we can suppose that the rst N k of the rectangles are disjoint and all contained in D and together all the rectangles cover D. Now look at the next one, DN k+1 . Subdivide it using all the endpoints of the intervals for the earlier rectangles D1 , . . . , Dk and D. After subdivision of DN k+1 each resulting rectangle is either contained in one of the Dj , j N k or is not contained in D. All these can be discarded and the result is to decrease k by 1 (maybe increasing N but that is okay). So, by induction we can decompose and throw away rectangles until what is left are disjoint and individually contained in D but still cover. The sum of the areas of the remaining rectangles is precisely the area of D by the previous result, so the sum of the areas must originally have been at least this large. (5) Now, for a countable collection of rectangles covering D = [a1 , b1 ) [a2 , b2 ) we proceed as in the one-dimensional case. First, we can assume that there is a xed upper bound C on the lengths of the sides. Make the kth rectangle a little larger by extending both the upper limits by 2k where > 0. The area increases, but by no more than 2C2k . After extension the interiors of the countable collection cover the compact set [a1 , b1 ] [a2 , b1 ]. By compactness, a nite number of these open rectangles cover, and hence there semi-closed version, with the same endpoints, covers [a1 , b1 )

LECTURE NOTES FOR 18.102, SPRING 2009

39

[a2 , b1 ). Applying the preceeding nite result we see that (6.66) Sum of areas + 2C Area D 2C. Since this is true for all > 0 the result follows. I encourage you to go through the discussion of integrals of step functions now based on rectangles instead of intervals and see that everything we have done can be extended to the case of two dimensions. In fact if you want you can go ahead and see that everything works in Rn ! Problem 2.4 (1) Show that any continuous function on [0, 1] is the uniform limit on [0, 1) of a sequence of step functions. Hint:- Reduce to the real case, divide the interval into 2n equal pieces and dene the step functions to take inmim of the continuous function on the corresponding interval. Then use uniform convergence. (2) By using the telescoping trick show that any continuous function on [0, 1) can be written as the sum (6.67) fj (x) x [0, 1)
i

where the fj are step functions and

|fj (x)| < for all x [0, 1).

(3) Conclude that any continuous function on [0, 1], extended to be 0 outside this interval, is a Lebesgue integrable function on R. Solution. (1) Since the real and imaginary parts of a continuous function are continuous, it suces to consider a real continous function f and then add afterwards. By the uniform continuity of a continuous function on a compact set, in this case [0, 1], given n there exists N such that |x y| 2N = |f (x)f (y)| 2n . So, if we divide into 2N equal intervals, where N depends on n and we insist that it be non-decreasing as a function of n and take the step function fn on each interval which is equal to min f = inf f on the closure of the interval then (6.68) |f (x) Fn (x)| 2n x [0, 1)

since this even works at the endpoints. Thus Fn f uniformly on [0, 1). (2) Now just dene f1 = F1 and fk = Fk Fk1 for all k > 1. It follows that these are step functions and that (6.69)
n k=1

= fn .

Moreover, each interval for Fn+1 is a subinterval for Fn . Since f can varying by no more than 2n on each of the intervals for Fn it follows that (6.70) |fn (x)| = |Fn+1 (x) Fn (x)| 2n n > 1. Thus |fn | 2n and so the series is absolutely summable. Moreover, it actually converges everywhere on [0, 1) and uniformly to f by (6.68). (3) Hence f is Lebesgue integrable.

40

LECTURE NOTES FOR 18.102, SPRING 2009

(4) For some reason I did not ask you to check that 1 (6.71) f= f (x)dx
0

where on the right is the Riemann integral. However this follows from the fact that (6.72) f = lim Fn
n

and the integral of the step function is between the Riemann upper and lower sums for the corresponding partition of [0, 1].

MIT OpenCourseWare http://ocw.mit.edu

18.102 Introduction to Functional Analysis


Spring 2009

For information about citing these materials or our Terms of Use, visit: http://ocw.mit.edu/terms.

LECTURE NOTES FOR 18.102, SPRING 2009

41

Lecture 7. Thursday, Feb 26 So, what was it with my little melt-down? I went too cheap on the monotonicity theorem and so was under-powered for Fatous Lemma. In my defense, I was trying to modify things on-the-y to conform to how we are doing things here. I should also point out that at least one person in the audience made a comment which amounted to pointing out my error. So, here is something closer to what I should have said it is not far from what I did say of course. Proposition 12. [Montonicity again] If fj L1 (R) is a monotone sequence, either fj (x) fj+1 (x) for all x R and all j or fj (x) fj+1 (x) for all x R and all j, and fj is bounded then (7.1) {x R; lim fj (x) is nite} = R \ E
j

where E has measure zero and (7.2) f = lim fj (x) a.e. is an element of L1 (R) j with lim |f fj | = 0.
j

Moral of the story drop the assumption of positivity and replace it with the bound on the integral. In the approach through measure theory this is not necessary because one has the concept of a measureable, non-negative, function for which the integral exists but is innite we do not have this. Proof. Since we can change the sign of the fi (now) it suces to assume that the fi are monotonically increasing. The sequence of integrals is therefore also montonic increasing and, being bounded, converges. Thus we can pass to a subsequence gi = fni with the property that (7.3) |gj gj1 | = gj gj1 < 2j j > 1. This means that the series h1 = g1 , hj = gj gj1 , j > 1, is absolutely summable. So we know for the result last time that it converges a.e., that the limit, f, is integrable and that (7.4) f = lim
j

j
k=1

hk = lim
j j

gj = lim
n

fj .

In fact, everywhere that the series

hj (x), which is to say the sequence gk (x),

converges so does fn (x), since the former is a subsequence of the latter which is monotonic. So we have (7.1) and the rst part of (7.2). The second part, corresponding to convergence for the equivalence classes in L1 (R) follows from monotonicity, since (7.5) |f fj | = f fj 0 as j .

42

LECTURE NOTES FOR 18.102, SPRING 2009

Now, to Fatous Lemma. This really just takes the monotonicity result and applies it to a general sequence of integrable functions with bounded integral. You should recall as I did that the max and min of two integrable functions is integrable and that (7.6) min(f, g) min( f, g). Lemma 3. [Fatou]. Let fj L1 (R) be a sequence of non-negative (so real-valued integrable) functions such that fj is bounded above in R, then (7.7) f (x) = lim inf fn (x) exists a.e., f L1 (R) and n lim inf fn lim inf fn .

Proof. You should remind yourself of the properties of liminf as necessary! Fix k and consider (7.8) Fk,n =
kpk+n

min

fp (x) L1 (R)

as discussed briey above. Moreover, this is a decreasing sequence, as n increases, because the minimum is over an increasing set an all elements are non-negative. Thus the integrals are bounded below by 0 so the monotonicity result above applies and shows that (7.9) gk (x) = inf fp (x) L1 (R), gk fn n k.
pk

Note that for a decreasing sequence of non-negative numbers the limit exists every where and is indeed the inmum. Thus in fact, (7.10) gk lim inf fn . Now, let k vary. Then, the inmum in (7.9) is over a set which decreases as k increases. Thus the gk (x) are increasing. The integral is always bounded by one of the fn and hence is bounded above independent of k since we assumed a bound on the fn s. So, now we can apply the monotonicity result again to see that (7.11) f (x) = lim gk (x) exists a.e and f L1 (R) has k f lim inf fn .

Since f (x) = lim inf fn (x), by denition of the latter, we have proved the Lemma. Now, we apply Fatous Lemma to prove what we are really after: Theorem 2. [Lebesgues dominated convergence]. Suppose fj L1 (R) is a se quence of integrable functions such that (7.12) h L1 (R) with |fj (x)| h(x) a.e. and f (x) = lim fj (x) exists a.e. n f = limn fn (including the assertion that this limit

Then f L1 (R) and exists).

LECTURE NOTES FOR 18.102, SPRING 2009

43

Proof. First, we can assume that the fj are real since the hypotheses hold for its real and imaginary parts and together give the desired result. Moroever, we can change all the fj s to make them zero on the set on which the estimate in (7.12) does not hold. Then this bound on the fj s becomes (7.13) h(x) fj (x) h(x) x R. In particular this means that gj = g fj is a non-negative sequence of integrable functions and the sequence of integrals is also bounded, since (7.12) also implies that |fj | h, so gj 2 h. Thus Fatous Lemma applies to the gj . Since we have assumed that the sequence gj (x) converges a.e. to f we know that (7.14) h h f (x) = lim inf gj (x) a.e. and f lim inf (h fj ) = h lim sup fj .

Notice the change on the right from liminf to limsup because of the sign. Now we can apply the same argument to gj (x) = h(x) + fj (x) since this is also non-negative and has integrals bounded above. This converges a.e. to h(x) + f (x) so this time we conclude that (7.15) h + f lim inf (h + fj ) = h + lim inf fj . In both inequalities (7.14) and (7.15) we can cancel and h and combining them we nd (7.16) lim sup fj f lim inf fj . In particular the limsup on the left is smaller than, or equal to, the liminf on the right, for the same real sequence. This however implies that the are equal and that the sequence fj converges (look up properties of liminf and limsup if necessary ...). Thus indeed (7.17) f = lim fn .
n

Generally in applications it is Lebesgues dominated convergence which is used to prove that some function is integrable. Finally I want to make sure that we agree that L1 (R) is a Banach space. Note once again that I have used the somewhat non-standard notation (7.18) L1 (R) = {f : R C; f is integrable.} This is a curly L. We know that f L1 (R) implies that |f | L1 (R) (if you are wondering the converse might not be true if f oscillates badly enough). Now, we know exactly when the integral of the abolute value vanishes. Namely (7.19) N = {f L1 (R); |f | = 0} = {f : R C; f (x) = 0 x R \ E, E of measure zero}. Namely, this is the linear space of null functions. We then dened (7.20) L1 (R) = L1 (R)/N .

44

LECTURE NOTES FOR 18.102, SPRING 2009

This has a non-curly L the notation is by no means standard but the denition (7.20) certainly is. Thus the elements of L1 (R) are equivalence classes of functions (7.21) [f ] = f + N , f L1 (R). That is, we identify to element of L1 (R) if (and only if) there dierence is null, which is to say they are equal o a set of measure zero. Note that the set is not xed, but can depend on the functions. Anyway, for an element of L1 (R) the integral of the absolute value is well-dened: (7.22) [f ]L1 = |f | since the right side is independent of which representative we choose. Theorem 3. The function L1 in (7.22) is a norm on L1 (R) with respect to which it is a Banach space. The integral of the absolute value, |f | is a semi-norm on L1 (R) it satises all the properties of a norm except that |f | = 0 does not imply f = 0, only f N . We are killing this problem by taking the quotient. Proof. I will not go through the proofs of the norm properties but you should. So, the only issue remaining is the completeness of L1 (R) with respect to }L1 . The completeness is a direct consequence of the Theorem in the last lecture on absolutely summable series of Lebesgue functions, so remind yourself of what this says. Also recall how we showed that if f is integrable, so is |f |. Namely, if fj is an absolutely summable series (originally of step functions, now of Lebesgue integrable functions) then we dened (7.23) g1 = |f1 |, gj = | fk | | fk |
kj kj1

and observed that (7.24) |gj | |fj | j.


j

Thus, gj is also absolutely summable and everywhere (7.25)


jN

fj (x) converges,

(x) = |

jN

fj (x)| |f (x)| as N .

This shows that |f | L1 (R), but more than that since (7.26) |f | = lim | fj (x)| |fj |.
N jN j

Roughly speaking this is why we have been using absolutely summable series from the beginning. 1 So, going back to fj and absolutely summable series in L (R), in the sense that |fj |, we can apply the discussion above to the truncated series starting at point
j

N. Namely, the fj for j N give an absolutely convergent series which sums a.e. to (7.27) f (x) fj (x) = fj (x).
j<N j>N

LECTURE NOTES FOR 18.102, SPRING 2009

45

Now, applying (7.26) we see that (7.28) |f (x) fj (x)| |fj |.


j<N j>N

However, the absolute convergence means that the tail on the right is small with N, that is, (7.29) lim |f fj | = 0.
N j<N

So, nally it is only necessary to think about L1 (R) instead of L1 (R). An abso lutely summable sequence in Fj in L1 (R) is a series of equivalence classes fj + N where fj L1 (R). The absolutely summability condition is (7.30) Fj L1 = |fj | <
j j

is what we need to start the discussion above. Namely, we have shown that the sum a.e. f of the series fj is an element of L1 (R) and (7.29) holds. But this just means that the equivalenct class F = f + N satises (7.31) lim F Fj L1 = lim |f fj | = 0.
N j<N N j<N

Thus,

N j=1

Fj = F in L1 (R) which is therefore complete.

Note that despite the fact that it is technically incorrect, everyone says L1 (R) is the space of Lebesgue integrable functions even though it is really the space of equivalence classes of these functions modulo equality almost everywhere. Not much harm can come from doing this.

MIT OpenCourseWare http://ocw.mit.edu

18.102 Introduction to Functional Analysis


Spring 2009

For information about citing these materials or our Terms of Use, visit: http://ocw.mit.edu/terms.

46

LECTURE NOTES FOR 18.102, SPRING 2009

Lecture 8. Tuesday, Mar 3: Cauchys inequality and Lebesgue measure I rst discussed the denition of preHilbert and Hilbert spaces and proved Cauchys inequality and the parallelogram law. This can be found in all the lecture notes and many other places so I will not repeat it here. Another nice source is the book of G.F. Simmons, Introduction to topology and modern analysis. I like it but I think it is out of print. In case anyone is interested in how to dene Lebesgue measure from where we are now and I may have time to do this later we can just use the integral as I outlined on Tuesday. First, we dene locally integrable functions. Thus f : R C is locally integrable if f (x) x [N, N ] (8.1) F[N,N ] = L1 (R) N. 0 x if |x| > N For example any continuous function on R is locally integrable. Lemma 4. The locally integrable functions form a linear space. Proof. Follows from the linearity of L1 (R).

Denition 5. A set A R is measurable if its characteristic function A is locally integrable. A measurable set A has nite measure if A L1 (R) and then (8.2) (A) = N is the Lebesgue measure of A. If A is measurable but not of nite measure then (A) = by denition. We know immediately that any interval (a, b) is measurable (whether open, semiopen or closed) and has nite measure if and only if it is bounded then the measure is b a. Some things to check: Proposition 13. The complement of a measurable set is measureable and any countable union of measurable sets is measurable. Proof. The rst part follows from the fact that the constant function 1 is locally integrable and hence R\A = 1 A is locally integrable if and only if A is locally integrable. Notice the relationship between characteristic functions and the sets they dene: (8.3) AB = max(A , B ), AB = min(A , B ).  If we have a sequence of sets An then Bn = kn Ak is clearly an increasing sequence of sets and (8.4) Bn B , B = An
n

is an increasing sequence which converges pointwise (at each point it jumps to 1 somewhere and then stays or else stays at 0.) Now, if we multiply by [N,N ] then (8.5) fn = [N,N ] Bn B[N,N ] is an increasing sequence of integrable functions assuming that is that the Ak s are measurable with integral bounded above, by 2N. Thus by our monotonicity  theorem the limit is integrable so B is locally integrable and hence n An is measurable.

LECTURE NOTES FOR 18.102, SPRING 2009

47

Proposition 14. The (Lebesgue) measurable subsets of R form a collection, M, of the power set of R, including and R which is closed under complements, countable unions and countable intersections. Proof. The countable intersection property follows from the others or directly by a similar argument to that above but with decreasing sequences. We have declared a set A which is measurable but not of nite measure to have innite measure for instance R is of innite measure in this sense. Since the measure of a set is always non-negative (or undened if it isnt measurable) this does not cause any problems and in fact Lebesgue measure is countable additive provided we allow as a value of the measure:

(8.6) An M, n N = An M and mu( An ) (An )
n n n

with equality if Aj Ak = for j = k. It is a good exercise to prove this!

48

LECTURE NOTES FOR 18.102, SPRING 2009

Problem set 4, Due 11AM Tuesday 10 Mar. Just to compensate for last week, I will make this problem set too short and easy! Problem 4.1 Let H be a normed space in which the norm satises the parallelogram law: (8.7) u + v2 + u v2 = 2(u2 + v2 ) u, v H. Show that the norm comes from a positive denite sesquilinear (i.e. Hermitian) inner product. Big Hint:- Try 1 u + v2 u v2 + iu + iv2 iu iv2 ! (8.8) (u, v) = 4 Problem 4.2 Let H be a nite dimensional (pre)Hilbert space. So, by denition H has a basis {vi }n , meaning that any element of H can be written i=1 (8.9) v= ci vi
i

and there is no dependence relation between the vi s the presentation of v = 0 in the form (8.9) is unique. Show that H has an orthonormal basis, {ei }n satisfying i=1 (ei , ej ) = ij (= 1 if i = j and 0 otherwise). Check that for the orthonormal basis the coecients in (8.9) are ci = (v, ei ) and that the map (8.10) T : H v ((v, ei )) Cn is a linear isomorphism with the properties (8.11) (u, v) = (T u)i (T v)i , uH = T uCn u, v H.
i

Why is a nite dimensional preHilbert space a Hilbert space?

LECTURE NOTES FOR 18.102, SPRING 2009

49

Solutions to Problem set 3 This problem set is also intended to be a guide to what will be on the in-class test on March 5. In particular I will ask you to prove some of the properties of the Lebesgue integral, as below, plus one more abstract proof. Recall that equality a.e (almost everywhere) means equality on the complement of a set of measure zero. Problem 3.1 If f and g L1 (R) are Lebesgue integrable functions on the line show that (1) If f (x) 0 a.e. then f 0. (2) If f (x) g(x) a.e. then f g. (3) If f is complex valued then its real part, Re f, is Lebesgue integrable and | Re f | |f |. (4) For a general complex-valued Lebesgue integrable function (8.12) | f | |f |. Hint: You can look up a proof of this easily enough, but the usual trick is to choose [0, 2) so that ei f = (ei f ) 0. Then apply the preceeding estimate to g = ei f. (5) Show that the integral is a continuous linear functional (8.13) : L1 (R) C. Solution: (1) If f is real and fn is a real-valued absolutely summable series of step func tions converging to f where it is absolutely convergent (if we only have a complex-valued sequence use part (3)). Then we know that (8.14) g1 = |f1 |, gj = |fj | |fj1 |, f 1

is an absolutely convergent sequence converging to |f | almost everywhere. It follows that f+ = 1 (|f | + f ) = f, if f 0, is the limit almost everywhere 2 of the series obtained by interlacing 1 gj and 1 fj : 2 2 1 gk n = 2k 1 (8.15) hn = 2 fk n = 2k. Thus f+ is Lebesgue integrable. Moreover we know that k k | (8.16) f+ = lim hk = lim fj | + fj
k n2k k j=1 j=1

where each term is a non-negative step function, so f+ 0. (2) Apply the preceeding result to g f which is integrable and satises (8.17) g f = (g f ) 0.

50

LECTURE NOTES FOR 18.102, SPRING 2009

(3) Arguing from rst principles again, if fn is now complex valued and an absolutely summable series of step functions converging a.e. to f then dene Re fk n = 3k 2 (8.18) hn = Im fk n = 3k 1 Im fk n = 3k. This series of step functions is absolutely summable and (8.19) |hn (x)| < |fn (x)| < = hn (x) = Re f.
n n n

Thus Re f is integrable. Since Re f |f | (8.20) Re f |f | = | Re f | |f |. (4) For a complex-valued f proceed as suggested. Choose z C with |z | = 1 such that z f [0, ) which is possible by the properties of complex numbers. Then by the linearity of the integral (8.21) z f= (zf ) = Re(zf ) |z Re f | |f | = | f| = z f |f |. (where the second equality follows from the fact that the integral is equal to its real part). (5) We know that the integral denes a linear map (8.22) I : L1 (R) [f ] f C since f = g if f = g a.e. are two representatives of the same class in L1 (R). To say this is continuous is equivalent to it being bounded, which follows from the preceeding estimate (8.23) |I([f ])| = | f | |f | = [f ]L1 (Note that writing [f ] instead of f L1 (R) is correct but would normally be considered pedantic at least after you are used to it!) (6) I should have asked and might do on the test: What is the norm of I as an element of the dual space of L1 (R). It is 1 better make sure that you can prove this. Problem 3.2 If I R is an interval, including possibly (, a) or (a, ), we dene Lebesgue integrability of a function f : I C to mean the Lebesgue integrability of : R C, f (x) = f (x) x I (8.24) f 0 x R \ I. The integral of f on I is then dened to be (8.25) f = f.
I

(1) Show that the space of such integrable functions on I is linear, denote it L1 (I). (2) Show that is f is integrable on I then so is |f |.

LECTURE NOTES FOR 18.102, SPRING 2009

51

(3) Show that if f is integrable on I and I |f | = 0 then f = 0 a.e. in the sense that f (x) = 0 for all x I \ E where E I is of measure zero as a subset of R. (4) Show that the set of null functions as in the preceeding question is a linear space, denote it N (I). (5) Show that I |f | denes a norm on L1 (I) = L1 (I)/N (I). (6) Show that if f L1 (R) then f (x) x I (8.26) g : I C, g(x) = 0 xR\I is in L1 (R) an hence that f is integrable on I. (7) Show that the preceeding construction gives a surjective and continuous linear map restriction to I (8.27) L1 (R) L1 (I). (Notice that these are the quotient spaces of integrable functions modulo equality a.e.) Solution: (1) If f and g are both integrable on I then setting h = f + g, h = f + g , directly from the denitions, so f + g is integrable on I if f and g are by the linearity of L1 (R). Similarly if h = cf then h = cf is integrable for any constant c if f is integrable. Thus L1 (I) is linear. (2) Again from the denition, |f | = h if h = |f |. Thus f integrable on I L1 (R), which, as we know, implies that |f | L1 (R). So in turn implies f L1 (R) where h = |f |, so |f | L1 (I). h (3) If f L1 (I) and I |f | = 0 then R |f | = 0 which implies that f = 0 on R \ E where E R is of measure zero. Now, EI = E I E is also of measure zero (as a subset of a set of measure zero) and f vanishes outside EI . (4) If f, g : I C are both of measure zero in this sense then f + g vanishes on I \ (Ef Eg ) where Ef I and Ef I are of measure zero. The union of two sets of measure zero (in R) is of measure zero so this shows f + g is null. The same is true of cf + dg for constant c and d, so N (I) is a linear space. (5) If f L1 (I) and g N (I) then |f + g| |f | N (I), since it vanishes where g vanishes. Thus (8.28) |f + g| = |f | f L1 (I), g N (I).
I I

Thus (8.29) [f ]I =
I

|f |

is a well-dened function on L1 (I) = L1 (R)/N (I) since it is constant on equivalence classes. Now, the norm properties follow from the same prop erties on the whole of R.

52

LECTURE NOTES FOR 18.102, SPRING 2009

(6) Suppose f L1 (R) and g is dened in (8.26) above by restriction to I. We need to show that g L1 (R). If fn is an absolutely summable series of step functions converging to f wherever, on R, it converges absolutely consider (8.30) gn (x) = fn (x) 0 on I on R \ I

where I is I made half-open if it isnt already by adding the lower end point (if there is one) and removing the upper end-point (if thereis one). Then gn is a step function (which is why we need I ). Moreover, |gn | |fn | so the series gn is absolutely summable and converges to gn outside I and at all points inside I where the series is absolutely convergent (since it is then the same as fn ). Thus g is integrable, and since f diers from g by its values at two points, at most, it too is integrable so f is integrable on I by denition. (7) First we check we do have a map. Namely if f N (R) then g in (8.26) is certainly an element of N (I). We have already seen that restriction to I maps L1 (R) into L1 (I) and since this is clearly a linear map it denes (8.27) the image only depends on the equivalence class of f. It is clearly linear and to see that it is surjective observe that if g L1 (I) then extending it as zero outside I gives an element of L1 (R) and the class of this function maps to [g] under (8.27). Problem 3.3 Really continuing the previous one. (1) Show that if I = [a, b) and f L1 (I) then the restriction of f to Ix = [x, b) is an element of L1 (Ix ) for all a x < b. (2) Show that the function (8.31) F (x) =
Ix

f : [a, b) C

is continuous. (3) Prove that the function x1 cos(1/x) is not Lebesgue integrable on the interval (0, 1]. Hint: Think about it a bit and use what you have shown above. Solution: (1) This follows from the previous question. If f L1 ([a, b)) with f a repre sentative then extending f as zero outside the interval gives an element of L1 (R), by dention. As an element of L1 (R) this does not depend on the choice of f and then (8.27) gives the restriction to [x, b) as an element of L1 ([x, b)). This is a linear map. (2) Using the discussion in the preceeding question, we now that if fn is an absolutely summable series converging to f (a representative of f ) where it converges absolutely, then for any a x b, we can dene (8.32)
fn = ([a, x))fn , fn = ([x, b))fn

LECTURE NOTES FOR 18.102, SPRING 2009

53

where ([a, b)) is the characteristic function of the interval. It follows that fn converges to f ([a, x)) and fn to f ([x, b)) where they converge abso lutely. Thus (8.33) f = f ([x, b)) = fn , f = f ([a, x)) = fn .
[x,b) n [a,x) n

Now, for step functions, we know that fn = (8.34) f= f+ f


[a,b) [a,x) [x,b)

fn

fn

so

as we have every right to expect. Thus it suces to show (by moving the end point from a to a general point) that (8.35) lim f =0
x a [a,x)

for any f integrable on [a, b). Thus can be seen in terms of a dening absolutely summable sequence of step functions using the usual estimate that (8.36) | f| | fn | + |fn |.
[a,x) [a,x) nN n>N [a,x)

The last sum can be made small, independent of x, by choosing N large enough. On the other hand as x a the rst integral, for xed N, tends to zero by the denition for step functions. This proves (8.36) and hence the continuity of F. (3) If the function x1 cos(1/x) were Lebesgue integrable on the interval (0, 1] (on which it is dened) then it would be integrable on [0, 1) if we dene it arbitrarily, say to be 0, at 0. The same would be true of the absolute value and Riemann integration shows us easily that 1 (8.37) lim x| cos(1/x)|dx = .
t0 t

This is contrary to the continuity of the integral as a function of the limits just shown. Problem 3.4 [Harder but still doable] Suppose f L1 (R). (1) Show that for each t R the translates (8.38) ft (x) = f (x t) : R C are elements of L1 (R). (2) Show that (8.39)
t0

lim

|ft f | = 0.

This is called Continuity in the mean for integrable functions. Hint: I will add one! (3) Conclude that for each f L1 (R) the map (it is a curve) (8.40) is continuous.
Solution:
R t [ft ] L1 (R)

54

LECTURE NOTES FOR 18.102, SPRING 2009

(1) If fn is an absolutely summable series of step functions converging to f where it converges absolutely then fn ( t) is such a series converging to f ( t) for each t R. Thus, each of the f (x t) is Lebesgue integrable, i.e. are elements of L1 (R) (2) Now, we know that if fn is a series converging to f as above then (8.41) |f | |fn |.
n

We can sum the rst terms and then start the series again and so it follows that for any N, (8.42) |f | | fn | + |fn |.
nN n>N

(8.43)

Applying this to the series fn ( t) fn () we nd that |ft f | | fn ( t) fn ()| + |fn ( t) fn ()|


nN n>N

The second sum here is bounded by 2


n>N

|fn |. Given > 0 we can choose

N so large that this sum is bounded by /2, by the absolute convergence. So the result is reduce to proving that if |t| is small enough then (8.44) | fn ( t) fn ()| /2.
nN

This however is a nite sum of step functions. So it suces to show that (8.45) | g( t) g()| 0 as t 0 for each component, i.e. a constant, c, times the characteristic function of an interval [a, b) where it is bounded by 2|c||t|. (3) For the curve ft which is a map (8.46) R t ft L1 (R)

it follows that ft+s = (ft )s so we can apply the argument above to show that for each s, (8.47) lim |ft fs | = 0 = lim [ft ] [fs ]L1 = 0
ts ts

which proves continuity of the map (8.46). Problem 3.5 In the last problem set you showed that a continuous function on a compact interval, extended to be zero outside, is Lebesgue integrable. Using this, and the fact that step functions are dense in L1 (R) show that the linear space of continuous functions on R each of which vanishes outside a compact set (which depends on the function) form a dense subset of L1 (R). Solution: Since we know that step functions (really of course the equivalence classes of step functions) are dense in L1 (R) we only need to show that any step function is the limit of a sequence of continuous functions each vanishing outside a

LECTURE NOTES FOR 18.102, SPRING 2009

55

compact set, with respect to L1 . So, it suces to prove this for the charactertistic function of an interval [a, b) and then multiply by constants and add. The sequence 0 x < a 1/n n(x a + 1/n) a 1/n x a (8.48) gn (x) = 0 a<x<b n(b + 1/n x) b x b + 1/n 0 x > b + 1/n is clearly continuous and vanishes outside a compact set. Since 1 b+1/n (8.49) |gn ([a, b))| = gn + gn 2/n
a1/n b

it follows that [gn ] [([a, b))] in L1 (R). This proves the density of continuous functions with compact support in L1 (R). Problem 3.6 (1) If g : R C is bounded and continuous and f L1 (R) show that gf L1 (R) and that (8.50) |gf | sup |g | |f |.
R

(2) Suppose now that G C([0, 1] [0, 1]) is a continuous function (I use C(K) to denote the continuous functions on a compact metric space). Recall from the preceeding discussion that we have dened L1 ([0, 1]). Now, using the rst part show that if f L1 ([0, 1]) then (8.51) F (x) = G(x, () C )f
[0,1]

(where is the variable in which the integral is taken) is well-dened for each x [0, 1]. (3) Show that for each f L1 ([0, 1]), F is a continuous function on [0, 1]. (4) Show that (8.52) L1 ([0, 1]) f F C([0, 1]) is a bounded (i.e. continuous) linear map into the Banach space of contin uous functions, with supremum norm, on [0, 1]. Solution: (1) Lets rst assume that f = 0 outside [1, 1]. Applying a result form Prob lem set there exists a sequence of step functions gn such that for any R, gn g uniformly on [0, 1). By passing to a subsequence we can arrange that sup[1,1] |gn (x) gn1 (x)| < 2n . If fn is an absolutly summable se ries of step functions converging a.e. to f we can replace it by fn ([1, 1]) as discussed above, and still have the same conclusion. Thus, from the uniform convergence of gn , (8.53) gn (x)
n k=1

fk (x) g(x)f (x) a.e. on R.

56

LECTURE NOTES FOR 18.102, SPRING 2009 n k=1 n1 k=1

So dene h1 = g1 f1 , hn = gn (x) (8.54) |hn | A|fn (x)| + 2n


k<n

fk (x) gn1 (x)

fk (x). This series

of step functions converges to gf (x) almost everywhere and since |fk (x)|,
n

|hn | A

|fn | + 2

|fn | <

it is absolutely summable. Here A is a bound for |gn | independent of n. Thus gf L1 (R) under the assumption that f = 0 outside [0, 1) and (8.55) |gf | sup |g| |f | follows from the limiting argument. Now we can apply this argument to fp which is the restriction of p to the interval [p, p + 1), for each p Z. Then we get gf as the limit a.e. of the absolutely summable series gfp where (8.55) provides the absolute summablitly since (8.56) |gfp | sup |g| |f | < .
p 1 p [p,p+1)

Thus, gf L (R) by a theorem in class and (8.57) |gf | sup |g| |f |. (2) If f L1 [(0, 1]) has a representative f then G(x, )f () L1 ([0, 1)) so (8.58) F (x) = G(x, )f () C
[0,1]

is well-dened, since it is indpendent of the choice of f , changing by a null function if f is changed by a null function. (3) Now by the uniform continuity of continuous functions on a compact metric space such as S = [0, 1] [0, 1] given > 0 there exist > 0 such that (8.59) sup |G(x, y) G(x , y)| < if |x x | < .
y[0,1]

Then if |x x | < , (8.60) |F (x) F (x )| = |


[0,1]

(G(x, ) G(x , ))f ()|

|f |.

Thus F C([0, 1]) is a continuous function on [0, 1]. Moreover the map f F is linear and (8.61) sup |F | sup |G| ||f |
[0,1] S [0,1]

which is the desired boundedness, or continuity, of the map (8.62) I : L1 ([0, 1]) C([0, 1]), F (f )(x) = G(x, )f (), I(f )sup sup |G|f L1 .

MIT OpenCourseWare http://ocw.mit.edu

18.102 Introduction to Functional Analysis


Spring 2009

For information about citing these materials or our Terms of Use, visit: http://ocw.mit.edu/terms.

LECTURE NOTES FOR 18.102, SPRING 2009

57

Lecture 9. Thursday, March 5 My attempts to distract you all during the test did not seem to work very well. Here is what I wrote up on the board, more or less. We will use Baires theorem later (it is also known as Baire category theory although it has nothing to do with categories in the modern sense). This is a theorem about complete metric spaces it could be included in 18.100B but the main applications are in Functional Analysis. Theorem 4 (Baire). If M is a non-empty complete metric space and Cn M, n N, are closed subsets such that
(9.1) M= Cn
n

then at least one of the Cn s has an interior point. Proof. We can assume that the rst set C1 = since they cannot all be empty and dropping some empty sets does no harm. Lets assume the contrary of the desired conclusion, namely that each of the Cn s has empty interior, hoping to arrive at a contradiction to (9.1) using the other properties. This means that an open ball B(p, ) around a point of M (so it isnt empty) cannot be contained in any one of the Cn . So, choose p C1 . Now, there must be a point p1 B(p, 1/3) which is not in C1 . Since C1 is closed there exists 1 > 0, and we can take 1 < 1/3, such that B(p1 , 1 ) C1 = . Continue in this way, choose p2 B(p1 , 1 /3) which is not in C2 and 2 > 0, 2 < 1 /3 such that B(p2 , 2 ) C2 = . Here we use both the fact that C2 has empty interior and the fact that it is closed. So, inductively there is a sequence pi , i = 1, . . . , k and positive numbers 0 < k < k1 /3 < k2 /32 < < 1 /3k1 < 3k such that pj B(pj1 , j1 /3) and B(pj , j ) Cj = . Then we can add another pk+1 by using the properties of Ck it has non-empty interior so there is some point in B(pk , k /3) which is not in Ck+1 and then B(pk+1 , k+1 ) Ck+1 = where k+1 > 0 but k+1 < k /3. Thus, we have a sequence {pk } in M. Since d(pk+1 , pk ) < k /3 this is a Cauchy sequence, in fact (9.2) d(pk , pk+l ) < k /3 + + k+l1 /3 < 3k .

Since M is complete the sequence converges to a limit, q M. Notice however that / pl B(pk , 2k /3) for all k > l so d(pk , q) 2k /3 which implies that q Ck for any k. This is the desired contradiction to (9.1). Thus, at least one of the Cn must have non-empty interior. One application of this, which we will get to later, is the uniform boundedness principle (which is just a theorem). Theorem 5. Let B be a Banach space and suppose that Tk is a sequence of bounded (i.e. continuous) linear operators Tn : B V where V is a normed space. Suppose that for each b B the set {Tn (b)} V is bounded (in norm of course) then supn Tn < . Proof. You can look it up, but it follows from an application of Baires theorem to the sets (9.3) Sp = {b B, b < 1, Tn bV p n}, p N..

58

LECTURE NOTES FOR 18.102, SPRING 2009

You can check that these are closed and that their union must be the closed ball of radius one around the origin in B (because of the assumption of pointwise boundedness) So, by Baires theorem one of them has non-empty interior. This means that for some p, some v Sp and some > 0, (9.4) (9.5) w B, wB = Tn (v + w)V p n. w B, wB = Tn (w)V 2p = Tn 2p/ Using the triangle inequality, and the fact that Tn (v)V p this means since the norm of the operator is sup{T wV ; wB = 1 Why this should be useful we shall see!

MIT OpenCourseWare http://ocw.mit.edu

18.102 Introduction to Functional Analysis


Spring 2009

For information about citing these materials or our Terms of Use, visit: http://ocw.mit.edu/terms.

LECTURE NOTES FOR 18.102, SPRING 2009

59

Lecture 10. Tuesday, Mar 10 All of this is easy to nd in the various reference notes and/or books so I will keep these notes very brief. (1) Bessels inequality If in a preHilbert space H, ei , i = 1, . . . , N are orthonormal so (ei , ej ) = ij then for any element u H, set v= (10.1) v2 = H
N (u, ei )ei then i=1 N i=1

|(u, ei )|2 u2 , H

(u v) ei , i = 1, . . . , N. The last statement follows immediately by computing (u, ej ) = (v, ej ) and similarly v2 can be computed directly. Then the inequality, which is Bessels inequality, follows from Cauchys inequality since from the last statement (10.2) v2 = (v, v) = (v, u) + (v, v u) = (v, u) = |(v, u)| vu shows v u. (2) Orthonormal bases: Since in the inequality in (10.1) the right side is independent of N it follows that if {ei } is a countable orthonormal set then i=1 (10.3)
i=1

|(u, ei )|2 u2 . H

From this it follows that the sequence n (10.4) vn = (u, ei )ei


i=1

is Cauchy since if m > n, (10.5) vn vm 2 = (u, ej )|2 (u, ej )|2


n<jm j=n+1

and the right side is small if n is large, independent of m. Lemma 5. If H is a Hilbert space so now we assume completeness and {ei } is an orthonormal sequence then for each u H, i=1 (10.6) v=
j=1

(u, ej )ej H

converges and (u v) ej for all j. Proof. The limit exists since the sequence is Cauchy and the space is com plete. The orthogonality follows from the fact that (u vn , ej ) = 0 as soon as n j and (10.7) (u v, ej ) = lim (u vn , ej ) = 0
n

60

LECTURE NOTES FOR 18.102, SPRING 2009

by continuity of the inner product (which follows from Cauchys inequality). Now, we say an orthonormal sequence is complete, or is and orthonormal basis of H if u ej = 0 for all j implies u = 0. Then we see: Proposition 15. If {ej is an orthonormal basis in a Hilbert space H j=1 then (10.8) u= (u, ej )ej u H.
j=1

Proof. From the lemma the series converges to v and (u v) ej for all j so by the assumed completeness, u = v which is (10.8). (3) Gram-Schmidt Theorem 6. Every separable Hilbert space has an orthonormal basis. Proof. Take a countable dense subset which can be arranged as a se quence {vj } and the existence of which is the denition of separability and orthonormalize it. Thus if v1 = 0 set ei = v1 /v1 . Proceeding by induction we can suppose to have found for a given integer n elements ei , i = 1, . . . , m, where m n, which are orthonormal and such that the linear span (10.9) sp(e1 , . . . , em ) = sp(v1 , . . . , vn ). To show the inductive step observe that if vn+1 is in the span(s) in (10.9) then the same ei work for n + 1. So it follows that n w (10.10) w = vn+1 (vn+1 , ej )ej = 0 so em+1 = w j=1 makes sense. Adding em+1 gives the equality of the spans for n + 1. Thus we may continue indenitely. There are only two possibilities, either we get a nite set of ei s or an innite sequence. In either case this must be an orthonormal basis. That is we claim (10.11) H u ej j = u = 0. This uses the density of the vn s. That is, there must exist a sequence wj where each wj is a vn , such that wj u in H. Now, each each vn , and hence each wj , is a nite linear combination of ek s so, by Bessels inequality (10.12) wj 2 = |(wj , ek )|2 = |(u wj , ek )|2 u wj 2
k k

where (u, ej ) = 0 for all j has been used. Thus wj 0 and u = 0.


2

(4) Isomorphism to l A nite dimensional Hilbert space is isomorphic to Cn with its standard inner product. Similarly from the result above Proposition 16. Any innite-dimensional separable Hilbert space (over the complex numbers) is isomorphic to l2 , that is there exists a linear map (10.13) T : H L2

LECTURE NOTES FOR 18.102, SPRING 2009

61

which is 1-1, onto and satises (T u, T v)l2 = (u, v)H and T ul2 = uH for all u, v H. Proof. Choose an orthonormal basis which exists by the discussion above and set (10.14) T u = {(u, ej ) . j=1 This maps H into l2 by Bessels inequality. Moreover, it is linear since the entries in the sequence are linear in u. It is 1-1 since T u = 0 implies (u, ej ) = 0 for all j implies u = 0 by the assumed completeness of the orthonormal basis. It is surjective since if {cj } then j=1 (10.15) u=
j=1

cj ej

converges in H. This is the same argument as above the sequence of partial sums is Cauchy by Bessels inequality. Again by continuity of the inner product, T u = {cj } so T is surjective. The equality of the norms follows from equality of the inner products and the latter follows by computation for nite linear combinations of the ej and then in general by continuity.

62

LECTURE NOTES FOR 18.102, SPRING 2009

Problem set 5, Due 11AM Tuesday 17 Mar. You should be thinking about using Lebesgues dominated convergence at several points below. Problem 5.1 Let f : R C be an element of L1 (R). Dene f (x) x [L, L] (10.16) fL (x) = 0 otherwise. Show that fL L1 (R) and that |fL f | 0 as L . Problem 5.2 Consider a real-valued function f : R R which is locally inte grable in the sense that f (x) x [L, L] (10.17) gL (x) = 0 x R \ [L, L] is Lebesgue integrable of each L N. (1) Show that for each xed L the function gL (x) if gL (x) [N, N ] (N ) (10.18) gL (x) = N if gL (x) > N N if gL (x) < N is Lebesgue integrable. (N ) (2) Show that |gL gL | 0 as N . (3) Show that there is a sequence, hn , of step functions such that (10.19) (4) Dening 0 h (x) n = N N
(N ) (N )

hn (x) f (x) a.e. in R. x [L, L] if hn (x) [N, N ], x [L, L] . if hn (x) > N, x [L, L] if hn (x) < N, x [L, L]

(10.20)

hn,L

(N )

Show that

|hn,L gL | 0 as n .

Problem 5.3 Show that L2 (R) is a Hilbert space. First working with real functions, dene L2 (R) as the set of functions f : R R which are locally integrable and such that |f |2 is integrable. (1) For such f choose hn and dene gL , gL and hn by (10.17), (10.18) and (10.20). (N ) (N ) (N ) (2) Show using the sequence hn,L for xed N and L that gL and (gL )2 are (N ) (N ) in L1 (R) and that |(hn,L )2 (gL )2 | 0 as n . (N ) (3) Show that (gL )2 L1 (R) and that |(gL )2 (gL )2 | 0 as N . 2 2 (4) Show that |(gL ) f | 0 as L . (5) Show that f, g L2 (R) then f g L1 (R) and that 2 (10.21) | f g| |f g| f L2 gL2 , f L2 = |f |2 .
(N ) (N )

LECTURE NOTES FOR 18.102, SPRING 2009

63

(6) Use these constructions to show that L2 (R) is a linear space. (7) Conclude that the quotient space L2 (R) = L2 (R)/N , where N is the space of null functions, is a real Hilbert space. (8) Extend the arguments to the case of complex-valued functions. Problem 5.4 Consider the sequence space (10.22) h2,1 = c : N j cj C; (1 + j 2 )|cj |2 < .
j

(1) Show that (10.23) h2,1 h2,1 (c, d) c, d = (1 + j 2 )cj dj


j 2,1

is an Hermitian inner form which turns h into a Hilbert space. (2) Denoting the norm on this space by 2,1 and the norm on l2 by 2 , show that (10.24) h2,1 l2 , c2 c2,1 c h2,1 . Problem 5.5 In the separable case, prove Riesz Representation Theorem directly. Choose an orthonormal basis {ei } of the separable Hilbert space H. Suppose T : H C is a bounded linear functional. Dene a sequence (10.25) wi = T (ei ), i N. (1) Now, recall that |T u| CuH for some constant C. Show that for every nite N, (10.26)
2 N j=1

|wi |2 C 2 .

(2) Conclude that {wi } l and that (10.27) w= wi ei H.


i

(3) Show that (10.28) T (u) = u, wH u H and T = wH .

64

LECTURE NOTES FOR 18.102, SPRING 2009

Solutions to Problem set 4 Just to compensate for last week, I will make this problem set too short and easy! Problem 4.1 Let H be a normed space in which the norm satises the parallelogram law: (10.29) u + v2 + u v2 = 2(u2 + v2 ) u, v H. Show that the norm comes from a positive denite sesquilinear (i.e. Hermitian) inner product. Big Hint:- Try 1 (10.30) (u, v) = u + v2 u v2 + iu + iv2 iu iv2 ! 4 Solution: Setting u = v, even without the parallelogram law, 1 (10.31) (u, u) = 2u2 + i(1 + i)u2 i(1 i)u2 = u2 . 4 So the point is that the parallelogram law shows that (u, v) is indeed an Hermitian inner product. Taking complex conjugates and using properties of the norm, u + iv = v iu etc 1 v + u2 v u2 iv iu2 + iv + iu2 = (v, u). (10.32) (u, v) = 4 Thus we only need check the linearity in the rst variable. This is a little tricky! First compute away. Directly from the identity (u, v) = (u, v) so (u, v) = (u, v) using (10.32). Now, (10.33) 1 (2u, v) = u + (u + v)2 u + (u v)2 4 + iu + (u + iv)2 iu + (u iv)2 1
= u + v2 + u2 u v2 u2
2 + i(u + iv)2 + iu2 iu iv2 iu2 1 u (u + v)2 u (u v)2 + iu (u + iv)2 iu (u iv)2 4 =2(u, v). Using this and (10.32), for any u, u and v, 1 (u + u , v) = (u + u , 2v) 2 1 1 = (u + v) + (u + v)2 (u v) + (u v)2 24 + i(u + iv) + (u iv)2 i(u iv) + (u iv)2 1 = u + v + u + v2 u v u v2 (10.34) 4 + i(u + iv)2 + iu iv2 iu iv iu iv2 1 1 (u + v) (u + v)2 (u v) (u v)2 24 + i(u + iv) (u iv)2 i(u iv) = (u iv)2 = (u, v) + (u , v).

LECTURE NOTES FOR 18.102, SPRING 2009

65

Using the second identity to iterate the rst it follows that (ku, v) = k(u, v) for any u and v and any positive integer k. Then setting nu = u for any other positive integer and r = k/n, it follows that (10.35) (ru, v) = (ku , v) = k(u , v) = rn(u , v) = r(u, v)

where the identity is reversed. Thus it follows that (ru, v) = r(u, v) for any rational r. Now, from the denition both sides are continuous in the rst element, with respect to the norm, so we can pass to the limit as r x in R. Also directly from the denition, 1 iu + v2 iu v2 + iiu + iv2 iiu iv2 = i(u, v) (10.36) (iu, v) = 4 so now full linearity in the rst variable follows and that is all we need. Problem 4.2 Let H be a nite dimensional (pre)Hilbert space. So, by denition H has a basis {vi }n , meaning that any element of H can be written i=1 (10.37) v= c i vi
i

and there is no dependence relation between the vi s the presentation of v = 0 in the form (10.37) is unique. Show that H has an orthonormal basis, {ei }n satis i=1 fying (ei , ej ) = ij (= 1 if i = j and 0 otherwise). Check that for the orthonormal basis the coecients in (10.37) are ci = (v, ei ) and that the map (10.38) T : H v ((v, ei )) Cn

is a linear isomorphism with the properties (10.39) (u, v) = (T u)i (T v)i , uH = T uCn u, v H.
i

Why is a nite dimensional preHilbert space a Hilbert space? Solution: Since H is assumed to be nite dimensional, it has a basis vi , i = 1, . . . , n. This basis can be replaced by an orthonormal basis in n steps. First replace v1 by e1 = v1 /v1 where v1 = 0 by the linear indepedence of the basis. Then replace v2 by (10.40) e2 = w2 /w2 , w2 = v2 v2 , e1 e1 .

Here w2 e1 as follows by taking inner products; w2 cannot vanish since v2 and e1 must be linearly independent. Proceeding by nite induction we may assume that we have replaced v1 , v2 , . . . , vk , k < n, by e1 , e2 , . . . , ek which are orthonormal and span the same subspace as the vi s i = 1, . . . , k. Then replace vk+1 by (10.41) ek+1 = wk+1 /wk+1 , wk+1 = vk+1
k vk+1 , ei ei . i=1

By taking inner products, wk+1 ei , i = 1, . . . , k and wk+1 = 0 by the linear independence of the vi s. Thus the orthonormal set has been increased by one element preserving the same properties and hence the basis can be orthonormalized. Now, for each u H set (10.42) ci = u, ei .

66

LECTURE NOTES FOR 18.102, SPRING 2009 n i=1

It follows that U = u (10.43)

ci ei is orthogonal to all the ei since


i

u, ej = u, ej

ci ei , ej = u.ej cj = 0.
i

This implies that U = 0 since writing U =

di ei it follows that di = U, ei = 0.

Now, consider the map (10.38). We have just shown that this map is injective, since T u = 0 implies ci = 0 for all i and hence u = 0. It is linear since the ci depend linearly on u by the linearity of the inner product in the rst variable. Moreover it is surjective, since for any ci C, u = ci ei reproduces the ci through (10.42).
i

Thus T is a linear isomorphism and the rst identity in (10.39) follows by direct computation: n (T u)i (T v)i = u, ei
i=1 i

(10.44)

= u,

v, ei ei

= u, v. Setting u = v shows that T uCn = uH . Now, we know that Cn is complete with its standard norm. Since T is an isomorphism, it carries Cauchy sequences in H to Cauchy sequences in Cn and T 1 carries convergent sequences in Cn to convergent sequences in H, so every Cauchy sequence in H is convergent. Thus H is complete.

MIT OpenCourseWare http://ocw.mit.edu

18.102 Introduction to Functional Analysis


Spring 2009

For information about citing these materials or our Terms of Use, visit: http://ocw.mit.edu/terms.

LECTURE NOTES FOR 18.102, SPRING 2009

67

Lecture 11. Thursday, 12 Mar Quite a lot of new material, but all of it in the various notes and books. So, I will keep it brief. (1) Convex sets and length minimizer The following result does not need the hypothesis of separability of the Hilbert space and allows us to prove the subsequent results especially Riesz theorem in full generality. Proposition 17. If C H is a subset of a Hilbert space which is (a) Non-empty (b) Closed (c) Convex, in the sense that v1 , v1 C implies 1 (v1 + v2 ) C 2 then there exists a unique element v C closest to the origin, i.e. such that (11.1) vH = inf uH .
uC

Proof. By denition of inf there must exist a sequence {vn } in C such that vn d = inf uC uH . We show that vn converges and that the limit is the point we want. The parallelogram law can be written (11.2) vn vm 2 = 2vn 2 + 2vm 2 4(vn + vm )/22 . Since vn d, given > 0 if N is large enough then n > N implies 2vn 2 < 2d2 +2 /2. By convexity, (vn +vm )/2 C so (vn +vm )/22 d2 . Combining these estimates gives (11.3) n, m > N =uC uH 4d2 + 2 4d2 so {vn } is Cauchy. Since H is complete, vn v C since C is closed. Moreover, the distance is continuous so vH = limn vn = d. Thus v exists and uniqueness follows again from the parallelogram law. If v and v are two points in C with v = v = d then (v + v )/2 C so (11.4) v v 2 = 2v2 + 2v 2 4(v + v )/22 0 = v = v . (2) Orthocomplements Proposition 18. If W H is a linear subspace of a Hilbert space the (11.5) W = {u H; (u, w) = 0 w W } is also a linear subspace and W W = {0}. If W is also closed then (11.6) H = W W meaning that any u H has a unique decomposition u = w + w where w W and w W . Proof. That W dened by (11.5) is a linear subspace follows from the linearity of the condition dening it. If u W and u W u u by the denition so (u, u) = u2 = 0 and u = 0. Now, suppose W is closed. If W = H then W = {0} and there is / nothing to show. So consider u H, u W. Consider (11.7) C = u + W = {u H; u = u + w, w W }.

68

LECTURE NOTES FOR 18.102, SPRING 2009

Then C is closed, since a sequence in it is of the form u = u+wn where wn n is a sequence in W and u converges if and only if wn converges. Now, C is n non-empty, since u C and it is convex since u = u + w and u = u + w in C implies (u + u )/2 = u + (w + w )/2 C. Thus the length minimization result above applies and there exists a unique v C such that v = inf u C u . The claim is that this v is perpendicular to W draw a picture in two real dimensions! To see this consider an aritrary point w W and C then v + w C and (11.8) v + w2 = v2 + 2 Re((v, w)) + ||2 w2 . Choose = tei where the phase is chosen so that ei (v, w) = |(v, w)| 0. Then the fact that v is minimal means that (11.9) t(2|(v, w)| + tw2 ) 0 t R = |(v, w)| = 0 which is what we wanted to show. Thus indeed, give u W we have constructed v W such that u = / v + w, w W. This is (11.6) with the uniqueness of the decomposition already shown since it reduces to 0 having only the decomposition 0 + 0 and this in turn is W W = {0}. (3) Riesz theorem The most important application of these results is to prove Riesz rep resentation theorem (for Hilbert space, there is another one to do with measures). Theorem 7. If H is a Hilbert space then any continuous linear functional T : H C there exists a unique element H such that (11.10) T (u) = (u, ) u H. Proof. (a) Here is the proof I gave quickly in Lecture 10, not using the preceeding Lemma. If T is the zero functional then w = 0 satises (11.10). Otherwise there exists some u H such that T (u ) = 0 and then u H, namely u = u /F (u ), such that F (u) = 1. Thus (11.11) C = {u H; T (u) = 1} = T 1 ({1}) is non-empty. The continuity of T and the second form shows that C is closed, as the inverse image of a closed set under a continuous map. Moreover C is convex since (11.12) T ((u + u )/2) = (T (u) + T (u ))/2. Thus, there exists an element v C of minimal length. As in the proof above, it follows that v + w2 v2 for all w W and C this implies that v W . Now continue as in the proof below. (b) Here is the proof I gave in Lecture 11 using the orthocomplement above. Since T is continuous the null space (11.13) W = T 1 ({0}) = {u H; T (u) = 0} is a closed linear subspace. Thus (11.14) H = W W

LECTURE NOTES FOR 18.102, SPRING 2009

69

by Proposition 18 above. Now, if T = 0 is the zero functional then W = H and W = {0} and w = 0 works in (11.10). Othewise, W v , which is not in W, i.e. has T (v ) = 0 and hence v W with T (v) = 1. Then for any u H, (11.15)
uT (u)v satises T (uT (u)v) = T (u)T (u)T (v) = 0 u = w+T (u)v, w W.
Then, (u, v) = T (u)v2 since (w, v) = 0. Thus if = v/v2 then (11.16) u = w + (u, )v = T (u) = (u, )T (v) = (u, ). (4) Adjoints of bounded operators. As an application of Riesz theorem I showed that any bounded linear operator on a Hilbert space (11.17) A : H H, HuH CuH u H has a unique adjoint operator. That is there exists a unique bounded linear operator A : H H such that (11.18) (Au, v)H = (u, A v) u, v H. To see the existence of A v we need to work out what A v should be for each xed v H. So, x v in the desired identity (11.18), which is to say consider (11.19) (11.20) H u (Au, v) C. This is a linear map and it is clearly bounded, since |(Au, v)| AuH vH (CvH )uH . Thus it is a continuous linear functional on H which depends on v. In fact it is just the composite of two continouos linear maps (11.21) H H
uAu w(w,v)

C.

By Riesz theorem there exists an unique element in H, which we can denote A v (since it only depends on v) such that (11.22) (Au, v) = (u, A v) u H. Now this denes the map A H H but we need to check that it is linear and continuous. Linearity follows from the uniqueness part of Riesz theorem. Thus if v1 , v2 H and c1 , c2 C then (11.23) (Au, c1 v1 + c2 v2 ) = c1 (Au, v1 ) + c2 (Au, v2 ) = c1 (u, A v1 ) + c2 (u, A v2 ) = (u, c1 A v2 + c2 A v2 ) where we have used the denitions of A v1 and A v2 by uniqueness we must have A (c1 v1 + c2 v2 ) = c1 A v1 + c2 A v2 . Since we know the optimality of Cauchys inequality (11.24) vH = sup |(u, v)|
u=1

(do we? If not set u = v/v to see it.) it follows that (11.25) A v = sup |(u, A v)| = sup |(Au, v)| Av.
u=1 u=1

70

LECTURE NOTES FOR 18.102, SPRING 2009

So in fact (11.26) A A. In fact it is immediately the case that (A ) = A so the reverse equality also holds and so (11.27) A = A.

MIT OpenCourseWare http://ocw.mit.edu

18.102 Introduction to Functional Analysis


Spring 2009

For information about citing these materials or our Terms of Use, visit: http://ocw.mit.edu/terms.

LECTURE NOTES FOR 18.102, SPRING 2009

71

Lecture 12. Tuesay, Mar 17: Compactness and weak convergence A subset in a general metric space is one with the property that any sequence in it has a convergent subsequence, with its limit in the set. You will recall with pleasure no doubt the equivalence of this condition to the (more general since it makes good sense in an arbitrary topological space) equivalence of this with the covering condition, that any open cover of the set has a nite subcover. So, in a separable Hilbert space the notion of a compact set is already xed. We want to characterize it in the problems this week you will be asked to prove several characterizations. A general result in a metric space is that any compact set is both closed and bounded, so this must be true in a Hilbert space. The Heine-Borel theorem gives a converse to this, Rn or Cn (and hence in any nite dimensional normed space) any closed and bounded set is compact. Also recall that the convergence of a sequence in Cn is equivalent to the convergence of the n sequences given by its components and this is what is used to pass rst from R to C and then to Cn . All of this fails in innite dimensions and we need some condition in addition to being bounded and closed for a set to be compact. To see where this might come from, observe that a set, S, consisting of the points of a convergent sequence, s : N M, together with its limit, s, in any metric space is always compact. The set here is the image of the sequence, thought of as a map from the integers into the metric space, together with the limit (which might of course already be in the image). Certainly this set is bounded, since the distance from the intial point is certainly bounded. Moreover it is closed, although you might need to think about this for a minute. A sequence in the set which is the image of another sequence consists of elements of the original sequence in any order and maybe repeated at will. Since the original sequence may itself have reapeated points, the labelling of points is by no means unique. However S is closed since M \ S is open a point in p M \ S is at a nite no-zero distance, d(p, s) from the limit so B(p, d(p, s)/2) can contain only nitely many elements of S hence a smaller open ball does not meet it. Lemma 6. The image of a convergent sequence in a Hilbert space is a set with equi-small tails with respect to any orthonormal sequence, i.e. if ek is an othonormal sequence and un u is a convergent sequence then given > 0 there exists N such that (12.1) |(un , ek )|2 < 2 n.
k>N

Proof. Bessels inequality shows that for any u H, (12.2) |(u, ek )|2 u2 .
k

The convergence of this series means that (12.1) can be arranged for any single element un or the limit u by choosing N large enough, thus given > 0 we can choose N so that (12.3) |(u, ek )|2 < 2 /2.
k>N

72

LECTURE NOTES FOR 18.102, SPRING 2009

In fact, for any orthonormal sequence such as ek whether complete or not, (12.4) P : H u P u = (u, ek )ek H
k

is continuous and in fact has norm at most one. Indeed from Bessels inequality, P u2 u2 . Now, applying this to (12.5) PN u = (u, ek )ek
k>N

the convergence un u implies the convergence in norm PN un PN u and so (12.6) |(u, ek )|2 < 2 .
k>N

So, we have arranged (12.1) for n > n with N = N . Of course, this estimate remains valid if N is increased, and we may arrange it for n n by chossing N large enough. Thus indeed (12.1) holds for all n if N is chosen large enough. This suggest one useful characterization of compact sets in a separable Hilbert space. Proposition 19. A set K H in a separabel Hilbert space is compact if and only if it is bounded, closed and has equi-small tails with respect to any one orthonormal basis. Proof. We already know that a compact set is closed and bounded. Suppose the equi-smallness of tails condition fails with respect to some orthonormal basis ek . This means that for some > 0 and all N there is an element uN K such that (12.7) |(uN , ek )|2 2 .
k>N

Then the sequence {uN } can have no convergent subsequence, since this would contradict the Lemma we have just proved, hence K is not compact in this case. Thus we have proved the equi-smallness of tails condition to be necessary for the compactness of a closed, bounded set. So, it remains to show that it is sucient. So, suppose K is closed, bounded and satises the equi-small tails condition with respect to an orthonormal basis ek and {un } is a sequence in K. We only need show that {un } has a Cauchy subsequence, since this will converge (H being complete) and the limit will be in K (since it is closed). Now, consider each of the sequences of coecients (un , ek ) in C. Here k is xed. This sequence is bounded: (12.8) |(un , ek )| un C by the boundedness of K. So, by the Heine-Borel theorem, there is a subsequence of unl such that (unl , ek ) converges as l . We can apply this argument for each k = 1, 2, . . . . First extracting a subsequence of {un } so that the sequence (un , e1 ) converges along this subsequence. Then extract a subsequence of this subsequence so that (un , e2 ) also converges along this sparser subsequence, and continue inductively. Then pass to the diagonal subsequence of {un } which has kth entry the kth term in the kth subsequence. It is eventually a subsequence of each of the subsequences previously constructed meaning it coincides with a subsequence for some point onward (namely the

LECTURE NOTES FOR 18.102, SPRING 2009

73

kth term onward for the kth subsquence). Thus, for this subsequence each of the (unl , ek ) converges. Now, lets relabel this subsequence vn for simplicity of notation and consider Bessels identity (the orthonormal set ek is complete by assumption) for the dier ence vn vn+l 2 = |(vn vn+l , ek )|2 + |(vn vn+l , ek )|2 H (12.9)
kN k>N 2

kN

|(vn vn+l , ek )| + 2

k>N

|(vn , ek )|2 + 2

k>N

|(vn+l , ek )|2

where the parallelogram law on C has been used. To make this sum less than 2 we may choose N so large that the last two terms are less than 2 /2 and this may be done for all n and l by the equi-smallness of the tails. Now, choose n so large that each of the terms in the rst sum is less than 2 /2N, for all l > 0 using the Cauchy condition on each of nite number of sequence (vn , ek ). Thus, {vn } is a Cauchy subsequence of {un } and hence as already noted convergent in K. Thus K is indeed compact. It is convenient to formalize the idea that each of the (un , ek ), the sequence of coecients of the Fourier-Bessel series, should converge. Denition 6. A sequence, {un }, in a Hilbert space, H, is said to converge weakly to an element u H if it is bounded in norm and (uj , v) (u, v) converges in C for each v H. This relationship is written (12.10) un u. In fact as we shall see next time, the assumption that un is bounded and that u exists are both unnecessary. That is, a sequence converges weakly if and only if (un , v) converges in C for each v H. Conversely, there is no harm in assuming it is bounded and that the weak limit u H exists. Note that the weak limit is unique since if u and u both have this property then (u u , v) = limn (un , v) limn (un , v) = 0 for all v H and setting v = u u it follows that u = u . Lemma 7. A (strongly) convergent sequence is weakly convergent with the same limit. Proof. This is the continuity of the inner product. If un u then (12.11) |(un , v) (u, v)| un uv 0 for each v H shows weak convergence.

Now, there is a couple of things I will prove here and leave some more to you for the homework. Lemma 8. For a bounded sequence in a separable Hilbert space, weak convergence is equivalent to component convergence with respect to an orthonormal basis. Proof. Let ek be an orthonormal basis. Then if un is weakly convergent it follows immediately that (un , ek ) (u, ek ) converges for each k. Conversely, suppose this is true for a bounded sequence, just that (un , ek ) ck in C for each k. The norm boundedness and Bessels inequality show that (12.12) |ck |2 = lim |(un , ek )|2 C 2 sup un 2
kp n kp n

74

LECTURE NOTES FOR 18.102, SPRING 2009

for all p. Thus in fact {ck } l2 and hence (12.13) u= wk ek H


k

by the completeness of H. Clearly (un , ek ) (u, ek ) for each k. It remains to show thta (un , v) (u, v) for all v H. This is certainly true for any nite linear combination of the ek and for a general v we can write (12.14) (un , v) (u, v) = (un , vp ) (u, vp ) + (un , v vp ) (u, v vp ) = |(un , v) (u, v)| = |(un , vp ) (u, vp )| + 2Cv vp where vp =
kp

(v, ek )ek is a nite part of the Fourier-Bessel series for v and C is a

bound for un . Now the convergence vp v implies that the last term in (12.14) can be made small by choosing p large, independent of n. Then the second last term can be made small by choosing n large since vp is a nite linear combination of the ek . Thus indeed, (un , v) (u, v) for all v H an it follows that un converges weakly to u. Proposition 20. Any bounded sequence {un } in a separable Hilbert space has a weakly convergent subsequence. This can be thought of as an analogue in innite dimensions of the Heine-Borel theorem if you say a bounded closed subset of a separable Hilbert space is weakly compact. Proof. Choose an orthonormal basis ek and apply the procedure in the proof of Proposition 19 to extract a subsequence of the given bounded sequence such that (unp , ek ) converges for each k. Now apply the preceeding Lemma to conclude that this subsequence converges weakly. Lemma 9. For a weakly convergent sequence un u (12.15) u lim inf un . Proof. Choose an orthonormal basis ek and observe that (12.16) u, ek 2 = lim un , ek 2 .
kp n

Now the sequence on the right is bounded by un 2 independently of p so (12.17) u, ek 2 lim inf un 2
kp n

by the denition of lim inf . Now, take p to conclude that (12.18) from which (12.15) follows. u2 lim inf un 2
n

LECTURE NOTES FOR 18.102, SPRING 2009

75

Problems 6: Due 11AM Tuesday, 31 Mar Hint: Dont pay too much attention to my hints, sometimes they are a little othe-cu and may not be very helpfult. An example being the old hint for Problem 6.2! Problem 6.1 Let H be a separable Hilbert space. Show that K H is compact if and only if it is closed, bounded and has the property that any sequence in K which is weakly convergent sequence in H is (strongly) convergent. Hint:- In one direction use the result from class that any bounded sequence has a weakly convergent subsequence. Problem 6.2 Show that, in a separable Hilbert space, a weakly convergent se quence {vn }, is (strongly) convergent if and only if the weak limit, v satises (12.19) vH = lim vn H .
n

Hint:- To show that this condition is sucient, expand (12.20) (vn v, vn v) = vn 2 2 Re(vn , v) + v2 . Problem 6.3 Show that a subset of a separable Hilbert space is compact if and only if it is closed and bounded and has the property of nite dimensional approxi mation meaning that for any > 0 there exists a linear subspace DN H of nite dimension such that (12.21) d(K, DN ) = sup inf {d(u, v)} .
uK vDN

Hint:- To prove necessity of this condition use the equi-small tails property of compact sets with respect to an orthonormal basis. To use the nite dimensional approximation condition to show that any weakly convergent sequence in K is strongly convergent, use the convexity result from class to dene the sequence {vn } in DN where vn is the closest point in DN to vn . Show that vn is weakly, hence strongly, convergent and hence deduce that {vn } is Cauchy. Problem 6.4 Suppose that A : H H is a bounded linear operator with the property that A(H) H is nite dimensional. Show that if vn is weakly convergent in H then Avn is strongly convergent in H. Problem 6.5 Suppose that H1 and H2 are two dierent Hilbert spaces and A : H1 H2 is a bounded linear operator. Show that there is a unique bounded linear operator (the adjoint) A : H2 H1 with the property (12.22) Au1 , u2 H2 = u1 , A u2 H1 u1 H1 , u2 H2 .

76

LECTURE NOTES FOR 18.102, SPRING 2009

Solutions to Problem set 5 You should be thinking about using Lebesgues dominated convergence at several points below. Problem 5.1 Let f : R C be an element of L1 (R). Dene f (x) x [L, L] (12.23) fL (x) = 0 otherwise. 1 Show that fL L (R) and that |fL f | 0 as L . Solution. If L is the characteristic function of [N, N ] then fL = f L . If fn is an absolutely summable series of step functions converging a.e. to f then fn L is absolutely summable, since |fn L | |fn | and converges a.e. to fL , so 1 fL L (R). Certainly |fL (x) f (x)| 0 for each x as L and |fL (x) f (x)| |fl (x)| + |f (x)| 2|f (x)| so by Lebesgues dominated convergence, |f fL | 0. Problem 5.2 Consider a real-valued function f : R R which is locally inte grable in the sense that f (x) x [L, L] (12.24) gL (x) = 0 x R \ [L, L] is Lebesgue integrable of each L N. (1) Show that for each xed L the function gL (x) if gL (x) [N, N ] (N ) (12.25) gL (x) = N if gL (x) > N N if gL (x) < N is Lebesgue integrable. (N ) (2) Show that |gL gL | 0 as N . (3) Show that there is a sequence, hn , of step functions such that (12.26) (4) Dening 0 h (x) n = N N x [L, L] if hn (x) [N, N ], x [L, L] . if hn (x) > N, x [L, L] if hn (x) < N, x [L, L] hn (x) f (x) a.e. in R.

(12.27)

hn,L

(N )

(N ) (N ) Show that |hn,L gL | 0 as n . Solution: (N ) (1) By denition gL = max(N L , min(N L , gL )) where L is the characteristic funciton of [L, L], thus it is in L1 (R). (N ) (N ) (2) Clearly gL (x) gL (x) for every x and |gL (x)| |gL (x)| so by Dom (N ) (N ) inated Convergence, gL gL in L1 , i.e. |gL gL | 0 as N since the sequence converges to 0 pointwise and is bounded by 2|g(x)|. (3) Let SL,n be a sequence of step functions converging a.e. to gL for ex ample the sequence of partial sums of an absolutely summable series of step functions converging to gL which exists by the assumed integrability.

LECTURE NOTES FOR 18.102, SPRING 2009

77

Then replacing SL,n by SL,n L we can assume that the elements all van ish outside [N, N ] but still have convergence a.e. to gL . Now take the sequence Sk,nk on [k, k] \ [(k 1), (k 1)], 1 k n, (12.28) hn (x) = 0 on R \ [n, n]. This is certainly a sequence of step functions since it is a nite sum of step functions for each n and on [L, L] \ [(L 1), (L 1)] for large integral L is just SL,nL gL . Thus hn (x) f (x) outside a countable union of sets of measure zero, so also almost everywhere. (N ) (N ) (4) This is repetition of the rst problem, hn,L (x) gL almost everywhere (N ) (N ) (N ) (N ) and |hn,L | N L so gL L1 (R) and |hn,L gL | 0 as n . Problem 5.3 Show that L2 (R) is a Hilbert space since it is rather central to the course I wanted you to go through the details carefully! First working with real functions, dene L2 (R) as the set of functions f : R R which are locally integrable and such that |f |2 is integrable. (1) For such f choose hn and dene gL , gL and hn by (12.24), (12.25) and (12.27). (N ) (N ) (N ) (2) Show using the sequence hn,L for xed N and L that gL and (gL )2 are (N ) (N ) in L1 (R) and that |(hn,L )2 (gL )2 | 0 as n . (N ) (3) Show that gL )2 L1 (R) and that |(gL )2 (gL )2 | 0 as N . ( 2 2 (4) Show that |(gL ) f | 0 as L . (5) Show that f, g L2 (R) then f g L1 (R) and that 2 (12.29) | f g| |f g| f L2 gL2 , f L2 = |f |2 . (6) Use these constructions to show that L2 (R) is a linear space. (7) Conclude that the quotient space L2 (R) = L2 (R)/N , where N is the space of null functions, is a real Hilbert space. (8) Extend the arguments to the case of complex-valued functions.
Solution:
(N ) (1) Done. I think it should have been hn,L . (2) We already checked that gL L1 (R) and the same argument applies to (N ) (N ) (N ) (gL ), namely (hn,L )2 gL almost everywhere and both are bounded by N 2 L so by dominated convergence (hn,L )2 gL )2 N 2 L a.e. = gL )2 L1 (R) and (12.30) |hn,L )2 gL )2 | 0 a.e. , (N ) 2 (N ) 2 (N ) (N ) 2 |hn,L ) gL ) | 2N L = |hn,L )2 gL )2 | 0.
(N ) (N ) (N ) (N ) (N ) (N ) (N ) (N ) (N ) (N )

(3) Now, as N , (gL )2 (gL )2 a.e. and (gL )2 (gL )2 f 2 so by (N ) dominated convergence, (gL )2 L1 and |(gL )2 (gL )2 | 0 as N . 2 (4) The same argument of dominated convergence shows now that gL f 2 2 2 2 1 and |gL f | 0 using the bound by f L (R).

78

LECTURE NOTES FOR 18.102, SPRING 2009

(5) What this is all for is to show that f g L1 (R) if f, F = g L2 (R) (for easier notation). Approximate each of them by sequences of step functions (N ) (N ) as above, hn,L for f and Hn,L for g. Then the product sequence is in L1 being a sequence of step functions and (12.31) hn,L (x)Hn,L (x) gL (x)GL (x) almost everywhere and with absolute value bounded by N 2 L . Thus by (N ) (N ) dominated convergence gL GL L1 (R). Now, let N ; this sequence converges almost everywhere to gL (x)GL (x) and we have the bound (12.32) 1 (N ) (N ) |gL (x)GL (x)| |f (x)F (x)| (f 2 + F 2 ) 2
(N ) (N ) (N ) (N )

so as always by dominated convergence, the limit gL GL L1 . Finally, letting L the same argument shows that f F L1 (R). Moreover, |f F | L1 (R) and (12.33) | f F | |f F | f L2 F L2 where the last inequality follows from Cauchys inequality if you wish, rst for the approximating sequences and then taking limits. (6) So if f, g L2 (R) are real-value, f + g is certainly locally integrable and (12.34) (f + g)2 = f 2 + 2f g + g 2 L1 (R)

by the discussion above. For constants f L2 (R) implies cf L2 (R) is directly true. (7) The argument is the same as for L1 versus L1 . Namely f 2 = 0 implies e that f 2 = 0 almost everywhere which is equivalent to f = 0 a@ . Then the norm is the same for all f + h where h is a null function since f h and h2 are null so (f + h)2 = f 2 + 2f h + h2 . The same is true for the inner product so it follows that the quotient by null functions (12.35) L2 (R) = L2 (R)/N is a preHilbert space. However, it remains to show completeness. Suppose {[fn ]} is an ab solutely summable series in L2 (R) which means that fn L2 < . It
n

follows that the cut-o series fn L is absolutely summable in the L1 sense since 1 2 1 2( (12.36) |fn L | L fn ) 2 by Cauchys inequality. Thus if we set Fn = almost everywhere for each L so in fact (12.37) Fn (x) f (x) converges almost everywhere. We want to show that f L2 (R) where it follows already that f is locally integrable by the completeness of L1 . Now consider the series (12.38)
2 2 2 g1 = F1 , gn = Fn Fn1 . n k1

fk then Fn (x)L converges

LECTURE NOTES FOR 18.102, SPRING 2009

79

The elements are in L1 (R) and by Cauchys inequality for n > 1, (12.39) 2 |gn | = |Fn Fn1 |2 Fn Fn1 L2 Fn + Fn1 L2 fn L2 2 fk L2
k

where the triangle inequality has been used. Thus in fact the series gn is absolutely summable in L1 (12.40) |gn | 2( fn L2 )2 .
n n 2 So indeed the sequence of partial sums, the Fn converge to f 2 L1 (R). 2 Thus f L (R) and moroever 2 2 2 (12.41) (Fn f ) = Fn + f 2 Fn f 0 as n .

Indeed the rst term converges to f 2 and, by Cauchys inequality, the series of products fn f is absulutely summable in L1 with limit f 2 so the third term converges to 2 f 2 . Thus in fact [Fn ] [f ] in L2 (R) and we have proved completeness. (8) For the complex case we need to check linearity, assuming f is locally integrable and |f |2 L1 (R). The real part of f is locally integrable and the (N ) (N ) approximation FL discussed above is square integrable with (FL )2 2 |f | so by dominated convergence, letting rst N and then L the real part is in L2 (R). Now linearity and completeness follow from the real case. Problem 5.4 Consider the sequence space (12.42) h2,1 = c : N j cj C; (1 + j 2 )|cj |2 < .
j

(1) Show that (12.43) h2,1 h2,1 (c, d) c, d =


j 2,1

(1 + j 2 )cj dj

is an Hermitian inner form which turns h into a Hilbert space. (2) Denoting the norm on this space by 2,1 and the norm on l2 by 2 , show that (12.44) Solution: (1) The inner product is well dened since the series dening it converges ab solutely by Cauchys inequality: 1 1 c, d = (1 + j 2 ) 2 cj (1 + j 2 ) 2 dj , (12.45)
j

h2,1 l2 , c2 c2,1 c h2,1 .

|(1 + j ) cj (1 + j 2 ) dj | (

1 2

1 2

1 1 (1 + j 2 )|cj |2 ) 2 ( (1 + j 2 )|dj |2 ) 2 .
j j

80

LECTURE NOTES FOR 18.102, SPRING 2009

It is sesquilinear and positive denite since 1 (12.46) c2,1 = ( (1 + j 2 )|cj |2 ) 2


j

only vanishes if all cj vanish. Completeness follows as for l2 if c(n) is a 1 (n) (n) Cauchy sequence then each component cj converges, since (1 + j) 2 cj is Cauchy. The limits cj dene an element of h2,1 since the sequence is bounded and N N (n) 2 1 2 |c |2 = lim (12.47) (1 + j ) j (1 + j 2 )|cj |2 A
j=1 n j=1

where A is a bound on the norms. Then from the Cauchy condition c(n) c in h2,1 by passing to the limit as m in c(n) c(m) 2,1 . (2) Clearly h2,2 l2 since for any nite N (12.48)
N j=1

|cj |2

N (1 + j)2 |cj |2 c2 2,1 j=1

and we may pass to the limit as N to see that (12.49) cl2 c2,1 . Problem 5.5 In the separable case, prove Riesz Representation Theorem directly.
Choose an orthonormal basis {ei } of the separable Hilbert space H. Suppose
T : H C is a bounded linear functional. Dene a sequence (12.50) wi = T (ei ), i N. (1) Now, recall that |T u| CuH for some constant C. Show that for every nite N, (12.51)
2 N j=1

|wi |2 C 2 .

(2) Conclude that {wi } l and that (12.52) w= wi ei H.


i

(3) Show that (12.53) Solution: (1) The nite sum wN = wN 2 = N (12.54)
N i=1 N i=1

T (u) = u, wH u H and T = wH .

wi ei is an element of the Hilbert space with norm

|wi |2 by Bessels identity. Expanding out

N n N T (wN ) = T ( w i ei ) = wi T (ei ) = |wi |2 i=1 i=1 i=1

and from the continuity of T, (12.55) |T (wN )| CwN H = wN 2 CwN H = wN 2 C 2 H

LECTURE NOTES FOR 18.102, SPRING 2009

81

which is the desired inequality. (2) Letting N it follows that the innite sum converges and (12.56) |wi |2 C 2 = w = wi ei H
i i

since wN w

|wi | tends to zero with N. u, ei ei by the completness of the {ei } so from the

(3) For any u H uN = continuity of T

j>N N

i=1

(12.57) T (u) = lim T (uN ) = lim


N

N i=1

u, ei T (ei )
N u, wi ei = lim u, wN = u, w i=1 N

= lim

where the continuity of the inner product has been used. From this and Cauchys inequality it follows that T = supuH =1 |T (u)| w. The converse follows from the fact that T (w) = w2 . H

MIT OpenCourseWare http://ocw.mit.edu

18.102 Introduction to Functional Analysis


Spring 2009

For information about citing these materials or our Terms of Use, visit: http://ocw.mit.edu/terms.

82

LECTURE NOTES FOR 18.102, SPRING 2009

Lecture 13. Thursday, Mar 19: Baires theorem Note from Lecture 9, modied and considerably extended. Theorem 8 (Baire). If M is a non-empty complete metric space and Cn M, n N, are closed subsets such that
(13.1) M= Cn
n

then at least one of the Cn s has an interior point. Proof. So, choose p1 C1 , which must exist since otherwise C1 contains an open / ball. Since C1 is closed there exists 1 > 0 such that B(p1 , 1 )C1 = . Next choose p2 B(p1 , 1 /3) which is not in C2 , which is possible since otherwise B(p1 , 1 /3) C2 , and 2 > 0, 2 < 1 /3 such that B(p2 , 2 )C2 = . So we have used both the fact that C2 has empty interior and the fact that it is closed. Now, proceed, inductively. Assume that there is a nite sequence pi , i = 1, . . . , k and positive numbers 0 < k < k1 /3 < k2 /32 < < 1 /3k1 < 3k such that pj B(pj1 , j1 /3) and B(pj , j ) Cj = . Then we can add another pk+1 by using the properties of Ck it has non-empty interior so there is some point in B(pk , k /3) which is not in Ck+1 and then B(pk+1 , k+1 ) Ck+1 = where k+1 > 0 but k+1 < k /3. Thus, we have constructe and innite sequence {pk } in M. Since d(pk+1 , pk ) < k /3 this is a Cauchy sequence. In fact (13.2) d(pk , pk+l ) < k /3 + + k+l1 /3 < 3k < 2k /3

for all l > 0, and this tends to zero as k . Since M is complete this sequence converges. From (13.2) the limit, q M / must lie in the closure of B(pk , 2k /3) for every k. Hence q Ck for any k which contradicts (13.1). Thus, at least one of the Cn must have non-empty interior. One application of this is often called the uniform boundedness principle, I will just call it: Theorem 9 (Uniform boundedness). Let B be a Banach space and suppose that Tn is a sequence of bounded (i.e. continuous) linear operators Tn : B V where V is a normed space. Suppose that for each b B the set {Tn (b)} V is bounded (in norm of course) then supn Tn < . Proof. This follows from a pretty direct application of Baires theorem to B. Con sider the sets (13.3) Sp = {b B, b 1, Tn bV p n}, p N.

Each Sp is closed because Tn is continuous, so if bk b is a convergent sequence then b 1 and Tn (p) p. The union if the Sp is the whole of the closed ball of radius one around the origin in B :
(13.4) {b B; d(b, 0) 1} = Sp
p

because of the assumption of pointwise boundedness each b with b 1 must be in one of the Sp s.

LECTURE NOTES FOR 18.102, SPRING 2009

83

So, by Baires theorem one of the sets Sp has non-empty interior. This means that for some p, some v Sp , and some > 0, (13.5) w B, wB = Tn (v + w)V p n. Moving v to (1 /2)v and halving as necessary it follows that this ball B(v, ) is contained in the open ball around the origin of radius 1. Thus, using the triangle inequality, and the fact that Tn (v)V p this implies (13.6) w B, wB = Tn (w)V 2p = Tn 2p/ since the norm of the operator is sup{T wV ; wB = 1 it follows that the norms are uniformly bounded: (13.7) as claimed. Tn 2p/

One immediate consequence of this is that, as I mentioned in last lecture, it is not necessary to assume that a weakly convergent sequence in a Hilbert space is norm bounded. Corollary 2. If un H is a sequence in a Hilbert space and for all v H (13.8) (un , v) F (v) converges in C then un H is bounded and there exists w H such that un w (converges weakly). Proof. Well, a corollary really should not need a proof but still I will give one since maybe it is a bit more than a corollary. Apply the Uniform Boundedness Theorem to the continuous functionals (13.9) Tn (u) = (u, un ), Tn : H C where we reverse the order to make them linear rather than anti-linear. Thus, each set |Tn (u)| is bounded in C since it is convergent. It follows that there is a bound (13.10) Tn C. However, the norm is just Tn = un H so the sequence must be bounded in H. Dene T : H C as the limit for each u : (13.11) T (u) = lim Tn (u) = lim (u, un ).
n n

This exists for each u by hypothesis. It is a linear map an from (13.10) it is bounded, T C. Thus by the Riesz Representation theorem, there exists w H such that (13.12) T (u) = (u, w) u H. Thus (un , u) (w, u) for all u H so un w as claimed. The second major application of Baires theorem is to Theorem 10 (Open Mapping). If T : B1 B2 is a bounded and surjective linear map between two Banach spaces then T is open: (13.13) T (O) B2 is open if O B1 is open. This is wrong way continuity and as such can be used to prove the continuity of inverse maps as we shall see. The proof uses Baires theorem but then another similar sort of argument is needed. I did not nish the second argument in the lecture.

84

LECTURE NOTES FOR 18.102, SPRING 2009

Proof.

(1) The rst part, of the proof, using Baires theorem shows that the closure of the image, so in B2 , of an open ball around the origin in B1 , has 0 as an interior point i.e. it contains an open ball around the origin in B2 . To see this we apply Baires theorem to the sets Cp = clB2 T (B(0, p))

(13.14)

the closure of the image of the ball in B1 or radius p. We need to take the closure since the sets in Baires theorem are closed, but even before doing that we know that (13.15) B2 = T (B(0, p))
p

since that is what surjectivity means every point is the image of some thing. Thus one of the closed sets Cp has an interior point, v. Since T is surjective, v = T u for some u B1 . The sets Cp increase with p so we can take a larger p and v is still an interior point, from which it follows that 0 = v T u is an interior point as well. Thus indeed (13.16) Cp B(0, )

for some > 0. (2) Having applied Baires thereom, consider now what (13.16) means. It fol lows that each v B2 with v < is the limit of a sequence T un where un p. What we want to arrange is that this sequence converges. Note that we can scale the norm of v using the linearity of T. Thus, for a gen eral v B2 we can apply (13.16) to v = v/2v to see that T u v n where u p. Then un = vu / satises T un v, un 2pv /. n n To simpliy the arithmetic, let me replace T by cT where c = p/2. This means that for each v B2 there is a sequence un in B1 with un v and T un v. Now, we can stop before we get to the limit of the sequence and get as close to v as we want. This means that 1 (13.17) For each v B2 , u B1 , u < v, v T u v. 2 This in turn we can iterate to construct a better sequence. Fix w = w1 B1 with w1 < 1 and choose u1 = u according to (13.17) for v = w = w1 . Thus u1 < 1 and w2 = w1 T u1 satises w2 < 1 . Now proceed by 2 induction, supposing that we have constructed a sequence uj , j < n, in B1 with uj 2j+1 and wj1 < 2j+1 where wj = wj1 T uj1 . Then we can choose un to extend the induction and so we get a sequence un such that for each n n n n (13.18) w T ( ) = w 1 T u1 = w2 T u2 = wn+1 .
j=1 j=2 j=3

The series with terms un is absolutely summabel, hence convergent since B1 is complete, and (13.19) w = T u, u = uj , u 2.
j

LECTURE NOTES FOR 18.102, SPRING 2009

85

So nally we have shown that each w B(0, 1) in B2 is in the image of B(0, 2) in B1 . Going back to the unscaled T it follows that for some > 0, (13.20) B(0, ) T (B(0, 1)). (3) It follows of course that the image T (O) of any open set is open, since if w T (O) then w = T u for some u O and hence B(w, ) is contained in the image of u + B(0, ) O for > 0 suciently small. So, as I did not quite nish the proof in lecture. However at the very end I mention the two most important applications of of this Open Mapping Theorem. Namely: Corollary 3. If T : B1 B2 is a bounded linear map between Banach spaces which is 1-1 and onto, i.e. is a bijection, then it is a homeomorphism meaning its inverse, which is necessarily linear, is also bounded. Proof. The only confusing thing is the notation. Note that T 1 is used to denote the inverse maps on sets. So, the inverse of T, lets call it S : B2 B2 is certainly linear. If O B1 is open then S 1 (O) = T (O) is open by the Open Mapping theorem, so S is continuous. The second application is Theorem 11 (Closed Graph). If T : B1 B2 is a linear map between Banach spaces then it is bounded if and only if its graph (13.21) Gr(T ) = {(u, v) B1 B2 ; u2 = T u1 } is a closed subset of the Banach space B1 B2 . Have we actually covered the product of Banach spaces explicitly? If not, think about it for a minute or two! Proof. Suppose rst that T is bounded, i.e. continuous. A sequence (un , vn ) B1 B2 is in Gr(T ) if and only if vn = T un . So, if it converges, then un u and vn = T un T v by the continuity of T, so the limit is in Gr(T ) which is therefore closed. Conversely, suppose the graph is closed. Given the graph we can reconstruct the map it comes from (whether linear or not) from a little diagram. Form B1 B2 consider the two projections, 1 (u, v) = u and 2 (u, v) = v. Both of them are continuous by inspection and we can restrict them to Gr(T ) B1 B2 to get (13.22) Gr(T ) 2 T B2 . B1
1

This little diagram commutes. Indeed there are two ways to map a point (u, v) Gr(T ) to B2 , either directly, sending it to v or rst sending it u B1 and then to T u. Since v = T u these are the same. Now, Gr(T ) B1 B2 is a closed subspace, so it too is a Banach space and 1 and 2 remain continuous when restricted to it. The map 1 is 1-1 and onto, because each u occurs as the rst element of precisely one pair, namely (u, T u) Gr(T ).

86

LECTURE NOTES FOR 18.102, SPRING 2009

Thus the Corollary above applies to 1 to show that its inverse, S is continuous. But then T = 2 S, from the commutativity, is also continuous proving the theorem.

MIT OpenCourseWare http://ocw.mit.edu

18.102 Introduction to Functional Analysis


Spring 2009

For information about citing these materials or our Terms of Use, visit: http://ocw.mit.edu/terms.

LECTURE NOTES FOR 18.102, SPRING 2009

87

Lecture 14. Tuesday, March 31: Fourier series and L2 (0, 2). Fourier series. Let us now try applying our knowledge of Hilbert space to a concrete Hilbert space such as L2 (a, b) for a nite interval (a, b) R. You showed that this is indeed a Hilbert space. One of the reasons for developing Hilbert space techniques originally was precisely the following result. Theorem 12. If u L2 (0, 2) then the Fourier series of u, 1 ikx (14.1) ck e , ck = u(x)eikx dx 2 (0,2)
kZ

converges in L (0, 2) to u. Notice that this does not say the series converges pointwise, or pointwise almost everywhere since this need not be true depending on u. We are just claiming that 1 (14.2) lim |u(x) ck eikx |2 = 0 n 2
|k|n

for any u L (0, 2). First lets see that our abstract Hilbert space theory has put us quite close to proving this. First observe that if e (x) = exp(ikx) then these elements of L2 (0, 2) k satisfy 2 0 if k = j (14.3) ek ej = exp(i(k j)x) = 2 if k = j. 0 Thus the functions (14.4) ek = e 1 k = eikx e 2 k

form an orthonormal set in L2 (0, 2). It follows that (14.1) is just the Fourier-Bessel series for u with respect to this orthonormal set: 1 (14.5) ck = 2u, ek = ck eikx = u, ek ek . 2 So, we alreay know that this series converges in L2 (0, 2) thanks to Bessels identity. So all we need to show is Proposition 21. The ek , k Z, form an orthonormal basis of L2 (0, 2), i.e. are complete: (14.6) ueikx = 0 k = u = 0 in L2 (0, 2). This however, is not so trivial to prove. An equivalent statement is that the nite e linear span of the ek is dense in L2 (0, 2). I will prove this using Fejrs method. In this approach, we check that any continuous function on [0, 2] satisfying the additional condition that u(0) = u(2) is the uniform limit on [0, 2] of a sequence in the nite span of the ek . Since uniform convergence of continuous functions certainly implies convergence in L2 (0, 2) and we already know that the continuous functions which vanish near 0 and 2 are dense in L2 (0, 2) (I will recall why later) this is enough to prove Proposition 21. However the proof is a serious piece of analysis, at least it is to me!

88

LECTURE NOTES FOR 18.102, SPRING 2009

So, the problem is to nd the sequence in the span of the ek . Of course the trick is to use the Fourier expansion that we want to check. The idea of Ces`ro is to a make this Fourier expansion converge faster, or maybe better. For the moment we can work with a general function u L2 (0, 2) or think of it as continuous if you prefer. So the truncated Fourier series is 1 (14.7) Un (x) = ( u(t)eikt dt)eikx 2 (0,2)
|k|n

where I have just inserted the denition of the ck s into the sum. This is just a nite sum so we can treat x as a parameter and use the linearity of the integral to write this as 1 iks (14.8) Un (x) = Dn (x t)u(t), Dn (s) = e . 2 (0,2)
|k|n

Now this sum can be written as an explicit quotient, since, by telescoping, (14.9) (2)Dn (s)(eis/2 eis/2 ) = ei(n+ 2 )s ei(n+ 2 )s .
1 1 1 1

So in fact, at least where s = 0, (14.10) Dn (s) = ei(n+ 2 )s ei(n+ 2 )s 2(eis/2 eis/2 )

and of course the limit as s 0 exists just ne. As I said, Ces`ros idea is to speed up the convergence by replacing Un by its a average (14.11) Vn (x) = 1 Ul . n+1
l=0 n

Again plugging in the denitions of the Ul s and using the linearity of the integral we see that n 1 (14.12) Vn (x) = Sn (x t)u(t), Sn (s) = Dl (s). n+1 (0,2)
l=0

So again we want to compute a more useful form for Sn (s) which is the Fejr e kernel. Since the denominators in (14.10) are all the same, (14.13) 2(n + 1)(eis/2 eis/2 )Sn (s) =
n l=0

ei(n+ 2 )s

n l=0

ei(n+ 2 )s .

Using the same trick again, (14.14) so (14.15) 2(n + 1)(eis/2 eis/2 )2 Sn (s) = ei(n+1)s + ei(n+1)s 2 = Sn (s) =
(n+1) 1 sin2 ( 2 s) . s n + 1 2 sin2 ( 2 )

(eis/2 eis/2 )

n l=0

ei(n+ 2 )s = ei(n+1)s 1

LECTURE NOTES FOR 18.102, SPRING 2009

89

Now, what can we say about this function? One thing we know immediately is that if we plug u = 1 into the disucssion above, we get Un = 1 for n 0 and hence Vn = 1 as well. Thus in fact (14.16) Sn (x ) = 1.
(0,2)

Now looking directly at (14.15) the rst thing to notice is that Sn (s) 0. Also, we can see that the denominator only vanishes when s = 0 or s = 2 in [0, 2]. Thus if we stay away from there, say s (, 2 ) for some > 0 then since sin is a bounded function (14.17) |Sn (s)| (n + 1)1 C on (, 2 ).

Now, we are interested in how close Vn (x) is to the given u(x) in supremum norm, where now we will take u to be continuous. Because of (14.16) we can write (14.18) u(x) = Sn (x t)u(x)
(0,2)

where t denotes the variable of integration (and x is xed in [0, 2]). This trick means that the dierence is (14.19) Vn (x) u(x) = Sx (x t)(u(t) u(x)).
(0,2)

For each x we split this integral into two parts, the set (x) where x t [0, ] or x t [2 , 2] and the remainder. So (14.20) |Vn (x) u(x)| Sx (x t)|u(t) u(x)| + Sx (x t)|u(t) u(x)|.
(x) (0,2)\(x)

Now on (x) either |t x| the points are close together or t is close to 0 and x to 2 so 2 x + t or conversely, x is close to 0 and t to 2 so 2 t + x . In any case, by assuming that u(0) = u(2) and using the uniform continuity of a continuous function on [0, 2], given > 0 we can choose so small that (14.21) |u(x) u(t)| /2 on (x).

On the complement of (x) we have (14.17) and since u is bounded we get the estimate (14.22) |Vn (x)u(x)| /2 Sn (xt)+(n+1)1 C () /2+(n+1)1 C ().
(x)

Here the fact that Sn is non-negative and has integral one has been used again to estimate the integral of Sn (x t) over (x) by 1. Thus, having chosen to make the rst term small, we can choose n large to make the second term small and it follows that (14.23) Vn (x) u(x) uniformly on [0, 2] as n

under the assumption that u C([0, 2]) satises u(0) = u(2). So this proves Proposition 21 subject to the density in L2 (0, 2) of the continuous functions which vanish near (but not of course in a xed neighbourhood) of the ends.

90

LECTURE NOTES FOR 18.102, SPRING 2009

In fact we know that the L2 functions which vanish near the ends are dense since we can chop of and use the fact that 2 2 (14.24) lim |f | + |f | = 0.
0 (0,) (2,2)

The L2 functions which vanish near the ends are in the closure of the span of the step functions which vanish near the ends. Each such step function can be approximated in L2 ((0, 2)) by a continuous function which vanishes near the ends so we are done as far as density is concerned. So we have proved Theorem 12. Problem set 7, Due 11AM Tuesday 7 Apr. I will put up some practice problems for the test next Thursday when I get a chance. Problem 7.1 Question:- Is it possible to show the completeness of the Fourier basis exp(ikx)/ 2 by computation? Maybe, see what you think. These questions are also intended to get you to say things clearly. (1) Work out the Fourier coecients ck (t) = (0,2) ft eikx of the step function 1 0x<t (14.25) ft (x) = 0 t x 2 for each xed t (0, 2). (2) Explain why this Fourier series converges to ft in L2 (0, 2) if and only if (14.26) 2 |ck (t)|2 = 2t t2 , t (0, 2).
k>0

(3) Write this condition out as a Fourier series and apply the argument again to show that the completeness of the Fourier basis implies identities for the sum of k 2 and k 4 . (4) Can you explain how reversing the argument, that knowledge of the sums of these two series should imply the completeness of the Fourier basis? There is a serious subtlety in this argument, and you get full marks for spotting it, without going ahead a using it to prove completeness. Problem 7.2 Prove that for appropriate constants dk , the functions dk sin(kx/2), k N, form an orthonormal basis for L2 (0, 2). Hint: The usual method is to use the basic result from class plus translation and rescaling to show that d exp(ikx/2) k Z form an orthonormal basis of k L2 (2, 2). Then extend functions as odd from (0, 2) to (2, 2). Problem 7.3 Let ek , k N, be an orthonormal basis in a separable Hilbert space, H. Show that there is a uniquely dened bounded linear operator S : H H, satisfying (14.27) Sej = ej+1 j N. Show that if B : H H is a bounded linear operator then S +B is not invertible if < 0 for some 0 > 0. Hint:- Consider the linear functional L : H C, Lu = (Bu, e1 ). Show that B u = Bu (Lu)e1 is a bounded linear operator from H to the Hilbert space

LECTURE NOTES FOR 18.102, SPRING 2009

91

H1 = {u H; (u, e1 ) = 0}. Conclude that S + B is invertible as a linear map from H to H1 for small . Use this to argue that S + B cannot be an isomorphism from H to H by showing that either e1 is not in the range or else there is a non-trivial element in the null space. Problem 7.4 Show that the product of bounded operators on a Hilbert space is strong continuous, in the sense that if An and Bn are strong convergent sequences of bounded operators on H with limits A and B then the product An Bn is strongly convergent with limit AB. Hint: Be careful! Use the result in class which was deduced from the Uniform Boundedness Theorem.

92

LECTURE NOTES FOR 18.102, SPRING 2009

Solutions to Problems 6 Hint: Dont pay too much attention to my hints, sometimes they are a little othe-cu and may not be very helpfult. An example being the old hint for Problem 6.2! Problem 6.1 Let H be a separable Hilbert space. Show that K H is compact if and only if it is closed, bounded and has the property that any sequence in K which is weakly convergent sequence in H is (strongly) convergent. Hint:- In one direction use the result from class that any bounded sequence has a weakly convergent subsequence. Problem 6.2 Show that, in a separable Hilbert space, a weakly convergent se quence {vn }, is (strongly) convergent if and only if the weak limit, v satises (14.28) vH = lim vn H .
n

Hint:- To show that this condition is sucient, expand (14.29) (vn v, vn v) = vn 2 2 Re(vn , v) + v2 . Problem 6.3 Show that a subset of a separable Hilbert space is compact if and only if it is closed and bounded and has the property of nite dimensional approxi mation meaning that for any > 0 there exists a linear subspace DN H of nite dimension such that (14.30) d(K, DN ) = sup inf {d(u, v)} .
uK vDN

Hint:- To prove necessity of this condition use the equi-small tails property of compact sets with respect to an orthonormal basis. To use the nite dimensional approximation condition to show that any weakly convergent sequence in K is strongly convergent, use the convexity result from class to dene the sequence {vn } in DN where vn is the closest point in DN to vn . Show that vn is weakly, hence strongly, convergent and hence deduce that {vn } is Cauchy. Problem 6.4 Suppose that A : H H is a bounded linear operator with the property that A(H) H is nite dimensional. Show that if vn is weakly convergent in H then Avn is strongly convergent in H. Problem 6.5 Suppose that H1 and H2 are two dierent Hilbert spaces and A : H1 H2 is a bounded linear operator. Show that there is a unique bounded linear operator (the adjoint) A : H2 H1 with the property (14.31) Au1 , u2 H2 = u1 , A u2 H1 u1 H1 , u2 H2 .

MIT OpenCourseWare http://ocw.mit.edu

18.102 Introduction to Functional Analysis


Spring 2009

For information about citing these materials or our Terms of Use, visit: http://ocw.mit.edu/terms.

LECTURE NOTES FOR 18.102, SPRING 2009

93

Lecture 15. Thursday, April 2 I recalled the basic properties of the Banach space, and algebra, of bounded operators B(H) on a separable Hilbert space H. In particular that it is a Banach space with respect to the norm (15.1) and that the norm satises (15.2) AB Ab. A = sup AuH
uH =1

Restatated and went through the proof again of the Theorem 13 (Open Mapping). If A : B1 B2 is a bounded linear operator between Banach spaces and A(B1 ) = B2 , i.e. A is surjective, then it is open: (15.3) A(O) B2 is open O B1 open.

Proof in Lecture 13, also the two consequences of it: If A : B1 B2 is bounded, 1-1 and onto (so it is a bijection) then its inverse is also bounded. Secondly the closed graph theorem. All this is in the notes for Lecture 13. As a second example of the Uniform Boundedness Theorem I also talked about strong convergence of operators. Thus a sequence of bounded operators (on a separable Hilbert space) An B(H) is said to converge strongly if for each u H An u converges. It follows that the limit is a bounded linear operator or you can include this in the denition if you prefer. The Uniform Boundedness Theorem shows that if An is strongly convergent then it is bounded, supn An < . You will need this for the problems this week. I also talked about the shift operator S : l2 l2 dened by (15.4)
S( cj ej ) = cj ej+1 j=1 j=1

dened by moving each element of the sequence up one and starting with zero. This is an example of a bounded linear operator, with S = 1 clearly enough, which is 1-1, since Au = 0 implies u = 0, but which is not surjective. Indeed the range of S is exactly the subspace (15.5) H1 = {u L2 ; (u, e1 ) = 0}.

Using the open mapping theorem (or directly) it is easy to see that S is invertible as a bounded linear map from H to H1 , but not on H. In fact as you should show in the problem set this week, it cannot be made invertible by a small perturbation. This shows in particular that the set of invertible elements of B(H) is not dense, which is quite dierent from the nite dimensional case. Finally I started to talk about the set of invertible elements: (15.6) GL(H) = {A B(H); B H(H), BA = AB = Id}.

Note that this is equivalent to saying A is 1-1 and onto in view of the discussion above. Lemma 10. If A B(H) and A < 1 then (15.7) Id A GL(H).

94

LECTURE NOTES FOR 18.102, SPRING 2009

Proof. Neumann series. If A < 1 then Aj Aj and it follows that the Neumann series (15.8) B= Aj
j

is absolutely summable in B(H) sicne


n j=0

1 j=0

Aj converges. Thus the sum converges.


n+1 j=1

Moreover by the continuity of the product with respect to the norm (15.9) AB = A lim
n

A = lim

Aj = B Id

an similarly BA = B Id . Thus (Id A)B = B(Id A) = Id shows that B is a (and hence the) 2-sided inverse of Id A. Proposition 22. The group of invertible elements GL(H) B(H) is open (but not dense if H is innite-dimensional). Proof. I will do the proof next time.

MIT OpenCourseWare http://ocw.mit.edu

18.102 Introduction to Functional Analysis


Spring 2009

For information about citing these materials or our Terms of Use, visit: http://ocw.mit.edu/terms.

LECTURE NOTES FOR 18.102, SPRING 2009

95

Lecture 16. Tuesday, April 7: partially reconstructed From last time Proposition 23. The invertible elements form an open subset GL(H) B(H). Proof. Recall that we showed using the convergence of the Neumann series that if B B(H) and B < 1 then Id B is invertible, meaning it has a two-sided inverse in B(H) (which we know, from the open mapping Theorem to be equivalent to it being a bijection). So, suppose G GL(H), meaning it has a two-sided (and unique) inverse G1 B(H) : (16.1) G1 G = GG1 = Id .

Then we wish to show that B(G; ) GL(H) for some > 0. In fact we shall see that we can take = G1 1 . The idea is that we wish to show that G + B is a bijection, and hence invertible. To do so set (16.2) E = G1 B = G + B = G1 (Id +G1 B). This is injective if Id +G1 B is injective, and surjective if Id +G1 B is surjective, since G1 is a bijection. From last time we know that (16.3) G1 B < 1 = Id +G1 B is invertible. Since G1 B G1 B this follows if B < G1 1 as anticipated.

Thus GL(H) B(H), the set of invertible elements, is open. It is also a group 1 since the inverse of G1 G2 if G1 , G2 GL(H) is G1 G1 . 2 This group of invertible elements has a smaller subgroup, U(H), the unitary group, dened by (16.4) U(H) = {U GL(H); U 1 = U }.

The unitary group consists of the linear isometric isomorphisms of H onto itself thus (16.5) (U u, U v) = (u, v), U u = u u, v H, U U(H).

This is an important object and we will use it a little bit later on. The unitary group on a separable Hilbert space may seem very similar to the familiar unitary group of n n matrices, U(n). It is, of course it is much bigger for one thing. In fact there are some other important dierences which I will describe a little later on (or get you to do some of it in the problems). On important fact that you should know, even though I will not prove it, is that U(H) is contractible as a metric space it has no signicant topology. This is to be constrasted with the U(n) which have a lot of topology, and not at all simple spaces especially for large n. One upshot of this is that U(H) does not look much like the limit of the U(n) as n . Now, for the rest of today I will talk about the opposite of the big operators such as the elements of GL(H). Denition 7. An operator T B(H) is of nite rank if its range has nite dimension (and that dimension is called the rank of T ); the set of nite rank operators is denoted R(B).

96

LECTURE NOTES FOR 18.102, SPRING 2009

Why not F(B)? Because we want to use this for the Fredholm operators. Clearly the sum of two operators of nite rank has nite rank, since the range is contained in the sum of the ranges (but is often smaller): (16.6) (T1 + T2 u) Ran(T1 ) + Ran(T2 ).

Since the range of a constant multiple of T is contained in the range of T it follows that the nite rank operators form a linear subspace of B(H). It is also clear that (16.7) B B(H) and T R(B) then BT R(B).

Indeed, the range of BT is the range of B restricted to the range of T and this is certainly nite dimensional since it is spanned by the image of a basis of Ran(T ). Similalry T B R(H) since the range of T B is contained in the range of T. Thus we have in fact proved most of Proposition 24. The nite rank operators form a -closed ideal in B(H), which is to say a linear subspace such that (16.8) B1 , B2 B(H), T R(H) = B1 T B2 , T R(H).

Proof. In fact it is only the fact that T is of nite rank if T is which remains to be checked. To do this let us nd an explicit representation for an operator of nite rank. First, since Ran(T ) is nite dimensional, we can choose a basis, fi i = 1, . . . , N, for it. Then for any element u H, (16.9) Tu =
N i=1

ci fi .

The constants ci are determined, since the fi are a basis, and so dene linear functionals u ci . These are continuous. In fact we can simply choose the fi to be orthonormal and then, pairing (16.9) with fj we see that (16.10) cj = (T u, fj ) = (u, T fj ). In particular there are elements (really by Riesz theorem) ei = T fi H sucht that (16.11) Tu =
N (u, ei )fi . i=1

Conversely, if T can be written in the form (16.11) then it is of nite rank, since its range is contained in the span of the fi . From (16.11) it follows that T is also of nite rank since (16.12) (T v, u) = (v, T u) =
N j=1

(v, fi )(ei , u) u H = T v =

N (u, fi )ei . i=1

The rles of the fi and ei are simply interchanged. o

Next time I will show that the closure of the ideal R(H) in B(H) is the ideal of compact operators. Of course this closure is certainly closed(!) Moreover it is a -closed ideal, since Tn K in norm and B1 , B2 B(H) implies (16.13)
B1 Tn B2 B1 KB2 , Tn K .

LECTURE NOTES FOR 18.102, SPRING 2009

97

So, once we prove that the compact operators are the closure of the nite rank operators we will know that they form a closed, -ideal. Notice that the importance of the ideal condition it is the analogue of the normal condition for a subgroup is that the quotient B/I of the algebra by an ideal is again an algebra. The quotient by the ideal, K(H), of compact operators is a Banach space since K is closed. It is called the Calkin algebra. Lemma 11 (Row rank=Colum rank). For any nite rank operator on a Hilbert space, the dimension of the range of T is equal to the dimension of the ranfe of T . Proof. We showed that a nite rank operator T always takes the form (16.11). If the fi are taken to be a basis for the range of T, so N = dim Ran(T ), then the ei must be linearly independent. Indeed, if not then one of the ei can be replaced by a linear combination ei = cj ej . Inserting this into (16.11) shows that
j=i

(16.14)

Tu =

(u, ej )(fj + cj fi )
j=i

from which it follows that the range has dimension at most N 1 which contradicts the choice of the fi . Since the ei are independent it follows from (16.12) that the range of T has dimension N (since the fi are independent) if you like just say dim Ran(T ) N for all nite rank T and then use the fact that (T ) = T to deduce equality.

98

LECTURE NOTES FOR 18.102, SPRING 2009

Problem set 8, Due 11AM Tuesday 14 April. Okay, I forgot to put the problems up. So, here are three problems that should be reasonably quick. If anyone is seriously inconvenienced by the limited time they have to work on them, just let me know and I will give you a couple of days. Problem 8.1 Show that a continuous function K : [0, 1] L2 (0, 2) has the property that the Fourier series of K(x) L2 (0, 2), for x [0, 1], converges uniformly in the sense that if Kn (x) is the sum of the Fourier series over |k | n then Kn : [0, 1] L2 (0, 2) is also continuous and (16.15) sup K(x) Kn (x)L2 (0,2) 0.
x[0,1]

Hint. Use one of the properties of compactness in a Hilbert space that you proved earlier. Problem 8.2 Consider an integral operator acting on L2 (0, 1) with a kernel which is continuous K C([0, 1]2 ). Thus, the operator is (16.16) T u(x) = K(x, y)u(y).
(0,1)

Show that T is bounded on L (I think we did this before) and that it is in the norm closure of the nite rank operators. Hint. Use the previous problem! Show that a continuous function such as K in this Problem denes a continuous map [0, 1] x K(x, ) C([0, 1]) and hence a continuous function K : [0, 1] L2 (0, 1) then apply the previous problem with the interval rescaled. Here is an even more expanded version of the hint: You can think of K(x, y) as a continuous function of x with values in L2 (0, 1). Let Kn (x, y) be the continuous function of x and y given by the previous problem, by truncating the Fourier series (in y) at some point n. Check that this denes a nite rank operator on L2 (0, 1) yes it maps into continuous functions but that is ne, they are Lebesgue square integrable. Now, the idea is the dierence K Kn denes a bounded operator with small norm as n becomes large. It might actually be clearer to do this the other way round, exchanging the roles of x and y. Problem 8.3 Although we have concentrated on the Lebesgue integral in one variable, you proved at some point the covering lemma in dimension 2 and that is pretty much all that was needed to extend the discussion to 2 dimensions. Lets just assume you have assiduously checked everything and so you know that L2 ((0, 2)2 ) is a Hilbert space. Sketch a proof noting anything that you are not sure of that the functions exp(ikx + ily)/2, k, l Z, form a complete orthonormal basis.

LECTURE NOTES FOR 18.102, SPRING 2009

99

Solutions to Problem set 7 Problem 7.1 Question:- Is it possible to show the completeness of the Fourier basis exp(ikx)/ 2 by computation? Maybe, see what you think. These questions are also intended to get you to say things clearly. (1) Work out the Fourier coecients ck (t) = (0,2) ft eikx of the step function 1 0x<t (16.17) ft (x) = 0 t x 2 for each xed t (0, 2). (2) Explain why this Fourier series converges to ft in L2 (0, 2) if and only if (16.18) 2 |ck (t)|2 = 2t t2 , t (0, 2).
k>0

(3) Write this condition out as a Fourier series and apply the argument again to show that the completeness of the Fourier basis implies identities for the sum of k 2 and k 4 . (4) Can you explain how reversing the argument, that knowledge of the sums of these two series should imply the completeness of the Fourier basis? There is a serious subtlety in this argument, and you get full marks for spotting it, without going ahead a using it to prove completeness. Problem 7.2 Prove that for appropriate constants dk , the functions dk sin(kx/2), k N, form an orthonormal basis for L2 (0, 2). Hint: The usual method is to use the basic result from class plus translation and rescaling to show that d exp(ikx/2) k Z form an orthonormal basis of k L2 (2, 2). Then extend functions as odd from (0, 2) to (2, 2). Problem 7.3 Let ek , k N, be an orthonormal basis in a separable Hilbert space, H. Show that there is a uniquely dened bounded linear operator S : H H, satisfying (16.19) Sej = ej+1 j N. Show that if B : H H is a bounded linear operator then S +B is not invertible if < 0 for some 0 > 0. Hint:- Consider the linear functional L : H C, Lu = (Bu, e1 ). Show that B u = Bu (Lu)e1 is a bounded linear operator from H to the Hilbert space H1 = {u H; (u, e1 ) = 0}. Conclude that S + B is invertible as a linear map from H to H1 for small . Use this to argue that S + B cannot be an isomorphism from H to H by showing that either e1 is not in the range or else there is a non-trivial element in the null space. Problem 7.4 Show that the product of bounded operators on a Hilbert space is strong continuous, in the sense that if An and Bn are strong convergent sequences of bounded operators on H with limits A and B then the product An Bn is strongly convergent with limit AB. Hint: Be careful! Use the result in class which was deduced from the Uniform Boundedness Theorem.

MIT OpenCourseWare http://ocw.mit.edu

18.102 Introduction to Functional Analysis


Spring 2009

For information about citing these materials or our Terms of Use, visit: http://ocw.mit.edu/terms.

100

LECTURE NOTES FOR 18.102, SPRING 2009

Lecture 17. Thursday April 9 was the second test (1) Problem 1 Let H be a separable (partly because that is mostly what I have been talking about) Hilbert space with inner product (, ) and norm . Say that a sequence un in H converges weakly if (un , v) is Cauchy in C for each v H. (a) Explain why the sequence un H is bounded. Solution: Each un denes a continuous linear functional on H by (17.1) Tn (v) = (v, un ), Tn = un , Tn : H C. For xed v the sequence Tn (v) is Cauchy, and hence bounded, in C so by the Uniform Boundedness Principle the Tn are bounded, hence un is bounded in R. (b) Show that there exists an element u H such that (un , v) (u, v) for each v H. Solution: Since (v, un ) is Cauchy in C for each xed v H it is convergent. Set (17.2) T v = lim (v, un ) in C.
n

This is a linear map, since (17.3) T (c1 v1 + c2 v2 ) = lim c1 (v1 , un ) + c2 (v2 , u) = c1 T v1 + c2 T v2


n

and is bounded since |T v| Cv, C = supn un . Thus, by Riesz theorem there exists u H such that T v = (v, u). Then, by denition of T, (17.4) (un , v) (u, v) v H.

(c) If ei , i N, is an orthonormal sequence, give, with justication, an example of a sequence un which is not weakly convergent in H but is such that (un , ej ) converges for each j. Solution: One such example is un = nen . Certainly (un , ei ) = 0 for all i > n, so converges to 0. However, un is not bounded, so the sequence cannot be weakly convergent by the rst part above. (d) Show that if the ei form an orthonormal basis, un is bounded and (un , ej ) converges for each j then un converges weakly. Solution: By the assumption that (un , ej ) converges for all j it fol lows that (un , v) converges as n for all v which is a nite lin ear combination of the ei . For general v H the convergence of the Fourier-Bessell series for v with respect to the orthonormal basis ej (17.5) v= (v, ek )ek
k

shows that there is a sequence vk v where each vk is in the nite span of the ej . Now, by Cauchys inequality (17.6) |(un , v) (um , v)| |(un vk ) (um , vk )| + |(un , v vk )| + |(um , v vk )|. Given > 0 the boundedness of un means that the last two terms can be arranged to be each less than /4 by choosing k suciently large. Having chosen k the rst term is less than /4 if n, m > N by

LECTURE NOTES FOR 18.102, SPRING 2009

101

the fact that (un , vk ) converges as n . Thus the sequence (un , v) is Cauchy in C and hence convergent. (2) Problem 2 Suppose that f L1 (0, 2) is such that the constants ck = f (x)eikx , k Z,
(0,2)

satisfy
kZ 2

|ck |2 < .

Show that f L (0, 2). Solution. So, this was a good bit harder than I meant it to be but still in principle solvable (even though no one quite got to the end). First, (for half marks in fact!) we know that the ck exists, since f L1 (0, 2) and eikx is continuous so f eikx L1 (0, 2) and then the con dition |ck |2 < implies that the Fourier series does converge in L2 (0, 2)
k

so there is a function (17.7) g= 1 ck eikx . 2


kC

Now, what we want to show is that f = g a.e. since then f L2 (0, 2). Set h = f g L1 (0, 2) since L2 (0, 2) L1 (0, 2). It follows from (17.7) that f and g have the same Fourier coecients, and hence that (17.8) h(x)eikx = 0 k Z.
(0,2)

So, we need to show that this implies that h = 0 a.e. Now, we can recall from class that we showed (in the proof of the completeness of the Fourier basis of L2 ) that these exponentials are dense, in the supremum norm, in continuous functions which vanish near the ends of the interval. Thus, by continuity of the integral we know that (17.9) hg = 0
(0,2)

for all such continuous functions g. We also showed at some point that we can nd such a sequence of continuous functions gn to approximate the characteristic function of any interval I . It is not true that gn I uniformly, but for any integrable function h, hgn hI in L1 . So, the upshot of this is that we know a bit more than (17.9), namely we know that (17.10) hg = 0 step functions g.
(0,2)

So, now the trick is to show that (17.10) implies that h = 0 almost everywhere. Well, this would follow if we know that (0,2) |h| = 0, so lets aim for that. Here is the trick. Since g L1 we know that there is a sequence (the partial sums of an absolutely convergent series) of step

102

LECTURE NOTES FOR 18.102, SPRING 2009

functions hn such that hn g both in L1 (0, 2) and almost everywhere and also |hn | |h| in both these senses. Now, consider the functions 0 if hn (x) = 0 (17.11) sn (x) = hn (x) |hn (x)| otherwise. Clearly sn is a sequence of step functions, bounded (in absolute value by 1 in fact) and such that sn hn = |hn |. Now, write out the wonderful identity (17.12) |h(x)| = |h(x)| |hn (x)| + sn (x)(hn (x) h(x)) + sn (x)h(x). Integrate this identity and then apply the triangle inequality to conclude that |h| = (|h(x)| |hn (x)| + sn (x)(hn h) (0,2) (0,2) (0,2) (17.13) (||h(x)| |hn (x)|| + |hn h| 0 as n .
(0,2) (0,2)

Here on the rst line we have used (17.10) to see that the third term on the right in (17.12) integrates to zero. Then the fact that |sn | 1 and the convergence properties. Thus in fact h = 0 a.e. so indeed f = g and f L2 (0, 2). Piece of cake, right! Mia culpa. (3) Problem 3 Consider the two spaces of sequences h2 = {c : N C;
j=1

j 4 |cj |2 < }.

Show that both h2 are Hilbert spaces and that any linear functional sat isfying T : h2 C, |T c| Cch2 for some constant C is of the form Tc = ci di
j=1

where d : N C is an element of h2 . Solution: Many of you hammered this out by parallel with l2 . This is ne, but to prove that h2 are Hilbert spaces we can actually use l2 itself. Thus, consider the maps on complex sequences (17.14) (T c)j = cj j 2 . Without knowing anything about h2 this is a bijection between the se quences in h2 and those in l2 which takes the norm (17.15) ch2 = T cl2 . It is also a linear map, so it follows that h are linear, and that they are indeed Hilbert spaces with T isometric isomorphisms onto l2 ; The inner products on h2 are then (17.16) (c, d)h2 =
j=1

j 4 cj dj .

LECTURE NOTES FOR 18.102, SPRING 2009

103

Dont feel bad if you wrote it all out, it is good for you! Now, once we know that h2 is a Hilbert space we can apply Riesz the orem to see that any continuous linear functional T : h2 C, |T c| Cch2 is of the form (17.17) T c = (c, d )h2 = j 4 cj d , d h2 . j
j=1

Now, if d h2 then dj = denes a sequence in h2 . Namely, (17.18) j 4 |dj |2 = j 4 |d |2 < . j


j j

j 4 d j

Inserting this in (17.17) we nd that (17.19) Tc = cj dj , d h2 .


j=1

MIT OpenCourseWare http://ocw.mit.edu

18.102 Introduction to Functional Analysis


Spring 2009

For information about citing these materials or our Terms of Use, visit: http://ocw.mit.edu/terms.

104

LECTURE NOTES FOR 18.102, SPRING 2009

Lecture 18. Tuesday April 14: Compact operators Last time we considered invertible elemenets of B(H), the algebra of bounded operators on a separable Hilbert space, and also the nite rank operators. The latter form an ideal which is closed under taking adjoints. We also showed the the closure of this ideal, the elements in B(H) which are the limits of (norm-convergent) sequences of nite rank operators, also form an ideal which is closed under taking adjoints and also norm, i.e. metrically, closed. Denition 8. An element K B(H), the bounded operators on a separable Hilbert space, is said to be compact (the old terminology was totally bounded and you might still see this) if the image of the unit ball is precompact, i.e. has compact closure that is if the closure of K{u H; uH 1} is compact in H. Lemma 12. An operator K B(H) is compact if and only if the image {Kun } of any weakly convergent sequence {un } in H is strongly, ie. norm, convergent. Proof. First suppose that un u is a weakly convergent sequence in H and that K is compact. We know that un < C is bounded so the sequence Kun is contained in CK(B(0, 1)) and hence in a compact set (clearly if D is compact then so is cD for any constant c.) Thus, any subsequence of Kun has a convergent subseqeunce and the limit is necessarily Ku since Kun Ku (true for any bounded operator by computing (18.1) (Kun , v) = (un , K v) (u, K v) = (Ku, v).)

But the condition on a sequence in a metric space that every subsequence of it has a subsequence which converges to a xed limit implies convergence. (If you dont remember this, reconstruct the proof: To say a sequence vn does not converge to v is to say that for some > 0 there is a subsequence along which d(vnk , v) . This is impossible given the subsequence of subsequence condition (coverging to the xed limit v.) Conversely, suppose that K has this property of turning weakly convergent into strongly convergent sequences. We want to show that K(B(0, 1)) has compact closure. This just means that any sequence in K(B(0, 1)) has a (strongly) con vergent subsequence where we do not have to worry about whether the limit is in the set or not. Such a sequence is of the form Kun where un is a sequence in B(0, 1). However we know that the ball is weakly compact, that is we can pass to a subsequence which converges weakly, unj u. Then, by the assumption of the Lemma, Kunj Ku converges strongly. Thus un does indeed have a convergent subsequence and hence K(B(0, 1)) must have compact closure. Proposition 25. An operator K B(H), bounded on a separable Hilbert space, is compact if and only if it is the limit of a norm-convergent sequence of nite rank operators, i.e. the ideal of compact operators K(H) is the norm closure of the ideal of nite rank operators. Proof. So, we need to show that a compact operators is the limit of a convergent sequence of nite rank operators. To do this we use one of the characterizations of compact subsets of a separable Hilbert space discussed earlier. Namely, if ei is an orthonormal basis of H then a subset I H is compact if and only if it is closed and bounded and has equi-small tails with respec to {ei }, meaning given > 0

LECTURE NOTES FOR 18.102, SPRING 2009

105

there exits N such that (18.2)


i>N

|(v, ei )|2 < 2 v I.

Now we shall apply this to the set K(B(0, 1)) where we assume that K is compact so this set is contained in a compact set. Hence (18.2) applies to it. Namely this means that for any > 0 there exists n such that (18.3) |(Ku, ei )|2 < 2 u H, uH 1.
i>n

For each n consider the rst part of these sequences and dene (18.4) Kn u = (Ku, ei )ei .
kn

This is clearly a linear operator and has nite rank since its range is contained in the span of the rst n elements of {ei }. Since this is an orthonormal basis, (18.5) Ku Kn u2 = |(Ku, ei )|2 H
i>n

Thus (18.3) shows that Ku Kn uH . Now, increasing n makes Ku Kn u smaller, so given > 0 there exits n such that for all N n, (18.6) K KN B = sup Ku Kn uH .
u1

Thus indeed, Kn K in norm and we have shown that the compact operators are contained in the norm closure of the nite rank operators. For the converse we assume that Tn K is a norm convergent sequence in B(H) where each of the Tn is of nite rank of course we know nothing about the rank except that it is nite. We want to conclude that K is compact, so we need to show that K(B(0, 1)) is precompact. It is certainly bounded, by the norm of K. By one of the results on compactness of sets in a separable Hilbert space we know that it suces to prove that every weakly convergent sequence in K(B(0, 1)) has a strongly convergent subsequence meaning norm convergent. The limit need not be in K(B(0, 1)) but must exist of the set is to have compact closure. So, suppose vk is a weakly convergent sequence in K(B(0, 1)). Well then, it is of the form Kuk where uk is a sequence in the unit ball. Of necessity this has a weakly convergent subsequence, so we can assume that uk u is weakly convergent, by passing to a subsequence of the original sequence. Now, each Tn is of nite rank so the sequences Tn vk are each strongly convergent as k namely they are weakly convergent because (Tn vk , w) = (vk , Tn w), and in a nite dimensional space. Use the triangle inequality and denition of the norm of an operator to see that (18.7) Kvk Kvl Kvk Tn vk + Tn vk Tn vl + Tn vl Kvl 2K Tn B + Tn vk Tn vl . Now, given > 0 rst choose n so large that K Tn < /3. Then, having xed n, use the fact that Tn vk is Cauchy to choose p such that k, l > p implies Tn vk Tn vl < /3. It follows that Kvk is Cauchy and hence convergent by the completeness of Hilbert space. Thus K is compact.

106

LECTURE NOTES FOR 18.102, SPRING 2009

Notice that this shows that the ideal of compact operators is itself closed you can see this from the last argument but of course it follows from the fact that it is the closure of the nite rank operators.

LECTURE NOTES FOR 18.102, SPRING 2009

107

Solutions to Problem set 8 Problem 8.1 Show that a continuous function K : [0, 1] L2 (0, 2) has the property that the Fourier series of K(x) L2 (0, 2), for x [0, 1], converges uniformly in the sense that if Kn (x) is the sum of the Fourier series over |k | n then Kn : [0, 1] L2 (0, 2) is also continuous and (18.8) sup K(x) Kn (x)L2 (0,2) 0.
x[0,1]

Hint. Use one of the properties of compactness in a Hilbert space that you proved earlier. Problem 8.2 Consider an integral operator acting on L2 (0, 1) with a kernel which is continuous K C([0, 1]2 ). Thus, the operator is (18.9) T u(x) = K(x, y)u(y).
(0,1)

Show that T is bounded on L (I think we did this before) and that it is in the norm closure of the nite rank operators. Hint. Use the previous problem! Show that a continuous function such as K in this Problem denes a continuous map [0, 1] x K(x, ) C([0, 1]) and hence a continuous function K : [0, 1] L2 (0, 1) then apply the previous problem with the interval rescaled. Here is an even more expanded version of the hint: You can think of K(x, y) as a continuous function of x with values in L2 (0, 1). Let Kn (x, y) be the continuous function of x and y given by the previous problem, by truncating the Fourier series (in y) at some point n. Check that this denes a nite rank operator on L2 (0, 1) yes it maps into continuous functions but that is ne, they are Lebesgue square integrable. Now, the idea is the dierence K Kn denes a bounded operator with small norm as n becomes large. It might actually be clearer to do this the other way round, exchanging the roles of x and y. Problem 8.3 Although we have concentrated on the Lebesgue integral in one variable, you proved at some point the covering lemma in dimension 2 and that is pretty much all that was needed to extend the discussion to 2 dimensions. Lets just assume you have assiduously checked everything and so you know that L2 ((0, 2)2 ) is a Hilbert space. Sketch a proof noting anything that you are not sure of that the functions exp(ikx + ily)/2, k, l Z, form a complete orthonormal basis.

MIT OpenCourseWare http://ocw.mit.edu

18.102 Introduction to Functional Analysis


Spring 2009

For information about citing these materials or our Terms of Use, visit: http://ocw.mit.edu/terms.

108

LECTURE NOTES FOR 18.102, SPRING 2009

Lecture 19. Thursday, April 16 I am heading towards the spectral theory of self-adjoint compact operators. This is rather similar to the spectral theory of self-adjoint matrices and has many useful applications. There is a very eective spectral theory of general bounded but selfadjoint operators but I do not expect to have time to do this. There is also a pretty satisfactory spectral theory of non-selfadjoint compact operators, which it is more likely I will get to. There is no satisfactory spectral theory for general non-compact and non-self-adjoint operators as you can easily see from examples (such as the shift operator). In some sense compact operators are small and rather like nite rank operators. If you accept this, then you will want to say that an operator such as (19.1) Id K, K K(H)

is big. We are quite interested in this operator because of spectral theory. To say that C is an eigenvalue of K is to say that there is a non-trivial solution of (19.2) Ku u = 0 where non-trivial means other than than the solution u = 0 which always exists. If = 0 we can divide by and we are looking for solutions of (19.3) (Id 1 K)u = 0 which is just (19.1) for another compact operator, namely 1 K. What are properties of Id K which migh show it to be big? Here are three: Proposition 26. If K K(H) is a compact operator on a separable Hilbert space then null(Id K) = {u H; (IdK )u = 0} is nite dimensional (19.4) and moreover (19.5) dim (null(Id K)) = dim Ran(Id K) . Ran(Id K) = {v H; u H, v = (Id K)u} is closed and Ran(Id K) = {w H; (w, Ku) = 0 u H} is nite dimensional

Denition 9. A bounded operator F B(H) on a Hilbert space is said to be Fredholm if it has the three properties in (19.4) its null space is nite dimensional, its range is closed and the orthocomplement of its range is nite dimensional. For general Fredholm operators the row-rank=colum-rank result (19.5) does not hold. Indeed the dierence of these two integers (19.6) ind(F ) = dim (null(Id K)) dim Ran(Id K) is a very important number with lots of interesting properties and uses. Notice that the last two conditions are really independent since the orthocom plement of a subspace is the same as the orthocomplement of its closure. There are for instance bounded opertors on a separable Hilbert space with trivial null space and dense range which is not closed. How could this be? Think for instance of the operator on L2 (0, 1) which is multiplication by the function x. This is assuredly bounded and an element of the null space would have to satisfy xu(x) = 0 almost everywhere, and hence vanish almost everywhere. Moreover the density of the L2

LECTURE NOTES FOR 18.102, SPRING 2009

109

functions vanishing in x < for some (non-xed) > 0 shows that the range is dense. However it is clearly not invertible. Before proving this result lets check that the third condition in (19.4) really follows from the rst. This is a general fact which I mentioned, at least, earlier but let me pause to prove it. Proposition 27. If B B(H) is a bounded operator on a Hilbert space and B is its adjoint then (19.7) Ran(B) = (Ran(B)) = {v H; (v, w) = 0 w Ran(B)} = Nul(B ). Proof. The denition of the orthocomplement of Ran(B) shows immediately that (19.8) v (Ran(B)) (v, w) = 0 w Ran(B) (v, Bu) = 0 u H (B v, u) = 0 u H B v = 0 v Nul(B ). On the other hand we have already observed that V = (B) for any subspace since the right side is certainly contained in the left and (u, v) = 0 for all v V implies that (u, w) = 0 for all w V by using the continuity of the inner product to pass to the limit of a sequence vn w. Thus as a corrollary we see that if Nul(Id K) is always nite dimensional for K compact (i.e. we check it for all compact operators) then Nul(Id K ) is nite dimensional and hence so is Ran(Id K) . Proof of Proposition 26. First lets check this in the case of a nite rank operator K = T. Then (19.9) Nul(Id T ) = {u H; u = T u} Ran(T ). A subspace of a nite dimensional space is certainly nite dimensional, so this proves the rst condition in the nite rank case. Similarly, still assuming that T is nite rank consider the range (19.10) Ran(Id T ) = {v H; v = (Id T )u for some u H}. Consider the subspace {u H; T u = 0}. We know that this this is closed, since T is certainly continuous. On the other hand from (19.10), (19.11) Ran(Id T ) Nul(T ).
N (u, ei )fi i=1

Remember that a nite rank operator can be written out as a nite sum (19.12) Tu =

where we can take the fi to be a basis of the range of T. We also know in this case that the ei must be linearly independent if they werent then we could write one of them, say the last since we can renumber, out as a sum, eN = ci ej , of
j<N

multiples of the others and then nd (19.13) Tu =


N 1 i=1

(u, ei )(fi + cj fN )

showing that the range of T has dimension at most N 1, contradicting the fact that the fi span it.

110

LECTURE NOTES FOR 18.102, SPRING 2009

So, going back to (19.12) we know that Nul(T ) has nite codimension every element of H is of the form (19.14) u = u +
N i=1

di ei , u Nul(T ).

So, going back to (19.11), if Ran(Id T ) = Nul(T ), and it need not be equal, we can choose using the fact that Nul(T ) is closed an element g Ran(Id T ) \ Nul(T ) which is orthogonal to Nul(T ). To do this, start with any a vector g in Ran(Id T ) which is not in Nul(T ). It can be split as g = u + g where g Nul(T ) (being a closed subspace) and u Nul(T ), then g = 0 is in Ran(Id T ) and orthongonal to Nul(T ). Now, the new space Nul(T ) Cg is again closed and contained in Ran(Id T ). But we can continue this process replacing Nul(T ) by this larger closed subspace. After a a nite number of steps we conclude that Ran(Id T ) itself is closed. What we have just proved is: Lemma 13. If V H is a subspace of a Hilbert space which contains a closed subspace of nite codimension in H meaning V W where W is closed and there are nitely many elements ei H, i = 1, . . . , N such that every element u H is of the form (19.15) then V itself is closed. So, this takes care of the case that K = T has nite rank! What about the general case where K is compact? Here we just use a consequence of the approximation of compact operators by nite rank operators proved last time. Namely, if K is compact then there exists B B(H) and T of nite rank such that (19.16) 1 . 2 Now, consider the null space of Id K and use (19.16) to write K = B + T, B < Id K = (Id B) T = (Id B)(Id T ), T = (Id B)1 T. u = u +
N i=1

ci ei , ci C,

(19.17)

Here we have used the convergence of the Neumann series, so (Id B)1 does exist. Now, T is of nite rank, by the ideal property, so (19.18) Nul(Id K) = Nul(Id T ) is nite dimensional. Here of course we use the fact that (Id K)u = 0 is equivalent to (Id T )u = 0 since Id B is invertible. So, this is the rst condition in (19.4). Similarly, to examine the second we do the same thing but the other way around and write (19.19) Id K = (Id B) T = (Id T )(Id B), T = T (Id B)1 . Ran(Id K) = Ran(Id T ) is closed Now, T is again of nite rank and (19.20)

again using the fact that Id B is invertible so every element of the form (Id K)u is of the form (Id T )u where u = (Id B)u and conversely.

LECTURE NOTES FOR 18.102, SPRING 2009

111

So, now we have proved all of (19.4) the third part following from the rst as discussed before. What about (19.5)? This time lets rst check that it is enough to consider the nite rank case. For a compact operator we have written (19.21) where G = Id B with B < want to see is that (19.22) (19.23) (Id K) = G(Id T )
1 2

is invertible and T is of nite rank. So what we

dim Nul(Id K) = dim Nul(Id T ) = dim Nul(Id K ). dim Nul(Id K ) = dim Nul(Id T )

However, Id K = (Id T )G and G is also invertible, so and hence it is enough to check that dim Nul(Id T ) = dim Nul(Id T ) which is to say the same thing for nite rank operators. Now, for a nite rank operator, written out as (19.12), we can look at the vector space W spanned by all the fi s and all the ei s together note that there is nothing to stop there being dependence relations among the combination although separately they are independent. Now, T : W W as is immediately clear and (19.24) T v =
N (v, fi )ei i=1

so T : W W too. In fact T w = 0 and T w = 0 if w W since then (w , ei ) = 0 and (w , fi ) = 0 for all i. It follows that if we write R : W W for the linear map on this nite dimensional space which is equal to Id T acting on it, then R is given by Id T acting on W and we use the Hilbert space structure on W induced as a subspace of H. So, what we have just shown is that (19.25) (Id T )u = 0 u W and Ru = 0, (Id T )u = 0 u W and R u = 0. Thus we really are reduced to the nite-dimensional theorem (19.26) dim Nul(R) = dim Nul(R ) on W. You no doubt know this result. It follows by observing that in this case, every thing now on W, Ran(W ) = Nul(R ) and nite dimensions (19.27) dim Nul(R) + dim Ran(R) = dim W = dim Ran(W ) + dim Nul(R ).

112

LECTURE NOTES FOR 18.102, SPRING 2009

Problem set 9, Due 11AM Tuesday 28 Apr. My apologies for all these errors. Here is a list they are xed below (I hope). (1) In P9.2 (2), and elsewhere, C (S) should be C 0 (S), the space of continuous functions on the circle with supremum norm. (2) In (19.40) it should be u = F v, not u = Sv. (3) Similarly, before (19.41) it should be u = F v. (4) Discussion around (19.43) claried. (5) Last part of P10.2 claried. This week I want you to go through the invertibility theory for the operator d2 + V (x))u(x) dx2 acting on periodic functions. Since we have not developed the theory to handle this directly we need to approach it through integral operators. Before beginning, we need to consider periodic functions. P9.1: Periodic functions Let S be the circle of radius 1 in the complex plane, centered at the origin, S = {z; |z| = 1}. (1) Show that there is a 1-1 correspondence (19.28) Qu = ( (19.29) C 0 (S) = {u : S C, continuous} {u : R C; continuous and satisfying u(x + 2) = u(x) x R}. (2) Show that there is a 1-1 correspondence (19.30) L2 (0, 2) {u L1 (R); u(0,2) L2 (0, 2) loc and u(x + 2) = u(x) x R}/NP where NP is the space of null functions on R satisfying u(x + 2) = u(x) for all x R. (3) If we denote by L2 (S) the space on the left in (19.30) show that there is a dense inclusion (19.31) R. P9.2: Schrdingers operator o Since that is what it is, or at least it is an example thereof: (19.32) d2 u(x) + V (x)u(x) = f (x), x R, dx2 (1) First we will consider the special case V = 1. Why not V = 0? Dont try to answer this until the end! (2) Recall how to solve the dierential equation d2 u(x) + u(x) = f (x), x R, dx2 C 0 (S) L2 (S).

So, the idea is that we can think of functions on S as 2-periodic functions on

(19.33)

where f (x) C 0 (S) is a continuous, 2-periodic function on the line. Show that there is a unique 2-periodic and twice continuously dierentiable

LECTURE NOTES FOR 18.102, SPRING 2009

113

function, u, on R satisfying (19.33) and that this solution can be written in the form (19.34) u(x) = (Sf )(x) = A(x, y)f (y)
0,2

where A(x, y) C (R ) satises A(x + 2, y + 2) = A(x, y) for all (x, y) R. Extended hint: In case you managed to avoid a course on dierential equations! First try to nd a solution, igonoring the periodicity issue. To do so one can (for example, there are other ways) factorize the dierential operator involved, checking that d2 u(x) dv du u + u(x) = ( + v) if v = 2 dx dx dx since the cross terms cancel. Then recall the idea of integrating factors to see that du d u = ex , = ex u, dx dx (19.36) dv d + v = ex , = ex v. dx dx (19.35) Now, solve the problem by integrating twice from the origin (say) and hence get a solution to the dierential equation (19.33). Write this out explicitly as a double integral, and then change the order of integration to write the solution as (19.37) u (x) = A (x, y)f (y)dy
0,2

where A is continuous on R [0, 2]. Compute the dierence u (2) u (0) and du (2) du (0) as integrals involving f. Now, add to u as solution dx dx to the homogeneous equation, for f = 0, namely c1 ex + c2 ex , so that the new solution to (19.33) satises u(2) = u(0) and du (2) = du (0). Now, dx dx check that u is given by an integral of the form (19.34) with A as stated. (3) Check, either directly or indirectly, that A(y, x) = A(x, y) and that A is real. (4) Conclude that the operator S extends by continuity to a bounded operator on L2 (S). (5) Check, probably indirectly rather than directly, that (19.38) S(eikx ) = (k 2 + 1)1 eikx , k Z.

(6) Conclude, either from the previous result or otherwise that S is a compact self-adjoint operator on L2 (S). (7) Show that if g C 0 (S)) then Sg is twice continuously dierentiable. Hint: Proceed directly by dierentiating the integral. (8) From (19.38) conclude that S = F 2 where F is also a compact self-adjoint 1 operator on L2 (S) with eigenvalues (k 2 + 1) 2 . (9) Show that F : L2 (S) C 0 (S). (10) Now, going back to the real equation (19.32), we assume that V is contin uous, real-valued and 2-periodic. Show that if u is a twice-dierentiable

114

LECTURE NOTES FOR 18.102, SPRING 2009

2-periodic function satisfying (19.32) for a given f C 0 (S) then (19.39) u + S((V 1)u) = Sf and hence u = F 2 ((V 1)u) + F 2 f and hence conclude that (19.40) u = F v where v L2 (S) satises v + (F (V 1)F )v = F f

where V 1 is the operator dened by multiplication by V 1. (11) Show the converse, that if v L2 (S) satises (19.41) v + (F (V 1)F )v = F f, f C 0 (S)

then u = F v is 2-periodic and twice-dierentiable on R and satises (19.32). (12) Apply the Spectral theorem to F (V 1)F (including why it applies) and show that there is a sequence j in R \ {0} with |j | 0 such that for all C \ {0}, the equation (19.42) v + (F (V 1)F )v = g, g L2 (S) has a unique solution for every g L2 (S) if and only if = j for any j. (13) Show that for the j the solutions of (19.43) j v + (F (V 1)F )v = 0, v L2 (S),

are all continuous 2-periodic functions on R. (14) Show that the corresponding functions u = F v where v satises (19.43) are all twice continuously dierentiable, 2-periodic functions on R satisfying (19.44) d2 u + (1 sj + sj V (x))u(x) = 0, sj = 1/j . dx2 (15) Conversely, show that if u is a twice continuously dierentiable, 2-periodic function satisfying d2 u + (1 s + sV (x))u(x) = 0, s C, dx2 and u is not identically 0 then s = sj for some j. (16) Finally, conclude that Fredholms alternative holds for the equation (19.32) Theorem 14. For a given real-valued, continuous 2-periodic function V on R, either (19.32) has a unique twice continuously dierentiable, 2 periodic, solution for each f which is continuous and 2-periodic or else there exists a nite, but positive, dimensional space of twice continuously dierentiable 2-periodic solutions to the homogeneous equation (19.46) d2 w(x) + V (x)w(x) = 0, x R, dx2 and (19.32) has a solution if and only if (0,2) f w = 0 for every 2-periodic solution, w, to (19.46).

(19.45)

Not to be handed in, just for the enthusiastic Check that we really can understand all the 2 periodic eigenfunctions of the Schrdinger operator using the discussion above. First of all, there was nothing o sacred about the addition of 1 to d2 /dx2 , we could add any positive number

LECTURE NOTES FOR 18.102, SPRING 2009

115

and get a similar result the problem with 0 is that the constants satisfy the homogeneous equation d2 u/dx2 = 0. What we have shown is that the operator d2 u u+Vu dx2 applied to twice continuously dierentiable functions has at least a left inverse unless there is a non-trivial solution of d2 u (19.48) 2 u + V u = 0. dx Namely, the left inverse is R = F (Id +F (V 1)F )1 F. This is a compact self-adjoint operator. Show and there is still a bit of work to do that (twice continuously dierentiable) eigenfunctions of Q, meaning solutions of Qu = u are precisely the non-trivial solutions of Ru = 1 u. What to do in case (19.48) does have a non-trivial solution? Show that the space of these is nite dimensional and conclude that essentially the same result holds by working on the orthocomplement in L2 (S). (19.47) u Qu =

MIT OpenCourseWare http://ocw.mit.edu

18.102 Introduction to Functional Analysis


Spring 2009

For information about citing these materials or our Terms of Use, visit: http://ocw.mit.edu/terms.

116

LECTURE NOTES FOR 18.102, SPRING 2009

Lecture 20. Thursday, April 23: Spectral theorem for compact self-adjoint operators Let A K(H) be a compact operator on a separable Hilbert space. We know of course, even without assuming that A is compact, that (20.1) Nul(A) H is a closed subspace, so Nul(A) is a Hilbert space although it could be nitedimensional (or even 0-dimensional in the uninteresting case that A = 0). Theorem 15. If A K(H) is a self-adjoint, compact operator on a separable Hilbert space, so A = A, then null(A) has an orthonormal basis consisting of eigenfunctions of A, uj such that (20.2) Auj = j uj , j R \ {0}, arranged so that |j | is a non-increasing sequence satisfying j 0 as j (in case Nul(A) is nite dimensional, this sequence is nite). Before going to the proof, lets notice some useful conclusions. One is called Fredholms alternative. Corollary 4. If A K(H) is a compact self-adjoint operator on a separable Hilbert space then the equation (20.3) u Au = f either has a unique solution for each f H or else there is a non-trivial nite dimensional space of solutions to (20.4) u Au = 0 and then (20.3) has a solution if and only if f is orthogonal to all these solutions. Proof. This is just saying that the null space of Id A is a complement to the range which is closed. So, either Id A is invertible or if not then the range is precisely the orthocomplement of Nul(Id A). You might say there is not much alternative from this point of view, since it just says the range is always the orthocomplement of the null space. Let me separate o the heart of the argument from the bookkeeping. Lemma 14. If A K(H) is a self-adjoint compact operator on a separable (possibly nite-dimensional) Hilbert space then (20.5) F (u) = (Au, u), F : {u H; u = 1} R is a continuous function on the unit sphere which attains its supremum and in mum. Furthermore, if the maximum or minimum is non-zero it is attained at an eivenvector of A with this as eigenvalue. Proof. So, this is just like function in nite dimensions, except that it is not. First observe that F is real-valued, which follows from the self-adjointness of A since (20.6) (Au, u) = (u, Au) = (A u, u) = (Au, u).
Moreover, continuity of F follows from continuity of A and of the inner product so
(20.7) |F (u) F (u )| |(Au, u) (Au, u )| + |(Au, u ) (Au , u )| 2Au u since both u and u have norm one.

LECTURE NOTES FOR 18.102, SPRING 2009

117

If we were in nite dimensions this nishes the proof, since the sphere is then compact and a continuous function on a compact set attains its sup and inf. In the general case we need to use the compactness of A. Certainly F is bounded, (20.8) |F (u)| sup |(Au, u)| A.
u=1

Thus, there is a sequence u+ such that F (u+ ) sup F and another u such that n n n F (u ) inf F. The weak compactness of the unit sphere means that we can pass n to a subsequence in each case, and so assume that u u converges weakly. n Then, by the compactness of A, Au Au converges strongly, i.e. in norm. But n then we can write (20.9) |F (u ) F (u )| |(A(u u ), u )| + |(Au , u u )| n n n n = |(A(u u ), u )| + |(u , A(u u ))| 2Au Au n n n n to deduce that F (u ) = lim F (u ) are respectively the sup and inf of F. Thus n indeed, as in the nite dimensional case, the sup and inf are attained, as in max and min. So, suppose that + = sup F > 0. Then for any v H with v u+ the curve (20.10) Lv : (, ) cos u+ + sin v

lies in the unit sphere. Computing out (20.11) F (Lv ()) = (ALv (), Lv ()) = cos2 F (u+ ) + 2 sin(2) Re(Au+ , v) + sin2 ()F (v) we know that this function must take its maximum at = 0. The derivative there (it is certainly continuously dierentiable on (, )) is Re(Au+ , v) which must therefore vanish. The same is true for iv in place of v so in fact (20.12) (Au+ , v) = 0 v u+ , v = 1. Taking the span of these vs it follows that (Au+ , v) = 0 for all v u+ so A+ u must be a multiple of u+ itself. Inserting this into the denition of F it follows that Au+ = + u+ is an eigenvector with eigenvalue + = sup F. The same argument applies to inf F if it is negative, for instance by replacing A by A. This completes the proof of the Lemma. Proof of Theorem 15. First consider the Hilbert space H0 = Nul(A) H. Then A maps H0 into itself, since (20.13) (Au, v) = (u, Av) = 0 u H0 , v Nul(A) = Au H0 . Moreover, A0 , which is A restricted to H0 , is again a compact self-adjoint operator where the compactness follows from the fact that A(B(0, 1)) for B(0, 1) H0 is smaller than (actually of course equal to) the whole image of the unit ball. Thus we can apply the Lemma above to A0 , with quadratic form F0 , and nd an eigenvector. Lets agree to take the one associated to sup FA0 unless supA0 < inf F0 in which case we take one associated to the inf . Now, what can go wrong here? Nothing except if F0 0. However, Lemma 15. In general for a self-adjoint operator on a Hilbert space (20.14) F 0 A 0.

118

LECTURE NOTES FOR 18.102, SPRING 2009

Proof. In principle F is only dened on the unit ball, but of course we can recover (Au, u) for all u H from it. Namely, if u = 0 it vanishes of course and otherwise u (20.15) (Au, u) = u2 F ( ). u Thenxs we can recover A by polarization. Since (20.16) 2(Au, v) = (A(u + v), u + v) + i(A(u + iv, u + iv). Thus if F 0 then A 0.

So, we know that we can nd an eigenvector unless A 0 which would imply Nul(A) = H. Now we proceed by induction. Suppose we have found N mutually or thogonal eigenvectors ej for A all with norm 1 and eigenvectors j an orthonormal set of eigenvectors and all in H0 . Then we consider (20.17) (20.18) HN = {u H0 = Nul(A) ; (u, ej ) = 0, j = 1, . . . , N }. (Au, ej ) = (u, Aej ) = j (u, ej ) = 0 if u HN = Au HN . From the argument above, A maps HN into itself, since Moreover this restricted operator is self-adjoint and compact on HN as before so we can again nd an eigenvector, with eigenvalue either the max of min of the new F for HN . The only problem arises if F 0 at some stage, but then A 0 on HN and since HN Nul(A) this implies HN = {0} so H0 must have been nite dimensional. Thus, either H0 is nite dimensional or we can grind out an innite orthonormal sequence ei of eigenvectors of A in H0 with the corresponding sequence of eigen values such that |i | is non-increasing since the successive FN s are restrictions of the previous ones the max and min are getting closer to (or at least no further from) 0. In fact it follows that j 0 in this case, since otherwise there must be one eigenvalue = 0 for which the space of eigenvectors is innte dimensional ruled out by the fact that (Id 1 A) has nite dimensional null space as shown last time. Finally then, why must this orthonormal sequence be an orthonormal basis of H0 ? If not, then we can form the closure of the span of the ei we have constructed, H , and its orthocomplement in H0 which would have to be non-trivial. However, as before F restricts to this space to be F for the restriction of A to it, which is again a compact self-adjoint operator. So, if F is not identically zero we can again construct an eigenfunction, with non-zero eigenvalue, which contracdicts the fact the we are always choosing a largest eigenvalue, in absolute value at least. Thus in fact F 0 so A 0 and the eigenvectors form and orthonormal basis of Nul(A) . This completes the proof of the theorem.

MIT OpenCourseWare http://ocw.mit.edu

18.102 Introduction to Functional Analysis


Spring 2009

For information about citing these materials or our Terms of Use, visit: http://ocw.mit.edu/terms.

LECTURE NOTES FOR 18.102, SPRING 2009

119

Lecture 21. Tuesday, April 28: Dirichlet problem on an interval I want to do a couple of serious applications of what we have done so far. There are many to choose from, and I will mention some more, but let me rst consider the Diriclet problem on an interval. I will choose the interval [0, 2] because we looked at it before. So, what we are interested in is the problem of solving d2 u(x) + V (x)u(x) = f (x) on (0, 2), u(0) = u(2) = 0 dx2 where the last part are the Dirichlet boundary conditions. I will assume that (21.1) (21.2) V : [0, 2] R is continuous.

Now, it certainly makes sense to try to solve the equation (21.1) for say a given f C 0 ([0, 2]), looking for a solution which is twice continuously dierentiable on the interval. It may not exist, depending on V but one thing we can shoot for, which has the virtue of being explicit is the following: Proposition 28. If V 0 as in (21.2) then for each f C 0 ([0, 2]) there exists a unique twice continuously dierentiable solution to (21.1). You will see that it is a bit hard to approach this directly especially if you remember some ODE theory from 18.03. There are in fact various approaches to this but we want to go through L2 theory not surprisingly of course. How to start? Well, we do know how to solve (21.1) if V 0 since we can use (Riemann) integration. Thus, ignoring the boundary conditions we can nd a solution to d2 v/dx2 = f on the interval by integrationg twice: x y (21.3) v(x) = f (t)dtdy saties d2 v/dx2 = f on (0, 2).
0 0

Moroever v really is twice continuously dierentiable if f is continuous. So, what has this got to do with operators? Well, we can change the order of integration in (21.3) to write v as x x 2 (21.4) v(x) = f (t)dydt = a(x, t)f (t)dt, a(x, t) = (t x)H(x t)
0 t 0

where the Heaviside function H(y) is 1 when y 0 and 0 when y < 0. Thus a(x, t) is actually continuous on [0, 2] [0, 2] since the t x factor vanishes at the jump in H(t x). Thus v is given by applying an integral operator to f. Before thinking more seriously about this, recall that there are also boundary conditions. Clearly, v(0) = 0 since we integrated from there. However, there is no particular reason why 2 (21.5) v(2) = (t 2)f (t)dt
0

should vanish. However, we can always add to v any linear function and still satify the dierential equation. Since we do not want to spoil the vanishing at x = 0 we can only aord to add cx but if we choose c correctly, namely consider 2 1 (21.6) c= (t 2)f (t)dt, then (v + cx)(2) = 0. 2 0

120

LECTURE NOTES FOR 18.102, SPRING 2009

So, nally the solution we want is 2 t 2 (21.7) w(x) = b(x, t)f (t)dt, b(x, t) = (t x)H(x t) x. 2 0 This is the unique, twice continuously dierentiable, solution of d2 w/dx2 = f in (0, 2) which vanishes at both end points. Lemma 16. The integral operator (21.7) extends by continuity from C 0 ([0, 2]) to a compact, self-adjoint operator on L2 (0, 2). Proof. Since w is given by an integral operator with a continuous real-valued kernel which is even in the sense that (check it) (21.8) b(t, x) = b(x, t)

we might as well give a more general result. Proposition 29. If b C 0 ([0, 2]2 ) then 2 (21.9) Bf (x) = b(x, t)f (t)dt
0

denes a compact operator on L2 (0, 2) if in addition b satises (21.10) then B is self-adjoint. Proof. b(t, x) = b(x, t)

Now, recall from one of the Problem sets that uk = c sin(kx/2), k N, is an orthonormal basis for L2 (0, 2). Moreover, dierentiating we nd straight away that (21.11) d2 uk k2 = uk . 2 4 dx

Since of course uk (0) = 0 = uk (2) as well, from the uniqueness above we conclude that 4 (21.12) Buk = 2 uk k. k Thus, in this case we know the orthonormal basis of eigenfunctions for B. They are the uk , each eigenspace is 1 dimensional and the eigenvalues are 4k 2 . So, this happenstance allows us to decompose B as the square of another operator dened directly on the othornormal basis. Namely (21.13) Auk = 2 uk = B = A2 . k

Here again it is immediate that A is a compact self-adjoint operator on L2 (0, 2) since its eigenvalues tend to 0. In fact we can see quite a lot more than this. Lemma 17. The operator A maps L2 (0, 2) into C 0 ([0, 2]) and Af (0) = Af (2) = 0 for all f L2 (0, 2).

LECTURE NOTES FOR 18.102, SPRING 2009

121

Proof. If f L2 (0, 2) we may expand it in Fourier-Bessel series in terms of the uk and nd (21.14) f= ck uk , (ck ) L2 .
k

Then of course, by denition, (21.15) Af = 2ck


k

uk .

Here each uk is a bounded continuous function, with the bound on uk being in dependent of k. So in fact (21.15) converges uniformly and absolutely since it is uniformly Cauchy, for any q > p, 1 2 q q q 2ck 1 2 (21.16) | uk | 2|c| |ck |k 2|c|f L2 k k
k=p k=p k=p

where Cauchy-Schwartz has been used. This proves that A : L2 (0, 2) C 0 ([0, 2]) is bounded and by the uniform convergence uk (0) = uk (2) = 0 for all k implies that Af (0) = Af (2) = 0. So, going back to our original problem we try to solve (21.1) by moving the V u term to the right side of the equation (dont worry about regularity yet) and hope to use the observation that (21.17) u = A2 (V u) + A2 f

should satisfy the equation and boundary conditions. In fact, lets hope that u = Av, which has to be true if (21.17) holds with v = AV u + Af, and look instead for (21.18) v = AV Av + Af = (Id +AV A)v = Af.

So, we know that multiplication by V, which is real and continuous, is a bounded self-adjoint operator on L2 (0, 2). Thus AV A is a self-adjoint compact operator so we can apply our spectral theory to Id +AV A. It has a complete orthonormal basis and is invertible if and only if it has trivial null space. An element of the null space would have to satisfy u = AV Au. On the other hand we know that AV A is positive since (21.19) (AV Aw, w) = (V Av, Av) = V (x)|Av|2 0 = |u|2 = 0,
(0,2) (0,2)

using the non-negativity of V. So, there can be no null space all the eigenvalues of AV A are at least non-negative so 1 is not amongst them. Thus Id +AV A is invertible on L2 (0, 2) with inverse of the form Id +Q, Q again compact and self-adjoint. So, to solve (21.18) we just need to take (21.20) Then indeed (21.21) u = Av satises u + A2 V u = A2 f. v = (Id +Q)Af v + AV Av = Af in L2 (0, 2).

122

LECTURE NOTES FOR 18.102, SPRING 2009

In fact since v L2 (0, 2) from (21.20) we already know that u C 0 ([0, 2]) vanishes at the end points. Moreover if f C 0 ([0, 2]) we know that Bf = A2 f is twice continuously dier entiable, since it is given by integrations that is where B came from. Now, we know that u is L2 satises u = A2 (V u) + A2 f. Since V u L2 ((0, 2) so is A(V u) and then, as seen above, A(A(V u) is continuous. So combining this with the result about A2 f we see that u itself is continuous and hence so is V u. But then, going through the routine again (21.22) u = A2 (V u) + A2 f is the sum of two twice continuously dierentiable functions. Thus is so itself. In fact from the properties of B = A2 it satisifes d2 u = V u + f dx2 which is what the result claims. So, we have proved the existence part of Proposi tion 28. The uniqueness follows pretty much the same way. If there were two twice continuously dierentiable solutions then the dierence w would satisfy (21.23) d2 w + V w = 0, w(0) = w(2) = 0 = w = Bw = A2 V w. dx2 Thus w = A, = AV w L2 (0, 2). Thus in turn satises = AV A and hence is a solution of (Id +AV A) = 0 which we know has none (assuming V 0). Since = 0, w = 0. This completes the proof of Proposition 28. To summarize what we have shown is that Id +AV A is an invertible bounded operator on L2 (0, 2) (if V 0) and then the solution to (21.1) is precisely (21.24) (21.25) u = A(Id +AV A)1 Af which is twice continuously dierentiable and satises the Dirichlet conditions for each f C 0 ([0, 2]). Now, even if we do not assume that V 0 we pretty much know what is happening. Proposition 30. For any V C 0 ([0, 2]) real-valued, there is an orthonormal basis wk of L2 (0, 2) consisting of twice-continuously dierentiable functions on [0, 2], 2 w vanishing at the end-points and satisfying ddx2k + V wk = Tk wk where Tk as k . The equation (21.1) has a (twice continuously dierentiable) solution for given f C 0 ([0, 2]) if and only if (21.26) TK = 0 = f wk = 0,
(0,2)

i.e. f is orthogonal to the null space of Id +A2 V, which is the image under A of the null space of Id +AV A, in L2 (0, 2).

LECTURE NOTES FOR 18.102, SPRING 2009

123

Problem set 10 (the last), Due 11AM Tuesday 5 May. By now you should have become reasonably comfortable with a separable Hilbert space such as l2 . However, it is worthwhile checking once again that it is rather large if you like, let me try to make you uncomfortable for one last time. An important result in this direction is Kuipers theorem, which I will not ask you to prove1. However, I want you to go through the closely related result sometimes known as Eilenbergs swindle. Perhaps you will appreciate the little bit of trickery. First some preliminary results. Note that everything below is a closed curve in the x [0, 1] variable you might want to identify this with a circle instead, I just did it the primitive way. Problem P10.1 Let H be a separable, innite dimensional Hilbert space. Show that the direct sum of two copies of H is a Hilbert space with the norm (21.27) H H (u1 , u2 ) (u1 2 + u2 2 ) 2 H H T : H H H, 1-1 and onto, uH = T uHH
1

either by constructing an isometric isomorphism (21.28)

or otherwise. In any case, construct a map as in (21.28). Problem P10.2 One can repeat the preceding construction any nite number of times. Show that it can be done countably often in the sense that if H is a separable, innite dimensional, Hilbert space then (21.29) l2 (H) = {u : N H; u22 (H) = ui 2 < } H l
i

has a Hilbert space structure and construct an explicit isometric isomorphism from l2 (H) to H. Problem P10.3 Recall, or perhaps learn about, the winding number of a closed curve with values in C = C \ {0}. We take as given the following fact:2 If Q = [0, 1]N and f : Q C is continuous then for each choice of b C satisfying exp(2ib) = f (0), there exists a unique continuous function F : Q C satisfying (21.30) exp(2iF (q)) = f (q), q Q and F (0) = b. Of course, you are free to change b to b + n for any n Z but then F changes to F + n, just shifting by the same integer. (1) Now, suppose c : [0, 1] C is a closed curve meaning it is continuous and c(1) = c(0). Let C : [0, 1] C be a choice of F for N = 1 and f = c. Show that the winding number of the closed curve c may be dened unambiguously as (21.31) wn(c) = C(1) C(0) Z. (2) Show that wn(c) is constant under homotopy. That is if ci : [0, 1] C , i = 1, 2, are two closed curves so ci (1) = ci (0), i = 1, 2, which are homotopic through closed curves in the sense that there exists f : [0, 1]2 C
1Kuipers theorem says that for any (norm) continuous map, say from any compact metric space, g : M GL(H) with values in the invertible operators on a separable innite-dimensional Hilbert space there exists a continuous map, an homotopy, h : M [0, 1] GL(H) such that h(m, 0) = g(m) and h(m, 1) = IdH for all m M. 2Of course, you are free to give a proof it is not hard.

124

LECTURE NOTES FOR 18.102, SPRING 2009

continuous and such that f (0, x) = c1 (x), f (1, x) = c2 (x) for all x [0, 1] and f (y, 0) = f (y, 1) for all y [0, 1], then wn(c1 ) = wn(c2 ). (3) Consider the closed curve Ln : [0, 1] x e2ix Idnn of n n matrices. Using the standard properties of the determinant, show that this curve is not homotopic to the identity through closed curves in the sense that there does not exist a continuous map G : [0, 1]2 GL(n), with values in the invertible n n matrices, such that G(0, x) = Ln (x), G(1, x) Idnn for all x [0, 1], G(y, 0) = G(y, 1) for all y [0, 1]. Problem P10.4 Consider the closed curve corresponding to Ln above in the case of a separable but now innite dimensional Hilbert space: (21.32) L : [0, 1] x e2ix IdH GL(H) B(H) taking values in the invertible operators on H. Show that after identifying H with H H as above, there is a continuous map (21.33) M : [0, 1]2 GL(H H) with values in the invertible operators and satisfying (21.34) M (0, x) = L(x), M (1, x)(u1 , u2 ) = (e4ix u1 , u2 ), M (y, 0) = M (y, 1), x, y [0, 1]. Hint: So, think of H H as being 2-vectors (u1 , u2 ) with entries in H. This allows one to think of rotation between the two factors. Indeed, show that (21.35) U (y)(u1 , u2 ) = (cos(y/2)u1 + sin(y/2)u2 , sin(y/2)u1 + cos(y/2)u2 ) denes a continuous map [0, 1] y U (y) GL(H H) such that U (0) = Id, U (1)(u1 , u2 ) = (u2 , u1 ). Now, consider the 2-parameter family of maps (21.36) U 1 (y)V2 (x)U (y)V1 (x) where V1 (x) and V2 (x) are dened on H H as multiplication by exp(2ix) on the rst and the second component respectively, leaving the other xed. Problem P10.5 Using a rotation similar to the one in the preceeding problem (or otherwise) show that there is a continuous map (21.37) such that (21.38) G(0, x)(u1 , u2 ) = (e2ix u1 , e2ix u2 ), G(1, x)(u1 , u2 ) = (u1 , u2 ), G(y, 0) = G(y, 1) x, y [0, 1]. Problem P10.6 Now, think about combining the various constructions above in the following way. Show that on l2 (H) there is an homotopy like (21.37), G : 2 [0, 1] GL(l2 (H)), (very like in fact) such that (21.39) G(0, x) {uk }k=1 = exp((1)k 2ix)uk k=1 , G(1, x) = Id, G(y, 0) = G(y, 1) x, y [0, 1]. Problem P10.7: Eilenbergs swindle For an innite dimenisonal separable Hilbert space, construct an homotopy meaning a continuous map G : [0, 1]2 GL(H) with G(0, x) = L(x) in (21.32) and G(1, x) = Id and of course G(y, 0) = G(y, 1) for all x, y [0, 1]. Hint: Just put things together of course you can rescale the interval at the end to make it all happen over [0, 1]. First divide H into 2 copies of itself and deform G : [0, 1]2 GL(H H)

LECTURE NOTES FOR 18.102, SPRING 2009

125

from L to M (1, x) in (21.34). Now, divide the second H up into l2 (H) and apply an argument just like the preceding problem to turn the identity on this factor into alternating terms multiplying by exp(4ix) starting with . Now, you are on H l2 (H), renumbering allows you to regard this as l2 (H) again and when you do so your curve has become alternate multiplication by exp(4ix) (with + rst). Finally then, apply the preceding problem again, to deform to the identity (always of course through closed curves). Presto, Eilenbergs swindle!

126

LECTURE NOTES FOR 18.102, SPRING 2009

Solutions to Problem set 9 P9.1: Periodic functions Let S be the circle of radius 1 in the complex plane, centered at the origin, S = {z; |z| = 1}. (1) Show that there is a 1-1 correspondence (21.40) C 0 (S) = {u : S C, continuous} {u : R C; continuous and satisfying u(x + 2) = u(x) x R}. Solution: The map E : R e2i S is continuous, surjective and 2-periodic and the inverse image of any point of the circle is precisly of the form + 2Z for some R. Thus composition denes a map (21.41) E : C 0 (S) C 0 (R), E f = f E.

This map is a linear bijection. (2) Show that there is a 1-1 correspondence (21.42) L2 (0, 2) {u L1 (R); u(0,2) L2 (0, 2) loc and u(x + 2) = u(x) x R}/NP where NP is the space of null functions on R satisfying u(x + 2) = u(x) for all x R. Solution: Our original denition of L2 (0, 2) is as functions on R which are square-integrable and vanish outside (0, 2). Given such a function u we can dene an element of the right side of (21.42) by assigning a value at 0 and then extending by periodicity (21.43) u(x) = u(x 2n), n Z

where for each x R there is a unique integer n so that x 2n [0, 2). Null functions are mapped to null functions his way and changing the choice of value at 0 changes u by a null function. This gives a map as in (21.42) and restriction to (0, 2) is a 2-sided invese. (3) If we denote by L2 (S) the space on the left in (21.42) show that there is a dense inclusion (21.44) C 0 (S) L2 (S). Solution: Combining the rst map and the inverse of the second gives an inclusion. We know that continuous functions vanishing near the end-points of (0, 2) are dense in L2 (0, 2) so density follows. So, the idea is that we can freely think of functions on S as 2-periodic functions on R and conversely. P9.2: Schrdingers operator o Since that is what it is, or at least it is an example thereof: (21.45) d2 u(x) + V (x)u(x) = f (x), x R, dx2

(1) First we will consider the special case V = 1. Why not V = 0? Dont try to answer this until the end!

LECTURE NOTES FOR 18.102, SPRING 2009

127

Solution: The reason we take V = 1, or at least some other positive constant is that there is 1-d space of periodic solutions to d2 u/dx2 = 0, namely the constants. (2) Recall how to solve the dierential equation (21.46) d2 u(x) + u(x) = f (x), x R, dx2

where f (x) C 0 (S) is a continuous, 2-periodic function on the line. Show that there is a unique 2-periodic and twice continuously dierentiable function, u, on R satisfying (21.46) and that this solution can be written in the form (21.47) u(x) = (Sf )(x) = A(x, y)f (y)
0,2

where A(x, y) C 0 (R2 ) satises A(x + 2, y + 2) = A(x, y) for all (x, y) R. Extended hint: In case you managed to avoid a course on dierential equations! First try to nd a solution, igonoring the periodicity issue. To do so one can (for example, there are other ways) factorize the dierential operator involved, checking that (21.48) d2 u(x) dv du + u(x) = ( + v) if v = u 2 dx dx dx

since the cross terms cancel. Then recall the idea of integrating factors to see that du d u = ex , = ex u, dx dx dv d + v = ex , = ex v. dx dx

(21.49)

Now, solve the problem by integrating twice from the origin (say) and hence get a solution to the dierential equation (21.46). Write this out explicitly as a double integral, and then change the order of integration to write the solution as (21.50) u (x) = A (x, y)f (y)dy
0,2

where A is continuous on R [0, 2]. Compute the dierence u (2) u (0) and du (2) du (0) as integrals involving f. Now, add to u as solution dx dx to the homogeneous equation, for f = 0, namely c1 ex + c2 ex , so that the new solution to (21.46) satises u(2) = u(0) and du (2) = du (0). Now, dx dx check that u is given by an integral of the form (21.47) with A as stated. Solution: Integrating once we nd that if v = du u then dx (21.51) v(x) = e
x

e f (s)ds, u (x) = e

et v(t)dt

128

LECTURE NOTES FOR 18.102, SPRING 2009


2

gives a solution of the equation d u + u (x) = f (x) so combinging these dx2 two and changing the order of integration x 1 yx u (x) = A(x, y)f (y)dy, A(x, y) = e exy 2 0 (21.52) 1 (eyx exy ) x y u (x) = A (x, y)f (y)dy, A (x, y) = 2 0 x y. (0,2) Here A is continuous since A vanishes at x = y where there might other wise be a discontinuity. This is the only solution which vanishes with its derivative at 0. If it is to extend to be periodic we need to add a solution of the homogeneous equation and arrange that (21.53) (21.54) u (2) =
0 2

u = u + u , u = cex + dex , u(0) = u(2), So, computing away we see that 1 y2 du e e2y f (y), (2) = 2 dx
0 2

du du (0) = (2). dx dx

1 y2 e + e2y f (y). 2

Thus there is a unique solution to (21.53) which must satify (21.55) du c(e2 1) + d(e2 1) = u (2), c(e2 1) d(e2 1) = (2) dx 2 2 2y y2 1 1 (e2 1)c = e f (y), (e2 1)d = e f (y). 2 0 2 0 Putting this together we get the solution in the desired form: (21.56) u(x) =
(0.2)

A(x, y)f (y), A(x, y) = A (x, y) + A(x, y) =

1 e2y+x 1 e2+yx = 2 e2 1 2 e2 1

cosh(|x y| ) . e e

(3) Check, either directly or indirectly, that A(y, x) = A(x, y) and that A is real.
Solution: Clear from (21.56).
(4) Conclude that the operator S extends by continuity to a bounded operator on L2 (S).
Solution. We know that S 2 sup |A|.
(5) Check, probably indirectly rather than directly, that (21.57) S(eikx ) = (k 2 + 1)1 eikx , k Z.

Solution. We know that Sf is the unique solution with periodic bound ary conditions and eikx satises the boundary conditions and the equation with f = (k 2 + 1)eikx . (6) Conclude, either from the previous result or otherwise that S is a compact self-adjoint operator on L2 (S). Soluion: Self-adjointness and compactness follows from (21.57) since we know that the eikx / 2 form an orthonormal basis, so the eigenvalues of S

LECTURE NOTES FOR 18.102, SPRING 2009

129

(7)

(8)

(9)

(21.58)

tend to 0. (Myabe better to say it is approximable by nite rank operators by truncating the sum). Show that if g C 0 (S)) then Sg is twice continuously dierentiable. Hint: Proceed directly by dierentiating the integral. Solution: Clearly Sf is continuous. Going back to the formula in terms of u + u we see that both terms are twice continuously dierentiable. From (21.57) conclude that S = F 2 where F is also a compact self-adjoint 1 operator on L2 (S) with eigenvalues (k 2 + 1) 2 . 1 Solution: Dene F (eikx ) = (k 2 + 1) 2 eikx . Same argument as above applies to show this is compact and self-adjoint. Show that F : L2 (S) C 0 (S). Solution. The series for Sf 1 1 2 Sf (x) = (2k + 1) 2 (f, eikx )eikx 2
k

converges absolutely and uniformly, using Cauchys inequality for instance it is Cauchy in the supremum norm: 1 (21.59) (2k 2 + 1) 2 (f, eikx )eikx | f L2
|k|>p

for p large since the sum of the squares of the eigenvalues is nite. (10) Now, going back to the real equation (21.45), we assume that V is contin uous, real-valued and 2-periodic. Show that if u is a twice-dierentiable 2-periodic function satisfying (21.45) for a given f C 0 (S) then (21.60) u + S((V 1)u) = Sf and hence u = F 2 ((V 1)u) + F 2 f and hence conclude that (21.61) u = F v where v L2 (S) satises v + (F (V 1)F )v = F f where V 1 is the operator dened by multiplication by V 1. Solution: If u satises (21.45) then (21.62) d2 u(x) + u(x) = (V (x) 1)u(x) + f (x) dx2

so by the uniquenss of the solution with periodic boundary conditions, u = S(V 1)u + Sf so u = F (F (V 1)u + F f ). Thus indeed u = F v with v = F (V 1)u + F f which means that v satises (21.63) v + F (V 1)F v = F f.

(11) Show the converse, that if v L2 (S) satises (21.64) v + (F (V 1)F )v = F f, f C 0 (S) then u = F v is 2-periodic and twice-dierentiable on R and satises (21.45). Solution. If v L2 (0, 2) satises (21.64) then u = F v C 0 (S) satises u+F 2 (V 1)u = F 2 f and since F 2 = S maps C 0 (S) into twice continuously dierentiable functions it follows that u satises (21.45).

130

LECTURE NOTES FOR 18.102, SPRING 2009

(12) Apply the Spectral theorem to F (V 1)F (including why it applies) and show that there is a sequence j in R \ {0} with |j | 0 such that for all C \ {0}, the equation (21.65) v + (F (V 1)F )v = g, g L2 (S) has a unique solution for every g L2 (S) if and only if = j for any j. Solution: We know that F (V 1)F is self-adjoint and compact so L2 (0.2) has an orthonormal basis of eigenfunctions of F (V 1)F with eigenvalues j . This sequence tends to zero and (21.65), for given C \ {0}, if and only if has a solution if and only if it is an isomorphism, meaning = j is not an eigenvalue of F (V 1)F. (13) Show that for the j the solutions of (21.66) j v + (F (V 1)F )v = 0, v L2 (S),

are all continuous 2-periodic functions on R. Solution: If v satises (21.66) with j = 0 then v = F (V 1)F/j C 0 (S). (14) Show that the corresponding functions u = F v where v satises (21.66) are all twice continuously dierentiable, 2-periodic functions on R satisfying (21.67) d2 u + (1 sj + sj V (x))u(x) = 0, sj = 1/j . dx2 Solution: Then u = F v satises u = S(V 1)u/j so is twice contin uously dierentiable and satises (21.67). (15) Conversely, show that if u is a twice continuously dierentiable, 2-periodic function satisfying d2 u + (1 s + sV (x))u(x) = 0, s C, dx2 and u is not identically 0 then s = sj for some j. Solution: From the uniquess of periodic solutions u = S(V 1)u/j as before. (16) Finally, conclude that Fredholms alternative holds for the equation (21.45) Theorem 16. For a given real-valued, continuous 2-periodic function V on R, either (21.45) has a unique twice continuously dierentiable, 2 periodic, solution for each f which is continuous and 2-periodic or else there exists a nite, but positive, dimensional space of twice continuously dierentiable 2-periodic solutions to the homogeneous equation (21.69) d2 w(x) + V (x)w(x) = 0, x R, dx2 and (21.45) has a solution if and only if (0,2) f w = 0 for every 2-periodic solution, w, to (21.69).

(21.68)

Solution: This corresponds to the special case j = 1 above. If j is not an eigenvalue of F (V 1)F then (21.70) v + F (V 1)F v = F f

has a unque solution for all f, otherwise the necessary and sucient condition is that (v, F f ) = 0 for all v satisfying v + F (V 1)F v = 0. Correspondingly either

LECTURE NOTES FOR 18.102, SPRING 2009

131

(21.45) has a unique solution for all f or the necessary and sucient condition is that (F v , f ) = 0 for all w = F v (remember that F is injetive) satisfying (21.69). Not to be handed in, just for the enthusiastic Check that we really can understand all the 2 periodic eigenfunctions of the Schrdinger operator using the discussion above. First of all, there was nothing o sacred about the addition of 1 to d2 /dx2 , we could add any positive number and get a similar result the problem with 0 is that the constants satisfy the homogeneous equation d2 u/dx2 = 0. What we have shown is that the operator d2 u u+Vu dx2 applied to twice continuously dierentiable functions has at least a left inverse unless there is a non-trivial solution of d2 u (21.72) 2 u + V u = 0. dx Namely, the left inverse is R = F (Id +F (V 1)F )1 F. This is a compact self-adjoint operator. Show and there is still a bit of work to do that (twice continuously dierentiable) eigenfunctions of Q, meaning solutions of Qu = u are precisely the non-trivial solutions of Ru = 1 u. What to do in case (21.72) does have a non-trivial solution? Show that the space of these is nite dimensional and conclude that essentially the same result holds by working on the orthocomplement in L2 (S). (21.71) u Qu =

MIT OpenCourseWare http://ocw.mit.edu

18.102 Introduction to Functional Analysis


Spring 2009

For information about citing these materials or our Terms of Use, visit: http://ocw.mit.edu/terms.

132

LECTURE NOTES FOR 18.102, SPRING 2009

Lecture 22. Thursday April 30: Dirchlet problem continued I did not nish the proof last time: Proof. Notice the form of the solution in case V 0 in (21.25). In general, we can choose a constant c such that V + c 0. Then the equation d2 w d2 w + V w = T wk 2 + (V + c)w = (T + c)w. 2 dx dx Thus, if w satises this eigen-equation then it also satises (22.1) (22.2) w = (T + c)A(Id +A(V + c)A)1 Aw Sw = (T + c)1 w, S = A(Id +A(V + c)A)1 A. Now, we have shown that S is a compact self-adjoint operator on L2 (0, 2) so we know that it has a complete set of eigenfunctions, ek , with eigenvalues k = 0. From the discussion above we then know that each ek is actually continuous since it is Aw with w L2 (0, 2) and hence also twice continuously dierentiable. So indeed, these ek satisfy the eigenvalue problem (with Dirichlet boundary conditions) with eigenvalues (22.3)
1 Tk = k + c as k .

The solvability part also follows much the same way.

MIT OpenCourseWare http://ocw.mit.edu

18.102 Introduction to Functional Analysis


Spring 2009

For information about citing these materials or our Terms of Use, visit: http://ocw.mit.edu/terms.

LECTURE NOTES FOR 18.102, SPRING 2009

133

Lecture 23. Tuesday, May 5: Harmonic oscillator As a second serious application of at least the completeness part of the spectral theorem for self-adjoint compact operators, I want to discuss the Hermite basis for L2 (R). Note that so far we have not found an explicit orthonormal basis on the whole real line, even though we know L2 (R) to be separable, so we certainly know that such a basis exists. How to construct one explicitly and with some handy properties? One way is to simply orthonormalize using Gramm-Schmidt some countable set with dense span. For instance consider the basic Gaussian function x2 ) L2 (R). 2 This is so rapidly decreasing at innity that the product with any polynomial is also square integrable: (23.1) exp( x2 ) L2 (R) k N0 = {0, 1, 2, . . . }. 2 Orthonormalizing this sequence gives an orthonormal basis, where completeness can be shown by an appropriate approximation technique. Rather than proceed directly we will discuss the invertibility of the harmonic oscillator d2 (23.3) H = 2 + x2 dx which we want to think of as an operator although for the moment I will leave vague the question of what it operates on. The rst thing to observe is that the Gaussian is an eigenfunction of H (23.2) xk exp( (23.4) Hex
2

/2

2 2 d (xex /2 + x2 ex /2 dx 2 2 2 (x2 1)ex /2 + x2 ex /2 = ex /2

with eigenvalue 1 for the moment this is only in a formal sense. In this special case there is an essentially algebraic way to generate a whole sequence of eigenfunctions from the Gaussian. To do this, write (23.5) Hu = ( d d + x)( + x)u + u = (CA + 1)u, dx dx C = ( again formally as operators. Then note that (23.6) Aex
2

d d + x), A = ( + x) dx dx

/2

=0

which again proves (23.4). The two operators in (23.5) are the creation operator and the annihilation operator. They almost commute in the sense that (23.7) (AC CA)u = 2u for say any twice continuously dierentiable function u. 2 Now, set u0 = ex /2 which is the ground state and consider u1 = Cu0 . From (23.7), (23.6) and (23.5), (23.8) Hu1 = (CAC + C)u0 = C 2 Au0 + 3Cu0 = 3u1 . Thus, u1 is an eigenfunction with eigenvalue 3.

134

LECTURE NOTES FOR 18.102, SPRING 2009

Lemma 18. For j N0 = {0, 1, 2, . . . } the function uj = C j u0 satises Huj = (2j + 1)uj . Proof. This follows by induction on j, where we know the result for j = 0 and j = 1. Then (23.9) HCuj = (CA + 1)Cuj = C(H 1)uj + 3Cuj = (2j + 3)uj . Again by induction we can check that uj = (2j xj + qj (x))ex /2 where qj is a polynomial of degree at most j 2. Indeed this is true for j = 0 and j = 1 (where q0 = q1 0) and then (23.10) Cuj = (2j+1 xj+1 + Cqj )ex
2 2

/2

and qj+1 = Cqj is a polynomial of degree at most j 1 one degree higher than qj . From this it follows in fact that the nite span of the uj consists of all the 2 products p(x)ex /2 where p(x) is any polynomial. Now, all these functions are in L2 (R) and we want to compute their norms. First, a standard integral computation3 shows that 2 x2 /2 2 (23.11) (e ) = ex =
R R

For j > 0, integration by parts (easily justied by taking the integral over [R, R] and then letting R ) gives (23.12) (C j u0 )2 = C j u0 (x)C j u0 (x)dx = u 0 Aj C j u 0 .
R R R

Now, from (23.7), we can move one factor of A through the j factors of C until it emerges and kills u0 (23.13) AC j u0 = 2C j1 u+ CAC j1 u0 = 2C j1 u0 + C 2 AC j2 u0 = 2jC j1 u0 . (23.14)
R

So in fact, (C j u0 )2 = 2j
R

(C j1 u0 )2 = 2j j! .

A similar argument shows that (23.15) uk uj = u0 Ak C j u0 = 0 if k = j.


R R

Thus the functions (23.16) ej = 2j/2 (j!) 2 4 C j ex


1 1 2

/2

form an orthonormal sequence in L2 (R).


3To compute the Gaussian integral, square it and write as a double integral then introduct polar coordinates Z Z Z Z 2 2 2 2 2 2 ( ex dx)2 = ex y dxdy = er rdrd = er 0 = .
R R2 0 0

LECTURE NOTES FOR 18.102, SPRING 2009

135

We would like to show this is orthonormal sequence is complete. Rather than argue through approximation, we can guess that in some sense the operator d d d2 + x)( + x) = 2 + x2 + 1 dx dx dx should be invertible, so one approach ist to try to construct its inverse and show this really is a compact, self-adjoint operator on L2 (R) and that its only eigenfunc tions are the ei in (23.16). Rather than do this I will proceed more indirectly. (23.17) AC = (

136

LECTURE NOTES FOR 18.102, SPRING 2009

Solutions to Problem set 10 Problem P10.1 Let H be a separable, innite dimensional Hilbert space. Show that the direct sum of two copies of H is a Hilbert space with the norm (23.18)
2 H H (u1 , u2 ) (u1 2 + u2 H ) 2 H
1

either by constructing an isometric isomorphism (23.19) T : H H H, 1-1 and onto, uH = T uHH

or otherwise. In any case, construct a map as in (23.19). Solution: Let {ei }iN be an orthonormal basis of H, which exists by virtue of the fact that it is an innite-dimensional but separable Hilbert space. Dene the map (23.20)
T : H u ( (u, e2i1 )ei , (u, e2i )ei ) H H i=1 i=1

The convergence of the Fourier Bessel series shows that this map is well-dened and linear. Injectivity similarly follows from the fact that T u = 0 in the image implies that (u, ei ) = 0 for all i and hence u = 0. Surjectivity is also clear from the fact that (23.21) S : H H (u1 , u2 ) ((u1 , ei )e2i1 + (u2 , ei )e2i ) H
i=1

is a 2-sided inverse and Bessels identity implies isometry since S(u1 , u2 )2 = u1 2 + u2 2 Problem P10.2 One can repeat the preceding construction any nite number of times. Show that it can be done countably often in the sense that if H is a separable, innite dimensional, Hilbert space then (23.22) l2 (H) = {u : N H; u22 (H) = ui 2 < } H l
i

has a Hilbert space structure and construct an explicit isometric isomorphism from l2 (H) to H. Solution: A similar argument as in the previous problem works. Take an orthor mal basis ei for H. Then the elements Ei,j l2 (H), which for each i, i consist of the sequences with 0 entries except the jth, which is ei , given an orthonromal basis for l2 (H). Orthormality is clear, since with the inner product is (23.23) (u, v)l2 (H) = (uj , vj )H .
j

Completeness follows from completeness of the orthonormal basis of H since if v = {vj } (v, Ej,i ) = 0 for all j implies vj = 0 in H. Now, to construct an isometric isomorphism just choose an isomorphism m : N2 N then (23.24) T u = v, vj = (u, em(i,j) )ei H.
i

I would expect you to go through the argument to check injectivity, surjectivity and that the map is isometric.

LECTURE NOTES FOR 18.102, SPRING 2009

137

Problem P10.3 Recall, or perhaps learn about, the winding number of a closed curve with values in C = C \ {0}. We take as given the following fact:4 If Q = [0, 1]N and f : Q C is continuous then for each choice of b C satisfying exp(2ib) = f (0), there exists a unique continuous function F : Q C satisfying (23.25) exp(2iF (q)) = f (q), q Q and F (0) = b. Of course, you are free to change b to b + n for any n Z but then F changes to F + n, just shifting by the same integer. (1) Now, suppose c : [0, 1] C is a closed curve meaning it is continuous and c(1) = c(0). Let C : [0, 1] C be a choice of F for N = 1 and f = c. Show that the winding number of the closed curve c may be dened unambiguously as (23.26)

wn(c) = C(1) C(0) Z.

Solution: Let C , be another choice of F in this case. Now, g(t) = C (t) C(t) is continuous and satises exp(2g(t)) = 1 for all t [0, 1] so by the uniqueness must be constant, thus C (1) C (0) = C(1) C(0) and the winding number is well-dened. (2) Show that wn(c) is constant under homotopy. That is if ci : [0, 1] C , i = 1, 2, are two closed curves so ci (1) = ci (0), i = 1, 2, which are homotopic through closed curves in the sense that there exists f : [0, 1]2 C continuous and such that f (0, x) = c1 (x), f (1, x) = c2 (x) for all x [0, 1] and f (y, 0) = f (y, 1) for all y [0, 1], then wn(c1 ) = wn(c2 ). Solution: Choose F using the fact corresponding to this homotopy f. Since f is periodic in the second variable the two curves f (y, 0), and f (y, 1) are the same so by the uniquess F (y, 0) F (y, 1) must be constant, hence wn(c2 ) = F (1, 1) F (1, 0) = F (0, 1) F (0, 0) = wn(c1 ). (3) Consider the closed curve Ln : [0, 1] x e2ix Idnn of n n matrices. Using the standard properties of the determinant, show that this curve is not homotopic to the identity through closed curves in the sense that there does not exist a continuous map G : [0, 1]2 GL(n), with values in the invertible n n matrices, such that G(0, x) = Ln (x), G(1, x) Idnn for all x [0, 1], G(y, 0) = G(y, 1) for all y [0, 1]. Solution: The determinant is a continuous (actually it is analytic) map which vanishes precisely on non-invertible matrices. Moreover, it is given by the product of the eigenvalues so (23.27) det(Ln ) = exp(2ixn). This is a periodic curve with winding number n since it has the lift xn. Now, if there were to exist such an homotopy of periodic curves of matrices, always invertible, then by the previous result the winding number of the determinant would have to remain constant. Since the winding number for the constant curve with value the identity is 0 such an homotopy cannot exist. Problem P10.4 Consider the closed curve corresponding toLn above in the case of a separable but now innite dimensional Hilbert space: (23.28) L : [0, 1] x e2ix IdH GL(H) B(H)

4Of course, you are free to give a proof it is not hard.

138

LECTURE NOTES FOR 18.102, SPRING 2009

taking values in the invertible operators on H. Show that after identifying H with H H as above, there is a continuous map (23.29) M : [0, 1]2 GL(H H)

with values in the invertible operators and satisfying (23.30)


M (0, x) = L(x), M (1, x)(u1 , u2 ) = (e4ix u1 , u2 ), M (y, 0) = M (y, 1), x, y [0, 1].
Hint: So, think of H H as being 2-vectors (u1 , u2 ) with entries in H. This allows one to think of rotation between the two factors. Indeed, show that (23.31) U (y)(u1 , u2 ) = (cos(y/2)u1 + sin(y/2)u2 , sin(y/2)u1 + cos(y/2)u2 ) denes a continuous map [0, 1] y U (y) GL(H H) such that U (0) = Id, U (1)(u1 , u2 ) = (u2 , u1 ). Now, consider the 2-parameter family of maps (23.32) U 1 (y)V2 (x)U (y)V1 (x) where V1 (x) and V2 (x) are dened on H H as multiplication by exp(2ix) on the rst and the second component respectively, leaving the other xed. Solution: Certainly U (y) is invertible since its inverse is U (y) as follows in the two dimensional case. Thus the map W (x, y) on [0, 1]2 in (23.32) consists of invertible and bounded operators on H H, meaning a continuous map W : [0, 1]2 GL(H H). When x = 0 or x = 1, both V1 (x) and v2 (x) reduce to the identiy, and hence W (0, y) = W (1, y) for all y, so W is periodic in x. Moreove at y = 0 W (x, 0) = V2 (x)V1 (x) is exactly L(x), a multiple of the identity. On the other hand, at x = 1 we can track composite as 2ix 4x u1 e u1 u2 u2 e u1 (23.33) . u2 u2 u2 e2x u1 e4x u1 This is what is required of M in (23.30). Problem P10.5 Using a rotation similar to the one in the preceeding problem (or otherwise) show that there is a continuous map (23.34) such that (23.35) G(0, x)(u1 , u2 ) = (e2ix u1 , e2ix u2 ), G(1, x)(u1 , u2 ) = (u1 , u2 ), G(y, 0) = G(y, 1) x, y [0, 1]. Solution: We can take (23.36) 2ix Id 0 e G(y, x) = U (y) U (y) 0 e2ix 0 0 . Id G : [0, 1]2 GL(H H)

By the same reasoning as above, this is an homotopy of closed curves of invertible operators on H H which satises (23.35). Problem P10.6 Now, think about combining the various constructions above in the following way. Show that on l2 (H) there is an homotopy like (23.34), G : [0, 1]2 GL(l2 (H)), (very like in fact) such that (23.37) G(0, x) {uk }k=1 = exp((1)k 2ix)uk k=1 , G(1, x) = Id, G(y, 0) = G(y, 1) x, y [0, 1].

LECTURE NOTES FOR 18.102, SPRING 2009

139

Solution: We can divide l2 (H) into its odd an even parts (23.38) D : l2 (H) v ({v2i1 }, {v2i }) l2 (H) l2 (H) H H. and then each copy of l2 (H) on the right with H (using the same isometric isomor phism). Then the homotopy in the previous problem is such that (23.39) G(x, y) = D1 G(y, x)D accomplishes what we want. Problem P10.7: Eilenbergs swindle For an innite dimenisonal separable Hilbert space, construct an homotopy meaning a continuous map G : [0, 1]2 GL(H) with G(0, x) = L(x) in (23.28) and G(1, x) = Id and of course G(y, 0) = G(y, 1) for all x, y [0, 1]. Hint: Just put things together of course you can rescale the interval at the end to make it all happen over [0, 1]. First divide H into 2 copies of itself and deform from L to M (1, x) in (23.30). Now, divide the second H up into l2 (H) and apply an argument just like the preceding problem to turn the identity on this factor into alternating terms multiplying by exp(4ix) starting with . Now, you are on H l2 (H), renumbering allows you to regard this as l2 (H) again and when you do so your curve has become alternate multiplication by exp(4ix) (with + rst). Finally then, apply the preceding problem again, to deform to the identity (always of course through closed curves). Presto, Eilenbergs swindle! Solution: By rescaling the variables above, we now have three homotopies, always through periodic families. On H H between L(x) = e2ix Id and the matrix 4ix e Id 0 (23.40) . 0 Id Then on H l2 (H) we can deform from 4ix 4ix Id 0 e e Id 0 (23.41) to 0 Id 0 G(0, x) with G(0, x) in (23.37). However we can then identify (23.42) H l2 (H) = l2 (H), (u, v) w = {wj }, w1 = u, wj+1 = vj , j 1. This turns the matrix of operators in (23.41) into G(0, x)1 . Now, we can apply the same construction to deform this curve to the identity. Notice that this really does ultimately give an homotopy, which we can renormalize to be on [0, 1] if you insist, of curves of operators on H at each stage we transfer the homotopy back to H.

MIT OpenCourseWare http://ocw.mit.edu

18.102 Introduction to Functional Analysis


Spring 2009

For information about citing these materials or our Terms of Use, visit: http://ocw.mit.edu/terms.

140

LECTURE NOTES FOR 18.102, SPRING 2009

Lecture 24. Thursday, May 7: Completeness of Hermite basis Here is what I claim was done last time. Starting from the ground state for the harmonic oscillator (24.1) H=
2 d2 + x2 , Hu0 = u0 , u0 = ex /2 dx2

and using the creation and annihilation operators (24.2) A= d d + x, C = + x, AC CA = 2 Id, H = CA + Id dx dx

I examined the higher eigenfunctions: (24.3) uj = C j u0 = pj (x)u0 (c), p(x) = 2j xj + l.o.ts, Huj = (2j + 1)uj

and showed that these are orthogonal, uj uk , j = k, and so when normalized give an orthonormal system in L2 (R) : (24.4) ej = uj
1 1 2j/2 (j!) 2 4

Now, what I want to show today, and not much more, is that the ej form an orthonormal basis of L2 (R), meaning they are complete as an orthonormal sequence. There are various proofs of this, but the only simple ones I know involve the Fourier inversion formula and I want to use the completeness to prove the Fourier inversion formula, so that will not do. Instead I want to use a version of Mehlers formula. I also tried to motivate this a bit last time. Namely, I suggested that to show the completeness of the ej s it is enough to nd a compact self-adjoint operator with these as eigenfunctions and no null space. It is the last part which is tricky. The rst part is easy. Remembering that all the ej are real, we can nd an operator with the ej ;s as eigenfunctions with corresponding eigenvalues j > 0 (say) by just dening (24.5) Au(x) = j (u, ej )ej (x) = j ej (x) ej (y)u(y).
j=0 j=0

For this to be an operator we need j 0 as j , although for convergence we just need the j to be bounded. So, the problem with this is to show that A has no null space which of course is just the completeness of the e since (assuming j all the j are positive) (24.6) Au = 0 u ej j.

Nevertheless, this is essentially what we will do. The idea is to write A as an integral operator and then work with that. I will take the j = wj where w [0, 1). The point is that we can nd an explicit formula for (24.7) Aw u =
j=0

wj ej (x)ej (y) = A(w, x, y).

I struggled a bit with this in class but did pretty much manage to do it.

LECTURE NOTES FOR 18.102, SPRING 2009

141

To nd A(w, x, y) we use some other things I did last time. First, I dened the Fourier transform and showed its basic propertyL
0 (24.8) F : L1 (R) C (R), F(u) = u,

u() =

eix u(x), sup |u| uL1 .

Then I computed the Fourier transform of u0 , namely (24.9) (Fu0 )() = 2u0 (). Now, we can use this formula, of if you like the argument to prove it, to show that 2 2 (24.10) v = ex /4 = v = e . Changing the names of the variables this just says 2 1 x2 (24.11) e = eixss /4 ds. 2 R Now, again as I discussed last time, the denition of the uj s can be rewritten
2 2 2 d d + x)j ex /2 = ex /2 ( )j ex . dx dx Plugging this into (24.11) and carrying out the derivatives which is legitimate since the integral is so strongly convergent gives 2 2 ex /2 (24.13) uj (x) = (is)j eixss /4 ds. 2 R

(24.12)

uj (x) = (

Now we can use this formula twice on the sum on the left in (24.7) and insert the normalizations in (24.4) to nd that ex2 /2+y2 /2 (1)j wj sj tj 2 2 j (24.14) w ej (x)ej (y) = eisx+itys /4t /4 dsdt. 2j j!
4 3/2 2 R j=0 j=0
The crucial thing here is that we can sum the series to get an exponential, this allows us to nally conclude: Lemma 19. The identity (24.7) holds with 1 1w 1+w 2 2 (24.15) A(w, x, y) = exp (x + y) (x y) 4(1 + w) 4(1 w) 1 w2 Proof. Summing the series in (24.14) we nd that 2 2 ex /2+y /2 1 (24.16) A(w, x, y) = exp( wst + isx + ity s2 /4 t2 /4)dsdt. 2 4 3/2 2 R Now, we can use the same formula as before for the Fourier transform of u0 to evaluate these integrals explicitly. I think the clever way, better than what I did in lecture, is to change variables by setting (24.17) s = (S + T )/ 2, t = (S T )/ 2 = dsdt = dSdT, 1 x+y 1 xy 1 wst + isx + ity s2 /4 t2 /4 = iS (1 + w)S 2 iT (1 w)T 2 . 2 4 4 2 2

142

LECTURE NOTES FOR 18.102, SPRING 2009

The formula for the Fourier transform of exp(x2 ) can be used, after a change of variable, to conclude that x + y 1 2 (x + y)2 exp(iS (1 + w)S 2 )dS = exp( ) 4 2(1 + w) 2 (1 + w) R (24.18) xy 1 2 (x y)2 2 exp(iT (1 w)T )dT = exp( ). 4 2(1 w) 2 (1 w) R Inserting these formul back into (24.16) gives 1 (x + y)2 (x y)2 x2 y2 (24.19) A(w, x, y) = exp + + 2(1 + w) 2(1 w) 2 2 1 w2 which after a little adjustment gives (24.15).

Now, this explicit representation of Aw as an integral operator allows us to show Proposition 31. For all real-valued f L2 (R), (24.20) Proof. By denition of Aw (24.21) so (24.20) reduces to (24.22)
w 1 j=1 j=1

|(u, ej )|2 = f 2 2 . L

|(u, ej )|2 = lim(f, Aw f )


w 1

lim(f, Aw f ) = f 2 2 . L

To prove (24.22) we will make our work on the integral operators rather simpler by assuming rst that f C 0 (R) is continuous and vanishes outside some bounded interval, f (x) = 0 in |x| > R. Then we can write out the L2 inner product as a doulbe integral, which is a genuine (iterated) Riemann integral: (24.23) (f, Aw f ) = A(w, x, y)f (x)f (y)dydx. Here I have used the fact that f and A are real-valued. Look at the formula for A in (24.15). The rst thing to notice is the factor (1 w2 ) 1 which blows up as w 1. On the other hand, the argument of the 2 exponential has two terms, the rst tends to 0 as w 1 and the second blows up, at least when x y = 0. Given the signs, we see that if > 0, X = {(x, y); |x| R, |y| R, |x y| } then (24.24) sup |A(w, x, y)| 0 as w 1.
X

So, the part of the integral in (24.23) over |x y| tends to zero as w 1. So, look at the other part, where |x y| . By the (uniform) continuity of f, given > 0 there exits > 0 such that (24.25) |x y| = |f (x) f (y)| .

LECTURE NOTES FOR 18.102, SPRING 2009

143

Now we can divide (24.23) up into three pieces: (24.26) (f, Aw f ) = A(w, x, y)f (x)f (y)dydx S{|xy|} + A(w, x, y)(f (x) f (y))f (y)dydx S{|xy|} + A(w, x, y)f (y)2 dydx
S{|xy|}

where S = [R, R]2 . Look now at the third integral in (24.26) since it is the important one. We can 1+w change variable of integration from x to t = 1w (x y) and then this becomes 1w A(w, y + t , y)f (y)2 dydt, 1+w S{|xy|} (24.27) A(w, y+t 1 w , y) 1+w 2 1 1w t 2 = exp (2y + t 1 w) exp . 4 4(1 + w) (1 + w) Here y is bounded; the rst exponential factor tends to 1 so it is straightforward to show that for any > 0 the third term in (24.26) tends to 2 (24.28) f 2 2 as w 1 since et /4 = 2 . L Noting that A 0 the same sort of argument shows that the second term is bounded by a constant multiple of . So this proves (24.22) (rst choose then ) and hence (24.20) under the assumption that f is continuous and vanishes outside some interval [R, R]. However, the general case follows by continuity since such continuous functions vanishing outside compact sets are dense in L2 (R) and both sides of (24.20) are continuous in f L2 (R). Now, (24.22) certainly implies that the ej form an orthonormal basis, which is what we wanted to show but hard work! I did it really to remind you of how we did the Fourier series computation of the same sort and to suggest that you might like to compare the two arguments.

MIT OpenCourseWare http://ocw.mit.edu

18.102 Introduction to Functional Analysis


Spring 2009

For information about citing these materials or our Terms of Use, visit: http://ocw.mit.edu/terms.

144

LECTURE NOTES FOR 18.102, SPRING 2009

Lecture 25. Tuesday, May 12: Fourier transform Last time I showed the completeness of the orthonormal sequence formed by the eigenfunctions of the harmonic oscillator. This allows us to prove some basic facts about the Fourier transform, which we already know is a linear operator 0 (25.1) L1 (R) C (R), u() = eix u(x)dx. Namely we have already shown the eect of the Fourier transform on the ground state: (25.2) F(u0 )() = 2e0 (). By a similar argument we can check that (25.3) F(uj )() = 2ij uj () j N. As usual we can proceed by induction using the fact that uj = Cuj1 . The integrals involved here are very rapidly convergent at innity, so there is no problem with the integration by parts in (25.4) T d duj1 F( uj1 ) = lim eix dx T T dx dx T ix T ix = lim (i)e uj1 dx + e uj1 (x) T = (i)F(uj1 ),
T T

F(xuj1 ) = i

deix d uj1 dx = i F(uj1 ). d d

Taken together these identities imply the validity of the inductive step: d d (25.5) F(uj ) = F(( + x)uj1 ) = (i( + )F(uj1 ) = iC( 2ij1 uj1 ) dx d so proving (25.3). So, we have found an orthonormal basis for L2 (R) with elements which are all in L1 (R) and which are also eigenfunctions for F. Theorem 17. The Fourier transform maps L1 (R) L2 (R) into L2 (R) and extends by continuity to an isomorphism of L2 (R) such that 1 F is unitary with the 2 inverse of F the continuous extension from L1 (R) L2 (R) of 1 (25.6) F(f )(x) = eix f (). 2 Proof. This really is what we have already proved. The elements of the Hermite basis ej are all in both L1 (R) and L2 (R) so if u L1 (R) L2 (R) its image under F because we can compute the L2 inner products and see that ix (25.7) (F(u), ej ) = ej ()e u(x)dxd = F(ej )(x)u(x) = 2ij (u, ej ).
R2

Now Bessels inequality shows that F(u) L2 (R) (it is of course locally integrable) since it is continuous). Everything else now follows easily.

LECTURE NOTES FOR 18.102, SPRING 2009

145

Notic in particular that we have also proved Parsevals and Plancherels identities: (25.8) F(u)L2 = 2uL2 , (F(u), F(v)) = 2(u, v), u, v L2 (R). Since we are at the end of the course, more or less, I do not have any time to remind you of the many applications of the Fourier transform. However, let me just indicate the denitions of Sobolev spaces and Schwartz space and how they are related to the Fourier transform. First Sobolev spaces. We now see that F maps L2 (R) isomorphically onto L2 (R) and we can see from (25.4) for instance that it turns dierentiations by x into multiplication by . Of course we do not know how to dierentiate L2 functions so we have some problems making sense of this. One way, the usual mathematicians trick, is to turn what we want into a denition. Denition 10. The the Sobolev spaces of order s, for any s (0, ), are dened as subspaces of L2 (R) : (25.9) H s (R) = {u L2 (R); (1 + ||2 ) 2 u L2 (R)}.
s

It is natural to identify H 0 (R) = L2 (R). Exercise 1. Show that the Sobolev spaces of positive order are Hilbert spaces with the norm (25.10) us = (1 + ||2 ) 2 uL2 .
s

Having dened these spaces, which get smaller as s increases it can be shown for instances that if n s is an integer then the set of n times continuously dierentiabel functions on R which vanish outside a compact set are dense in H s . This allows us to justify, by continuity, the following statement: Proposition 32. The bounded linear map d du v () = iu() : H s (R) H s1 (R), s 1, v(x) = dx dx is consistent with dierentiation on n times continuously dierentiable functions of compact support, for any integer n s. (25.11) One of the more important results about Sobolev spaces of which there are many is the relationship between these L2 derivatives and true derivatives. Theorem 18 (Sobolev embedding). If n is an integer and s > n + (25.12) H (R)
s n C (R) 1 2

then

consists of n times continuosly dierentiable functions with bounded derivatives to order n. This is actually not so hard to prove. These are not the only sort of spaces with more regularity one can dene and use. For instance one can try to treat x and more symmetrically and dene smaller spaces than the H s above by setting (25.13)
s Hiso (R) = {u L2 (R); (1 + ||2 ) 2 u L2 (R), (1 + |x|2 ) 2 u L2 (R)}.
s s

Exercise 2. Find a norm with respect to which these (isotropic Sobolev spaces) s Hiso (R) are Hilbert spaces.

146

LECTURE NOTES FOR 18.102, SPRING 2009

Exercise 3. (Probably too hard) Show that the harmonic oscillator extends by continuity to an isomorphism (25.14)
s+2 s H : Hiso (R) Hiso (R) s 2.

Finally in this general vein, I wanted to point out that Hilbert, and even Banach, spaces are not the end of the road! One very important space in relation to a direct treatment of the Fourier transform, is the Schwartz space. The denition is reasonably simple. Namely we denote Schwartz space by S(R) and say u S(R) u : R C (25.15) is continuously dierentiable of all orders and for every n and , dp u un = sup(1 + |x|)k | p | < . dx xR
k+pn

All these inequalities just mean that all the derivatives of u are rapidly decreasing at in the sense that they stay bounded when multiplied by any polynomial. So in fact we know already that S(R) is not empty since the elements of the Hermite basis, ej S(R) for all j. As you can see from the denition in (25.15) this is not likely to be a Banach space. Each of the n is a norm. However, S(R) is pretty clearly not going to be complete with respect to any one of these. However it is complete with respect to all, countably many, norms. What does this mean? In fact S(R) is a metric space with the metric u vn (25.16) d(u, v) = 2n 1 + u vn n as you can check, with some thought perhaps required. So the claim is that S(R) is comlete as a metric space such a thing is called a Frchet space. e What has this got to do with the Fourier transform? The point is that (25.17) du dF(u) F : S(R) S(R) is an isomorphism and F( ) = iF(u), F(xu) = i dx d where this now makes sense. The Sobolev embedding theorem implies that s (25.18) S(R) = Hiso (R).
n

The dual space of S(R) is the space, denoted S (R), of tempered distributions on R.

MIT OpenCourseWare http://ocw.mit.edu

18.102 Introduction to Functional Analysis


Spring 2009

For information about citing these materials or our Terms of Use, visit: http://ocw.mit.edu/terms.

LECTURE NOTES FOR 18.102, SPRING 2009

147

Lecture 26. Thursday, May 14:Review Now, there was one nal request before I go through a quick review of what we have done. Namely to state and prove the Hahn-Banach Theorem. This is about extension of functionals. Stately starkly, the basic question is: Does a normed space have any non-trivial continuous linear functionals on it? That is, is the dual space always non-trivial (of course there is always the zero linear functional but that is not very amusing). We did not really encounter this problem since for a Hilbert space, or even a pre-Hilbert space, there is always the space itse, giving continuous linear functionals through the pairing Riesz Theorem says that in the case of a Hilbert space that is all there is. I could have used the Hahn-Banach Theorem to show that any normed space has a completion, but I gave a more direct argument for this, which was in any case much more relevant for the cases of L1 (R) and L2 (R) for which we wanted concrete completions. Theorem 19 (Hahn-Banach). If M V is a linear subspace of a normed space and u : M C is a linear map such that (26.1) |u(t)| CtV t M

then there exists a bounded linear functional U : V C with U C and U M = u. First, by computation, we show that we can extend any continuous linear func tional a little bit without increasing the norm. Lemma 20. Suppose M V is a subspace of a normed linear space, x M / and u : M C is a bounded linear functional as in (26.1) then there exists u : M = {t V ; t = t + ax, a C such that (26.2) u M = u, |u (t + ax)| Ct + axV , t M, a C. Proof. Note that the decompositon t = t + ax of a point in M is unique, since t + ax = t + ax implies (a a)x M so a = a, since x M and hence t = t as / well. Thus (26.3) u (t + ax) = u (t) + au(x) = u(t) + a, = u (x)

and all we have at our disposal is the choice of . Any choice will give a linear functional extending u, the problem of course is to arrange the continuity estimate without increasing the constant. In fact if C = 0 then u = 0 and we can take the zero extension. So we might as well assume that C = 1 since dividing u by C arranges this and if u extends u/C then Cu extends u and the norm estimate in (26.2) follows. So we are assuming that (26.4) |u(t)| tV t M.

We want to choose so that (26.5) |u(t) + a| t + axV t M, a C.

Certainly when a = 0 this represents no restriction on . For a = 0 we can divide through by a and (26.5) becomes (26.6) t t |a||u( ) | = |u(t) + a| t + axV = |a| xV a a

148

LECTURE NOTES FOR 18.102, SPRING 2009

and since t/a M we only need to arrange that (26.7) |u(t) | t xV u M

and the general case follows. So, we will choose to be real. A complex linear functional such as u can be recovered from its real part, so set (26.8) w(t) = Re(u(t)) t M

and just try to extend w to a real functional it is not linear over the complex numbers of course, just over the reals, but what we want is the anaogue of (26.7): (26.9) |w(t) | t xV t M

which does not involve linearity. What we know about w is the norm estimate (26.4) which implies (26.10) |w(t1 ) w(t2 )| |u(t1 ) u(t2 )| t1 t2 t1 xV + t2 xV . w(t1 ) w(t2 ) t1 xV + t2 xV = w(t1 ) t1 x w(t2 ) + t2 xV t1 , t2 M.

Writing this out usual the reality we nd (26.11)

We can then take the sup on the right and the inf on the left and choose in between namely we have shown that there exists R with (26.12) w(t) t xV sup (w(t1 ) t1 x)
t2 M

inf (w(t1 ) + t1 x) w(t) + t xV t M.


t2 M

This in turn implies that (26.13) t xV w(t) + t xV = |w(t)| t xV t M. This is what we wanted we have extended the real part of u to (26.14) w (t + ax) = w(t) (Re a) and |w (t + ax)| t + axV . u (t + ax) = w (t + ax) iw (i(t + ax)).

Now, nally we get the extension of u itself by complexifying dening (26.15)

This is linear over the complex numbers since (26.16) u (z(t + ax)) = w (z(t + ax)) iw (iz(t + ax) = w (Re z(t + ax) + i Im z(t + ax)) iw (i Re z(t + ax)) + iw (Im z(t + ax)) = (Re z + i Im z)w (t + ax) i(Re z + i Im z)(w (i(t + ax)) = zu (t + ax). It certainly extends u from M since the same identity gives u in terms of its real part w. Finally then, to see the norm estimate note that (as we did long ago) there exists a uniqe [0, 2) such that (26.17) |u (t + ax)| = Re ei u (t + ax) = Re u (ei t + ei ax) = w (ei u + ei ax) ei (t + ax)V = t + axV .

This completes the proof of the Lemma.

LECTURE NOTES FOR 18.102, SPRING 2009

149

Of Hahn-Banach. This is an application of Zorns Lemma. I am not going to get into the derivation of Zorns Lemma from the Axiom of Choice, but if you believe the latter and you are advised to do so, at least before lunchtime you should believe the former. So, Zorns Lemma is a statement about partially ordered sets. A partial order on a set E is a subset of E E, so a relation, where the condition that (e, f ) be in the relation is written e f and it must satisfy (26.18) e e, e f and f e = e = f, e f and f g = e g.

So, the missing ingredient between this and an order is that two elements need not be related at all, either way. A subsets of a partially ordered set inherits the partial order and such a subset is said to be a chain if each pair of its elements is related one way or the other. An upper bound on a subset D E is an element e E such that d e for all d D. A maximal element of E is one which is not majorized, that is e f, f E, implies e = f. Lemma 21 (Zorn). If every chain in a (non-empty) partially ordered set has an upper bound then the set contains at least one maximal element. Now, we are given a functional u : M C dened on some linear subspace M V of a normed space where u is bounded with respect to the induced norm on M. We apply this to the set E consisting of all extensions (v, N ) of u with the same norm. That is, V N M must contain M, v M = u and vN = uM . This is certainly non-empty since it contains (u, M ) and has the natural partial order that (v1 , N1 ) (v2 , N2 ) if N1 N2 and v2 N = v1 . You can check that this is a partial 1 order. Let C be a chain in this set of extensions. Thus for any two elements (vi , N1 ) C, either (v1 , N1 ) (v2 , N2 ) or the other way around. This means that
(26.19) N = {N ; (v, N ) C for some v} V is a linear space. Note that this union need not be countable, or anything like that, but any two elements of N are each in one of the N s and one of these must be contained in the other by the chain condition. Thus each pair of elements of N is actually in a common N and hence so is their linear span. Similarly we can dene an extension (26.20) v : N C, v (x) = v(x) if x N, (v, N ) C.

There may be many pairs (v, N ) satisfying x N for a given x but the chain condition implies that v(x) is the same for all of them. Thus v is well dened, and is clearly also linear, extends u and satises the norm condition |v (x) uM vV . Thus (, N ) is an upper bound for the chain C. v So, the set of all extension E, with the norm condition, satises the hypothesis of Zorns Lemma, so must at least in the mornings have an maximal element (, M ). If M = V then we are done. However, in the contary case there exists u . This means we can apply our little lemma and construct an extension xV \M u u (u , M ) of (, M ) which is therefore also an element of E and satises (, M ) ) be maximal, so is (u , M ). This however contradicts the condition that (, M u forbidden by Zorn.

150

LECTURE NOTES FOR 18.102, SPRING 2009

There are many applications of Zorns Lemma, the main one being something like this: Proposition 33. For any normed space V and element x V there is a continuous linear functional f : V C with f (x) = 1 and f xV . Proof. Start with the one-dimensional space, M, spanned by x and dene u(zx) = z. This has norm xV . Extend it and you will get an admissible functional f. Now, nally the review!

You might also like